Вы находитесь на странице: 1из 141

PHARMACOLOGY RATIONALE

A.Y. 2017-2018
1ST LONG EXAM

QUESTION ANSWER & RATIONALE

1. A patient is on maintenance medication with Drug Answer: D


E, an enzyme inducer. Later, Drug M was added to his
therapeutic regimen. Which of the following would Inducers = ↑metabolism of substrate/another
be expected? drug = ↓half-life = ↓therapeutic
A. Drug E will decrease the Vd of Drug M effect/efficacy
B. Drug M will have longer half-life
C. Drug E will increase the protein binding of Inhibitors = ↓metabolism of substrate/another
Drug M drug = ↑half-life = ↑therapeutic
D. Drug M will have lower therapeutic effect effect/efficacy or ↑toxicity

2. Which of the following parameters is the most Answer: A


useful in designing drug dosage regimen? Dosage regimen is a plan for drug administration over a
A. Half-life period of time. Ideally, the dosing plan is based on
B. Bioavailability knowledge of both the minimum therapeutic and
C. Clearance (Cl) minimum toxic concentrations for the drug, as well as Cl
D. Apparent volume of distribution (Vd) and Vd. Half-life is completely determined by Vd and Cl.

3. Drug K has a half-life of 50 hours. You would Answer: C


expect the steady state concentration to be reached Plateau or steady state concentration (Css) is usually
after ___ days. reached in 4-5 t1/2….. t1/2 = 50 hours
A. 3-4 4 x 50 hours = 200 hours
B. 6-7 5 x 50 hours = 250 hours
C. 8-10 Then, convert to days…..
D. 12 200 hours/24 hours = 8.3333 days
250 hours/24 hours = 10.4167 days

4. Characteristics of drugs at risk for drug interaction: Answer: D


A. High therapeutic index Refer to #1
B. High Vd
C. Low slope of response curve
D. Metabolic enzyme inducer

5. This drug interaction produces an effect more than Answer: C


the sum of the drugs if given independently:
A. Competitive antagonism Summation/Addition: 1+1 = 2
B. Additive Synergism: 1+1 = >2
C. Synergism Potentiation: 0+1 = 2
D. Potentiation Antagonism: 1+2 = 1 or 1+0 = 0 or 1+1 = <2

Abraham, M. | Aquino, J. | Arada, M. | Aw, A. | Bagon, N. | Contreras, S. | Galapon, C.


Gison, R. | Obciana, D. | Palao, K. | Tangcueco, P. | Tumambing, M.
0
PHARMACOLOGY RATIONALE
A.Y. 2017-2018

6. Mechanisms of decreased oral bioavailability: Answer: B


A. Redistribution to peripheral tissues Low oral bioavailability (F) may be due to:
B. High intrinsic hepatic clearance ● Poor absorption
C. Metabolic enzyme induction ● Poorly formulated dosage form that fails to
D. Increased blood flow to gut wall disintegrate and dissolve in the GIT fluids
● Interaction between drug in the GIT
● Metabolism of drug in the GIT
● Poor intestinal permeability
● Hepatic extraction/First pass effect
○ Some drugs given PO pass through
liver via portal system to reach the
general circulation. In the liver,
metabolism happens… thus, the
concentration of the drug that
reaches circulation decreases → low F

7. This route can absolutely circumvent the first-pass Answer: A/B


effect: Hepatic first-pass effect can be avoided to great extent
A. Transdermal by use of sublingual (SL) tablets and transdermal preps,
B. Intramuscular and to a lesser extent by use of rectal suppositories. SL
C. Rectal absorption and transdermal route provides direct access
D. All of the above to systemic-not portal-veins. Drugs absorbed from
suppositories in the lower rectum enter vessels that
drain into the IVC, bypassing liver. However,
suppositories tend to move upward in the rectum into a
region where veins that lead to the liver predominate.
Only about 50% can be assumed to bypass the liver.

8. K.M. presents at the ER with nape pain and BP of Answer: B


150/90. He was given Drug P which lowered his BP Potency
to 120/80 at a dose of 50 mg. Ten days later, his BP ● Amount of drug required to produce a response
shoot up to 140/90 and was given Drug T, at a dose ● Measured by EC50 (conc. needed to produce 50%
of 100 mg which brought down his BP to 120/80. of maximal response)
Which of the following statements is TRUE? ● 1/EC50
A. Drug P is more efficacious than Drug T ● ↓EC50 = ↑Potency
B. Drug P is more potent than Drug T
C. Drug P and T are equipotent Both Drug P and Drug T produce the same response of
D. Drug P is efficacious as Drug T lowering the BP to 120/80, but Drug P is more potent
because it requires a lower dose (50mg) to lower the BP
compared to Drug T, which needs a higher dose of
100mg to produce the same response.

Abraham, M. | Aquino, J. | Arada, M. | Aw, A. | Bagon, N. | Contreras, S. | Galapon, C.


Gison, R. | Obciana, D. | Palao, K. | Tangcueco, P. | Tumambing, M.
1
PHARMACOLOGY RATIONALE
A.Y. 2017-2018

(#9-11) Given the PK data of Drug X in a 70 kg man: Answer: A


TC = 10 mg/L
Cl = 14 L/hr Vd = Dose/TC
Vd = 20 L Dose = VD x TC
Dose = 20 L x 10 mg/L
9. The dose to give to achieve the target Dose = 200 mg
concentration immediately is:
A. 200 mg
B. 140 mg
C. 2 mg
D. 1.4 mg

10. If you are to give Drug X by IV infusion, the Answer: B


dosing rate to achieve 10 mg/L concentration in a
70kg man is: Css = Infusion rate/Clearance
A. 280 mg/hr Infusion rate = Cl x Css
B. 140 mg/hr Infusion rate = 14 L/hr x 10 mg/L
C. 1.4 mg/hr Infusion rate = 140 mg/hr
D. 2 mg/hr

11. If the half-life of Drug X is 1 hr, the concentration Answer: A


in a 70 kg man 3 hrs post infusion is ___ mg/L: t1/2 (time taken for plasma concentration as well as the
A. 1.25 amount of drug in the body to fall by one-half) = 1 hour
B. 8.75 At 0 hour, Conc = 10 mg/L
C. 5 At 1 hour post-infusion, Conc = 5 mg/L
D. 2.5 At 2 hours, Conc = 2.5 mg/L
At 3 hours, Conc = 1.25 mg/L

(#12-13) Drug S at a dose of 75 mg produces Answer: A


sedation in 50% of individuals receiving the drug and
coma at a dose of 120 mg: Therapeutic Index (TI50) = LD50/ED50 or TD50/ED50
TD50 (dose required to produce a particular toxic effect
12. What is Drug S’s therapeutic index? in 50% of animals) = 120 mg
A. 1.6 ED50 (dose at which 50% of individuals exhibit the
B. 3.2 specified quantal effect) = 75 mg
C. 6.4 TI50 = 120mg/75 mg
D. 9.2 TI50 = 1.6

13. Is drug S safe to use and can be dispensed over- Answer: B


the-counter?
A. Yes TI50 = 1.6
B. No Narrow therapeutic index = ↑ toxicity

14. A 40 year old banker developed pruritic rash and Answer: B


tachycardia while eating green salad, tuna and rice. H1 receptor antagonists are often the 1st drugs used to
The most appropriate drug to give is: prevent or treat symptoms of allergic reaction. Causes
A. Beta2 agonist adrenoceptor blockade → ↓heart contractile force
B. H1 blocker
C. Beta1 agonist Beta1 agonist → ↑force and rate of contraction →
D. Steroid tachycardia

Abraham, M. | Aquino, J. | Arada, M. | Aw, A. | Bagon, N. | Contreras, S. | Galapon, C.


Gison, R. | Obciana, D. | Palao, K. | Tangcueco, P. | Tumambing, M.
2
PHARMACOLOGY RATIONALE
A.Y. 2017-2018

15. In which of the following clinical scenarios is H1 Answer: A


antagonist ineffective? Although H1 antagonists can block histamine-induced
A. 3 year old boy with bronchial asthma contraction of bronchiolar smooth muscle, it is not
B. 10 year old girl with chronic urticaria effective in asthma.
C. 5 year old girl with allergic rhinitis H1 antagonists indications:
D. 2 year old girl with atopic dermatitis ● DOC in urticaria
● 2nd line drugs for allergic rhinitis/hay fever (next
to glucocorticoids nasal spray - 1st line)
● Atopic dermatitis
● Hypersensitivity reactions
● Anaphylaxis/Anaphylactoid reactions

16. Some H1 antagonists have appetite stimulant Answer: D


effect caused by blockade of this receptor Serotonin blocking action → ↑ appetite
A. Cholinergic Antimuscarinic/cholinergic action: Atropine-like effects
B. Adrenoceptor ● Drying of secretions
C. Insulin ● Urinary retention
D. Serotonin ● Blurring of vision
● Anti-Parkinson effect
Adrenoceptor blocking action → orthostatic
hypotension

17. A 30 year old teacher was diagnosed with Answer: A


Meniere’s disease. She is about to ride a boat going
to Cebu. What drug can be given to her to prevent Drugs for motion sickness and vestibular disturbances
motion sickness? ● Scopolamine and some 1st gen H1 antagonists
A. Meclizine ● Diphenhydramine
B. Promethazine ● Promethazine → cause orthostatic
C. Levocetirizine hypotension
D. Desloratadine ● Piperazines (Cyclizine, Meclizine)

Abraham, M. | Aquino, J. | Arada, M. | Aw, A. | Bagon, N. | Contreras, S. | Galapon, C.


Gison, R. | Obciana, D. | Palao, K. | Tangcueco, P. | Tumambing, M.
3
PHARMACOLOGY RATIONALE
A.Y. 2017-2018

18. A 10 year old female was diagnosed with Acute Answer: C


Lymphocytic Leukemia. She is currently undergoing DRUG MOA INDICATIONS
chemotherapy. This 5-HT3 receptor antagonist can
be given to prevent nausea and vomiting: Buspirone 5-HT1A partial Anxiety,
A. Promethazine agonist Depression
B. Ranitidine
C. Ondansetron Sumatriptan 5-HT1B/1D Acute migraine,
(-triptan drugs) 1st choice for
D. Tegaserod
cluster headache

Phenoxybenzamine 5-HT2 Pheochromocyto


Also blocks ⍺1, ma

H1, Ach R

Cyproheptadine 5-HT2 Carcinoid


Also blocks syndrome, Cold
H1R urticaria,
Serotonin
syndrome

Ketanserin 5-HT2 Hypertension,


Also blocks ⍺1 vasospastic d/o

on vascular
smooth muscle
& platelets

Ondansetron 5-HT3 Prevention of n/v


(-setron drugs) antagonist

Cisapride 5-HT4 agonist GERD, motility


disorders

Tegaserod 5-HT4 partial IBD w/


agonist constipation
(TEGAS/TIGAS =
constipation)

19. A 30 year old man was diagnosed with Cluster Answer: A


Headache. This 5-HT1B/1D receptor agonist reduces Refer to #18
perivascular edema in the cerebral circulation:
A. Sumatriptan
B. Bromocriptine
C. Ergotamine
D. Celecoxib

Abraham, M. | Aquino, J. | Arada, M. | Aw, A. | Bagon, N. | Contreras, S. | Galapon, C.


Gison, R. | Obciana, D. | Palao, K. | Tangcueco, P. | Tumambing, M.
4
PHARMACOLOGY RATIONALE
A.Y. 2017-2018

20. This is a vasoactive peptide with vasoconstrictor Answer: C


effect: vasoCONSTRICTors vasoDILATors
A. Bradykinin
B. Neurotensin “AVENU” “KVSCANN”
C. Vasopressin Angiotensin II Kinins
D. Substance P Vasopressin VIP
Endothelin Substance P
Neuropeptide Y CGRP
Uropeptides Adrenomedullin
Natriuretic Peptide
Neurotensin

21. A 55 year old male was diagnosed with irritable Answer: B


bowel syndrome (IBS) for which he was given Refer to #18
Tegaserod. This drug acts as a partial agonist to this
receptor:
A. H1
B. 5-HT4
C. SERT
D. Alpha1

22. This physiologic antagonist to histamine Answer: A


receptors binds to alpha1 and alpha2 receptors
causing vasoconstriction: Epinephrine has smooth muscle actions opposite to
A. Epinephrine those of histamine, but they act at different receptors.
B. Cetirizine
C. Ranitidine
D. Diphenhydramine

23. A 3 year old girl has very itchy skin lesions which Answer: A
awakens her at night. The most effective Hydroxyzine - 1st gen H1-blocker, more sedation
antihistamine for the relief of itchiness is: Cetirizine - new gen, metabolite of hydroxyzine, less
A. Hydroxyzine sedation
B. Levocetirizine Levocetirizine - new gen, enantiomer of cetirizine, less
C. Cetirizine sedation
D. Loratadine Loratadine - less or non-sedating

24. A 70 year old man was prescribed H1 Answer: D


antihistamine which resulted in a fall upon standing Refer to #16
probably due to hypotension. This is most likely due
to blockade of which receptor?
A. Cholinergic
B. Nicotinic
C. Serotonergic
D. Alpha adrenergic

Abraham, M. | Aquino, J. | Arada, M. | Aw, A. | Bagon, N. | Contreras, S. | Galapon, C.


Gison, R. | Obciana, D. | Palao, K. | Tangcueco, P. | Tumambing, M.
5
PHARMACOLOGY RATIONALE
A.Y. 2017-2018

25. To which of the following sets of patients are the Answer: D


antimuscarinics beneficial?
A. 75 year old male with prostatic hyperplasia Poisoning with insecticide/organophosphate
B. 60 year old teacher with glaucoma (cholinomimetic): Antimuscarinics (Atropine) - antidote
C. 30 year old with gastric ulcer Prostatic Hyperplasia: In men, ⍺1 receptors mediate
D. 35 year old poisoning farmer with insecticide
prostatic smooth muscle contraction. Giving ⍺1-selective
blocking agents such as Prazosin (-zosin drugs) will
relax the smooth muscle, which is useful in BPH.
Glaucoma: Beta-blockers (Timolol), Cholinomimetics/
Cholinergic drugs (Pilocarpine, Physostigmine)
Gastric ulcer: M1-selective inhibitor (Pirenzepine), H2-
blockers, PPIs

26. A 40 year old man who presents with upper Answer: B


extremity weakness, ptosis and difficulty speaking Myasthenia gravis is a disorder of neuromuscular
was given edrophonium which caused improvement transmission characterized by weakness of cranial and
in his muscle strength for several minutes. This skeletal muscles. Autoantibodies directed against
indicates that the patient has: acetylcholine receptors damage the motor endplate
A. Hypertension portion of the neuromuscular junction, impairing the
B. Myasthenia gravis transmission of impulses to skeletal muscles. Clinical
C. Cholinergic poisoning manifestations may include diplopia, ptosis, and
D. Hypokalemic periodic paralysis weakness of facial, bulbar, respiratory, and proximal
limb muscles. (Adams et al., Principles of Neurology, 6th ed)
It is usually relieved by increasing
cholinomimetic treatment such as
Edrophonium, which is a short-acting
acetylcholinesterase inhibitor→increasing
Acetylcholine.

27. In which of the following conditions is Answer: D


acetylcholinesterase inhibitor NOT indicated? Acetylcholinesterase inhibitors:
A. Alzheimer’s disease ● Neostigmine, Pyridostigmine -Myasthenia gravis
B. Myasthenia gravis ● Physostigmine - Glaucoma
C. Glaucoma ● Donepezil, Rivastigmine, Galantamine -
D. Peptic Ulcer Disease Alzheimer‟s disease
M1-selective inhibitor (Pirenzepine) is used for the
treatment of PUD.

28. A 40 year old male complained of difficulty in Answer: A


voiding 1-2 days after appendectomy. Which of the Bethanechol - muscarinic
following would you give to promote voiding? agonist→parasympathomimetic effects such as
A. Bethanechol urinary emptying; used for bladder atony
B. Atropine Atropine - anticholinergic/anti-
C. Pilocarpine parasympathetic → urinary retention
D. Donepezil Pilocarpine - muscarinic agonist; used for Glaucoma
Donepezil - cholinesterase inhibitor;Alzheimer‟s Disease

Abraham, M. | Aquino, J. | Arada, M. | Aw, A. | Bagon, N. | Contreras, S. | Galapon, C.


Gison, R. | Obciana, D. | Palao, K. | Tangcueco, P. | Tumambing, M.
6
PHARMACOLOGY RATIONALE
A.Y. 2017-2018

(#29-30) A.M., a 50 year old patient was admitted to Answer: B


the ER because of an acute asthmatic attack. Terbutaline - Selective beta2 agonist causing relaxation
of bronchiolar smooth muscle, used as a bronchodilator
29. This sympathomimetic amine can be given to in asthma and tocolytic in premature labor
produce immediate bronchial smooth muscle Pseudoephedrine - Causes release of NE, used as
relaxation. decongestant in rhinitis and colds
A. Pseudoephedrine Phenylephrine - Selective alpha1 agonist, also used as
B. Terbutaline decongestant in rhinitis and colds
C. Phenylephrine Phentolamine - Competitively blocks alpha 1 receptors
D. Phentolamine (sympatholytic), used in the management of malignant
hypertension during operations for pheochromocytoma

30. History of the patient A.M. revealed that he is Answer: D


taking an antihypertensive which could have
triggered his asthmatic attack. The antihypertensive Nonselective beta blocker (β1&β2), such as Propranolol is
is most likely a: used in the treatment of hypertension, but excessive
A. Selective alpha 1 blocker blockade causes bronchospasm (β2 blockade) which can be
B. Selective alpha 2 blocker fatal in asthmatics.
C. Selective beta 1 blocker
D. Nonselective beta blocker

(#31-33) S.M., a 20 year old school teacher lives in a Answer: B


Manila condominium. One day the condominium
management announced a scheduled fumigation and Organophosphate toxicity (REMEMBER DUMBBELS!)
when she arrives home late in the afternoon she D - Diarrhea
experienced difficulty of breathing and coughing with U - Urination
increased bronchial gland secretions. M - Miosis
31. S.M. is most likely suffering from: B - Bradycardia
A. Acute muscarinic intoxication B - Bronchoconstriction
B. Acute organophosphate toxicity E - Emesis
C. Myasthenia gravis L - Lacrimation
D. Autonomic hyperreflexia S - Salivation/↑ Secretions

32. Expected cholinergic activity on S.M.’s stomach Answer: A


acid secretion:
A. Increased Parasympathetic stimulation increases GI secretion (M3).
B. Decreased
C. No change

33. Drug class that would be effective in treating Answer: A/B


S.M.’s condition: Atropine, an anticholinergic, specifically an
A. Anticholinergic drug antimuscarinic drug is used to treat organophosphate
B. Antimuscarinic drug toxicity. It acts as a competitive pharmacologic
C. Cholinesterase inhibitor drug antagonist at all M receptors (nonselective).
D. Depolarizing neuromuscular blocker

Abraham, M. | Aquino, J. | Arada, M. | Aw, A. | Bagon, N. | Contreras, S. | Galapon, C.


Gison, R. | Obciana, D. | Palao, K. | Tangcueco, P. | Tumambing, M.
7
PHARMACOLOGY RATIONALE
A.Y. 2017-2018

(#34-35) A.N., a 12 year old male patient was having Answer: B


diarrheic episode and his mother gave him a drug
that was supposed to stop the diarrhea. After the Atropine = Anticholinergic/Antimuscarinic AEs
second dose, his mother noted that his son was very ● Hot as a hare: increased body temperature
red and hot, and his son complained of blurring of ● Blind as a bat: mydriasis (dilated pupils)
vision and a “pounding chest”. ● Dry as a bone: dry mouth, dry eyes, decreased
34. The drug given by the mother probably contains: sweat
A. Betanechol ● Red as a beet: flushed face
B. Atropine ● Mad as a hatter: delirium
C. Pilocarpine
D. Neostigmine

35. The blurring of vision is due to what effect of the Answer: D


drug on the pupils? Parasympathetic effect (Eye)
A. Relaxation of the sphincter muscles of the iris
Action Receptor
B. Contraction of the radial muscles of the iris
C. Relaxation of the radial muscles of the iris Radial muscle of iris Relaxes M3
D. A and B only
Circular muscle of iris Contracts M3

Atropine = Blocks M3 receptors → Opposite


effect

TRUE OR FALSE Answer:


36. TRUE
True statements regarding inverse agonist and Inverse agonist has much higher affinity for
competitive antagonist: the inactive Ri state than active Ra,
36. In the presence of a full agonist, they both act to abolish or reduce constitutive activity
decrease agonist potency (activity in the absence of ligand), and may
37. In the absence of an agonist, a competitive produce contrasting physiologic effect →
agonist has no effect ↓potency.
38. In the absence of an agonist, an inverse agonist Competitive antagonist binds with equal affinity to the
inactivates constitutively active receptors Ri and Ra states, prevents binding by an agonist.
Because the antagonism is competitive, the presence of
antagonist increases the agonist concentration required
for a given degree of response. Sufficiently high
concentrations of the full agonist can surmount the
effect of a given concentration of the antagonist.
Greater concentration to produce response →
↓potency.

37. TRUE
In the presence of a fixed concentration of agonist,
increasing concentrations of a reversible competitive
antagonist progressively inhibit the agonist response.

38. TRUE
Inverse agonist reduces receptor activity below basal
levels in the absence of bound ligand.

Abraham, M. | Aquino, J. | Arada, M. | Aw, A. | Bagon, N. | Contreras, S. | Galapon, C.


Gison, R. | Obciana, D. | Palao, K. | Tangcueco, P. | Tumambing, M.
8
PHARMACOLOGY RATIONALE
A.Y. 2017-2018

MATCHING TYPE Answer:


Given the relative receptor affinities of the following
adrenergic drugs: 39. A
A. 1 = 2 ; 1 = 2 DOC for anaphylaxis - Epinephrine, it acts directly on
B. 2 >>> 1 both alpha and beta receptors
C. 1 > 2 >>>>>
D. 2 > 1 >>>>> 40. B
Match above to its most useful clinical application: The smooth muscle of the bronchi relaxes markedly in
39. A 10 year old boy who developed anaphylactic response to 2 agonists, e.g., Isoproterenol, Salbutamol
reaction to penicillin
40. A 5 year old girl with acute asthma exacerbation
41. A 15 year old student with nasal congestion 41. C
Alpha1 agonists are useful in which vasoconstriction is
appropriate. These include decongestant effects of
Phenylephrine. Phenylephrine acts directly on alpha
receptors and a selective alpha1 agonist.

MATCHING TYPE Answer:


Given the following drugs with their pKa: 42. A
A. Drug L = 2.8 43. C
B. Drug M = 6.1 44. D
C. Drug N = 9.2 pH (medium) < pKa (drug) → more protonated
D. Any of the above form
pH > pKa → more deprotonated form
Which of the above drugs:
42. Will be best absorbed in the stomach following Unionized form → readily absorbed
the pH-partition coefficient Ionized form → readily excreted
43. Will be excreted more rapidly in an acidic urine
44. Will be absorbed in the small intestine Weak acid in acidic medium → more unionized
form
● #42 Drug L - weak acid ; Stomach - acidic medium
Weak base in an acidic medium → more ionized
form
● #43 Drug N - weak base ; Acidic urine
Weak acid in a basic medium → more ionized
form
Weak base in a basic medium → more unionized
form

MATCHING TYPE Answer:


Given the types of dose-response relationships:
A. Quantal 45. B
B. Graded 46. A
C. Both 47. C
Which of the above:
45. Describes the effect of various doses of a drug on Both graded dose-response and quantal-dose response
an individual both demonstrates the response against increasing
46. Shows the effect of various doses of a drug on a concentrations of the drug.
population Graded-dose response: Emax and EC50

Abraham, M. | Aquino, J. | Arada, M. | Aw, A. | Bagon, N. | Contreras, S. | Galapon, C.


Gison, R. | Obciana, D. | Palao, K. | Tangcueco, P. | Tumambing, M.
9
PHARMACOLOGY RATIONALE
A.Y. 2017-2018

47. Demonstrates the effect of a drug as a function of Quantal-dose response: ED50, TD50 and LD50
its concentration

MATCHING TYPE Answer:


Match the H1 blockers to their pharmacologic 48. A 49. B 50. B 51. A
characteristics:
First gen 2nd gen
A. First generation (Diphenhydramine)
B. Second generation (Cetirizine) Mol wt lower higher

48. Neutral at physiologic pH Lipid solubility high low


49. Do not appreciable cross the blood brain barrier
50. Usually given once a day, at bedtime Affinity for high low
51. Additionally binds to cholinergic, -adrenergic cerebral H1R
and serotonergic receptors at therapeutic doses
sedation strong less

Blockage of yes little


autonomic R

Duration Shorter → Longer → once


more frequent a day dosing
dosing

VARIATION RELATIONSHIP Answer:


A. Increase in 1 will increase 2 52. B
B. Increase in 1 will decrease 2 Increasing the clearance will decrease the drug‟s t1/2.
C. Increase in 1 will not affect 2
53. A
52. (1) Cl with constant Vd (2) t1/2 Indices of drug bioavailability (F):
53. (1) AUC (2) F ● Cmax - maximum drug concentration in plasma
54. (1) Rate of infusion (2) t1/2 ● Tmax - time needed to reach maximum conc
55. (1) pKa (2) renal clearance in an acidic urine ● AUC (total area under the curve) for a time period
at least 3x the t1/2 of the active ingredient
Cmax and Tmax - determinants of rate of absorption
↑Cmax, ↓Tmax = Increased rate of absorption
AUC - reflects extent of absorption
↑AUC = ↑absorption = ↑bioavailability (F)

54. C
Increasing the rate of infusion will not increase or
decrease the t1/2. The two primary PK parameters that
are functions of half-life are (Vd) and (Cl).

55. A
Increase pKa = more basic
Weak base in an acidic medium → more ionized
form → Readily excreted/Increased clearance

Abraham, M. | Aquino, J. | Arada, M. | Aw, A. | Bagon, N. | Contreras, S. | Galapon, C.


Gison, R. | Obciana, D. | Palao, K. | Tangcueco, P. | Tumambing, M.
10
PHARMACOLOGY RATIONALE
A.Y. 2017-2018

QUANTITATIVE COMPARISON Answer:


A. Item 1 > Item 2 56. B
B. Item 1 < Item 2 Agonist+Competitive antagonism →
C. Item 1 = Item 2 ↑EC50=↓potency
Same maximal effect (Emax) is reached.
56. Potency of a drug to achieve Emax:
(1) Agonist + competitive antagonism 57. C
(2) Agonist alone Kd is the conc. of the drug required to occupy 50% of
57. Kd: the sites at equilibrium.
(1) Agonist alone Agonist+Noncompetitive antagonist →No change
(2) Agonist + noncompetitive antagonist on Kd
58. Ligand binding of a neutral antagonist: Causes a downward shift of the Emax.
(1) Ra
(2) Ri 58. C
59. Agonist with spare receptor at Emax: ● Full agonist has high affinity for the activated Ra
(1) EC50 receptor conformation.
(2) Kd ● Partial agonist favors the active state of the receptor
60. Ligand binding to Ri: (Ra) than the inactive (Ri) state.
(1) Inverse agonist ● Inactive compound/Antagonist binds equal affinity
(2) Partial agonist to the Ri and Ra states..
● Inverse agonist has higher affinity for the inactive Ri
state than Ra.

59. B
Spare receptors are said to exist if the maximal drug
response is obtained at less than 100% occupation of
the receptors. The determination is usually made by
comparing the EC50 with the Kd. If the EC50 is less than
the Kd, spare receptors are said to exist.

60. A (Refer to #58)

Abraham, M. | Aquino, J. | Arada, M. | Aw, A. | Bagon, N. | Contreras, S. | Galapon, C.


Gison, R. | Obciana, D. | Palao, K. | Tangcueco, P. | Tumambing, M.
11
PHARMACOLOGY RATIONALE
A.Y. 2017-2018
1ST SHIFTING EXAM

QUESTION ANSWER & RATIONALE

1.Which of the following is the clinical importance of Answer: A


plasma protein binding?
A. Helps in the interpretation of measured drug
concentration ↑protein binding =↓free drug conc = ↓Vd
B. Guides in designing a rational dosage regimen
C. Helps in the calculation of appropriate maintenance Extensive binding retards the rate at which the drug
dose reaches its site of action and may prolong the
D. Explains the marked variations in plasma duration of action
concentrations of drugs with high extraction ratio
Albumin * binds weakly acidic drugs, eg. Warfarin
Alpha-1 glycoprotein * binds weakly basic drugs,
eg. Lidocaine
Transcortin * binds steroids, eg. Cortisol

B, C * clearance

2. A male patient has cirrhosis with ascites and pleural Answer: D


effusion. Which of the following parameters will be
markedly increased? ↑extraction ratio = ↓bioavailability
A. Bioavailability ↑free drug = ↑ Vd
B. Clearance
C. Half-life
D. Volume of distribution

3. TRUE of spare receptors: Answer: C


A. Emax = Bmax
B. EC50 = KD Spare receptors
C. Seen in GPCR activation and signaling pathway -receptors are said to be “spare” if it is possible to
D. Decreases efficiency/sensitivity of the drug- elicit a maximal response even at less than maximum
receptor system concentration
-it increases the sensitivity of the system

-Emax > Bmax


Emax - maximal response that the drug can produce
Bmax - total number of receptor sites are occupied

***Emax = EC50 - related to efficacy


***Bmax = Kd - related to binding

-Kd > EC50


Kd - concentration of drug with which 50% of the
drug is bound
EC50 - concentration needed to produce 50% of
maximal response

Abraham, M. | Aquino, J. | Arada, M. | Aw, A. | Bagon, N. | Contreras, S. | Galapon, C.


Gison, R. | Obciana, D. | Palao, K. | Tangcueco, P. | Tumambing, M.
12
PHARMACOLOGY RATIONALE
A.Y. 2017-2018

4. Rapid diminution of response with B2 receptor Answer: A


activation is an example of:
A. Tachyphylaxis Tolerance - decreased responsiveness to a given
B. Tolerance dose as a consequence of continued drug
C. Idiosyncrasy administration (eg. Nitrates)
D. Hypersensitivity Tachyphylaxis - rapid diminution of responsiveness
after administration of a drug (eg. Beta 2 activation)

5. Which of the following does NOT measure/reflect the Answer: D


safety of the drug?
A. Slope of the dose-response curve Measures safety:
B. Therapeutic window A. Slope of drug-response curve * more slope, less
C. Margin of safety safe
D. LD50 B. Therapeutic window *ratio between minimum
effective concentrations (MEC) to the minimum toxic
concentration (MTC); more clinically relevant
measure of drug safety
C. Margin of safety * ratio between therapeutic
doses (ED50) and lethal doses (LD50); estimated by
Therapeutic Index (TI50)

6. A 40 year old teacher consulted a doctor for his arthritis. Answer: D


The doctor took into consideration the therapeutic index of
the following drugs for arthritis. Which is the safest to Therapeutic Index (TI50) - quantification of drug
prescribe? safety
A. Drug D = 1.5
B. Drug F = 4 TI50 = LD50/ED50 or TD50/ED50
C. Drug E = 6 ↑LD50 = ↓ED50 = ↑TI50 = safer
D. Drug B = 8

7. A patient with renal failure is given a drug every 48 Answer: C


hours. The peak concentration reached after a given dose
is known as: Cmax
A. Steady State Concentration -the maximum drug concentration in plasma
B. Target Concentration -one of 3 indices of bioavailability (F)
C. Cmax *Cmax, Tmax - rate of absorption
D. Therapeutic Concentration *AUC - extent of absorption

Steady state concentration (Css)


-when the amount of drug administered is equal to
the amount of drug eliminated, resulting in plateau or
constant serum drug level
-Css is reached after 4 to 5 t1/2
-so Css = t1/2 x 4 (or 5)

Abraham, M. | Aquino, J. | Arada, M. | Aw, A. | Bagon, N. | Contreras, S. | Galapon, C.


Gison, R. | Obciana, D. | Palao, K. | Tangcueco, P. | Tumambing, M.
13
PHARMACOLOGY RATIONALE
A.Y. 2017-2018

8. Drug B combines with the same site on the receptor as Answer: B


does Drug A (agonist). But unlike Drug A, Drug B does not
induce a response. Which of the following is correct about Full agonist - high affinity, high intrinsic activity
Drug B? Partial agonist - high affinity, low intrinsic activity
A. Inverse agonist Antagonists - high affinity, no intrinsic activity
B. Competitive agonist
C. Non-competitive agonist Inverse agonist - same site, opposite effect
D. Partial agonist Competitive antagonist - same site, no effect

Competitive antagonist - binds reversibly


Non-competitive antagonist - binds irreversibly

9. Drug A has lower EC50 and Emax than Drug B. Answer: B


Therefore, Drug A (agonist):
A. Will require higher dose to produce a given EC50 - concentration needed to produce 50% of
response maximal response (measures potency)
B. Will require lower dose to produce a given ↓EC50 = ↑potency = lower dose to produce
response response
C. Has more therapeutic benefit
D. Is less toxic Emax - maximal response that the drug can produce
(measures efficacy)
↑Emax = ↑efficacy

10. Which of the following exemplifies physiologic Answer: B


antagonism?
A. Meperidine (Phetedine) and Naloxone Types of Antagonism
B. Histamine and Epinephrine Physiologic - acts in opposition (eg. Histamine and
C. Protamine and Heparin Epinephrine)
D. Histamine and Cetirizine Pharmacologic - interaction at drug receptor (eg.
Histamine and Cetirizine, Naloxone and Opiates)
Chemical - acidic to basic and vice versa (eg.
Protamine and Heparin)

Abraham, M. | Aquino, J. | Arada, M. | Aw, A. | Bagon, N. | Contreras, S. | Galapon, C.


Gison, R. | Obciana, D. | Palao, K. | Tangcueco, P. | Tumambing, M.
14
PHARMACOLOGY RATIONALE
A.Y. 2017-2018

11. A 19 year old student developed extrapyramidal Answer: C


symptoms after intake of an anti-emetic agent. The most
appropriate H1 blocker to suppress these symptom is: H1 blockers or Antihistamines
A. Desloratadine
B. Promethazine Effects and examples:
C. Diphenhydramine •Sedation * old generation H1 blockers eg.
D. Hydroxyzine Hydroxyzine
•Antiemetic * Promethazine, Meclizine (Personal
Message mo si…)
•Antimotion sickness * Meclizine,
Diphenhydramine, Clemastine (M.D. Doc Clem…)
•Antimuscarinic * Promethazine,
Diphenhydramine, Clemastine (kung PeDe si Doc
Clem)
•Antiparkinsonism * Diphenhydramine
•Adrenoceptor blockade (hypotension) *
Phenothiazine
•Serotonin blockade (so increase appetite) *
Cyproheptadine
•Na channel blockade in excitable membranes *
Diphenhydramine, Promethazine

Antiparkinson drugs can be used in the management


of drug-induced EPS such as dystonia, akathisia and
parkinsonian symptoms

12. Which serotonin receptor subtype is partially Answer: B


antagonized by ondansetron?
A. 5-HT2C Refer to LQ1 #18
B. 5-HT3
C. 5-HT4
D. 5-HT5A

13. A newborn baby was diagnosed with right ventricular Answer: B


outflow obstruction. This prostanoid derivative can
maintain ductus arteriosus and increase survival of this Alprostadil (PGE1) - maintains PDA
baby before definitive surgery: Indomethacin (NSAID) - treats PDA
A. Epoprostenol
B. Alprostadil
C. Indomethacin
D. Meclofenamate

Abraham, M. | Aquino, J. | Arada, M. | Aw, A. | Bagon, N. | Contreras, S. | Galapon, C.


Gison, R. | Obciana, D. | Palao, K. | Tangcueco, P. | Tumambing, M.
15
PHARMACOLOGY RATIONALE
A.Y. 2017-2018

14. A 17 year old athlete with history of peptic ulcer Answer: C


disease has a sprained ankle. The most appropriate COX
inhibitor for this patient is: Since the patient has PUD, use selective COX2
A. Indomethacin inhibitors which has NO gastrointestinal SE but may
B. Paracetamol have cardiovascular SE
C. Etoricoxib eg. Etoricoxib - has highest selectivity ratio for
D. Aspirin inhibition of COX-2 relative to COX-1 (106:1)

Indomethacin - non-selective
Paracetamol - analgesic without anti-inflammatory
effect
Aspirin - oldest NSAID, AE: GI upset, bleeding

15. This PGF2a derivative reduces intraocular pressure in Answer: C


open-angle glaucoma:
A. Epoprostenol Open-angle glaucoma * Pilocarpine, Ecothiopate
B. Pilocarpine Close-angle glaucoma * Latanoprost
C. Latanoprost
D. Zafirlukast

16. The mechanism of action of indirect-acting Answer: B


cholinomimetic agent isL
A. Releasing acetylcholine from storage sites Direct-acting cholinomimetics - binding to and
B. Inhibition of the hydrolysis of endogenous activation of acetylcholine/cholinergic receptors
acetylcholine (muscarinic and nicotinic)
C. Stimulation of action of acetylcholinesterase •Naturally occuring - Muscarine, Arecoline,
D. Binding to and activation of muscarinic or nicotinic Pilocarpine)
receptors •Synthetic - Bethanechol, Carbachol

Indirect-acting cholinomimetics - inhibits


acetylcholinesterase
•Short acting - Edrophonium
•Medium acting - Neostigmine, Physostigmine
•Long acting - Organophosphates

17. Reye’s syndrome may be seen in children with Answer: D


chickenpox following administration of:
A. Ibuprofen Other AE of Aspirin:
B. Paracetamol At therapeutic dose - GI upset, GI bleeding
C. Celecoxib At large dose - salicylism (tinnitus, deafness,
D. Aspirin dizziness)
At toxic dose - uncompensated metabolic acidosis

Abraham, M. | Aquino, J. | Arada, M. | Aw, A. | Bagon, N. | Contreras, S. | Galapon, C.


Gison, R. | Obciana, D. | Palao, K. | Tangcueco, P. | Tumambing, M.
16
PHARMACOLOGY RATIONALE
A.Y. 2017-2018

18. Which of the following toxicity effects of NSAIDs is Answer: B


attributed to its inhibition of prostaglandin synthesis?
A. Liver With prostaglandin = Increased constriction of
B. Renal efferent arteriole
C. CNS
D. Hematologic

19. Low dose of this drug is associated with lower Answer: B


incidence of colon cancer:
A. Paracetamol Paracetamol - short term analgesic, antipyretic
B. Aspirin Aspirin - anti-inflammatory, anti-platelet
C. Morphine aggregation
D. Celecoxib Morphine - analgesic for chronic pain
Celecoxib - analgesic, antipyretic, anti-inflammatory
but NO anti-platelet aggregation

20. Indomethacin’s half-life is prolonged by the Answer: D


concomitant administration of probenecid through this
mechanism: Probenecid + Drug = inhibits renal and biliary
A. Inhibition of its absorption clearance of Drug = prolong t1/2 of Drug
B. Enhancement of its metabolism Use of Probenecid: Uricosurics, Gout
C. Increase in its Vd
D. Inhibition of its renal and biliary clearance

21. A 4 year old boy has intermittent low to moderate Answer: B


grade fever and colds for the past 3 days. The most
appropriate antipyretic with the best safety profile is: Aspirin (oldest NSAID) - do not give because fever in
A. Aspirin this case may be due to viral infection and will cause
B. Paracetamol Reye‟s Syndrome
C. Ibuprofen Paracetamol (antipyretic) - safest
D. Celecoxib Ibuprofen (non-selective COX inhibitor) - AE:
agranulocytosis, aplastic anemia
Celecoxib (selective COX inhibitor) - AE: CV side
effects, edema, hypertension

22. A 28 year old was diagnosed with an overdose of ***Answer: B


unrecalled medication after which she developed signs and
symptoms of severe and prolonged vasospasm. The Oxytocin - used to induce labor
unrecalled medication is most likely: Ergotamine (ergot alkaloid) - tx for migraine type HA
A. Oxytocin Ketanserin (5HT2 antagonist) - antihypertensive
B. Ergotamine Tegaserod (5HT4 agonist)-mgt of IBS and
C. Ketanserin constipation
D. Tegaserod

Abraham, M. | Aquino, J. | Arada, M. | Aw, A. | Bagon, N. | Contreras, S. | Galapon, C.


Gison, R. | Obciana, D. | Palao, K. | Tangcueco, P. | Tumambing, M.
17
PHARMACOLOGY RATIONALE
A.Y. 2017-2018

23. Concomitant administration of Ibuprofen and Aspirin Answer: B


may result in:
A. Potentiation of platelet aggregation Ibuprofen also antagonizes the irreversible platelet
B. Decreased total anti-inflammatory effect inhibition induced by Aspirin. Therefore, treatment of
C. Increase incidence of cardiovascular thrombotic patients at increased CV risk with Ibuprofen may
events limit the cardioprotective effects of Aspirin.
D. Decreased risk of gastrointestinal adverse effects

24. Which among the anticonvulsants below causes Answer: A


hyponatremia?
A. Carbamazepine Adverse Effects (emphasized in the lecture):
B. Phenytoin Carbamazepine - mild leukopenia, hyponatremia,
C. Lamotrigine osteomalacia
D. Lithium Phenytoin - gingival hyperplasia, folate deficiency
Lamotrigine - ataxia, diplopia
Lithium -

25. This anticonvulsant should NOT be given Answer: D


intramuscularly because of unpredictable absorption and
precipitation of the drug in the muscle: Phenytoin
A. Phenobarbital -IM injection is NOT recommended because of
B. Carbamazepine unpredictable absorption and drug precipitation in
C. Valproic acid the muscle
D. Phenytoin -highly protein bound (also Valproic)
-rate of elimination: NON-linear

26. Which of the following anticonvulsants act by Answer: C


inhibiting neurotransmission through NMDA receptor
antagonism? Topiramate - AMPA and kainate receptor
A. Topiramate antagonism
B. Levetiracetam Felbamate - NMDA receptor antagonism
C. Felbamate
D. Zonisamide Levetiracetam - binding to synaptic vesicle protein
(SV2A)

Zonisamide - reduction of Na conductance

27. This anticonvulsant, which acts by reducing sodium Answer: C


conductance, does not have any significant
pharmacokinetic interactions: Phenytoin, Lacosamide - reduction of Na
A. Phenytoin conductance
B. Phenobarbital *Phenytoin is an enzyme-inducer so prone
C. Lacosamide pharmacokinetic interactions
D. Valproic acid
Phenobarbital, Valproic - enhancement of GABA
current
*Phenobarbital - enzyme-inducer
*Valproic -enzyme-inhibitor

Abraham, M. | Aquino, J. | Arada, M. | Aw, A. | Bagon, N. | Contreras, S. | Galapon, C.


Gison, R. | Obciana, D. | Palao, K. | Tangcueco, P. | Tumambing, M.
18
PHARMACOLOGY RATIONALE
A.Y. 2017-2018

28. To which of the following patients is Levodopa given Answer: D


with caution or contraindicated?
A. 40 year old female with malignant melanoma Levodopa
B. 50 year old male with angle-closure glaucoma -immediate precursor of dopamine that can enter the
C. 28 year old male with active peptic ulcer disease brain where it is decarboxylated to dopamine
D. All of the above -however, only 1-3% enters brain unaltered and
remainder is metabolized extracellularly
-thus, it is given with a dopa decarboxylase inhibitor
(eg, Carbidopa, Benserazide) so that more levodopa
is available for entry into the brain and to decrease
SE associated with dopamine

Other CI: psychosis, MAOI, pregnancy and lactation

29. Pharmacokinetic advantage of Pramipexole (dopamine Answer: A


receptor agonist) over Levodopa in the treatment of
Parkinson’s disease: Dopamine receptor agonists - first line tx for PD
A. Does not require enzymatic conversion to an active Bromocriptine - D2, older
metabolite Pramipexole - D3, newer
B. Has less potential for producing toxic metabolite
C. More effective in reducing tremors and sedation Advantages:
D. Has very long half-life of 48 hours •do not require enzymatic conversion to an active
metabolite
•no potentially toxic metabolite
•do not compete with other substances for active
transport into the blood and across the BBB
•lower incidence of response fluctuations and
dyskinesias

30. A 28 year old executive, diagnosed to have bipolar Answer: D


disorder and epilepsy, needs a prophylactic agent for
migraine. The most appropriate drug is: Mood stabilizer - Carbamazepine, Valproic
A. Gabapentin Anti-epileptic -Carbamazepine, Valproic
B. Ethosuximide Prophy for Migraine -Valproic, Gabapentin,
C. Carbamazepine Phenytoin
D. Valproic acid

Abraham, M. | Aquino, J. | Arada, M. | Aw, A. | Bagon, N. | Contreras, S. | Galapon, C.


Gison, R. | Obciana, D. | Palao, K. | Tangcueco, P. | Tumambing, M.
19
PHARMACOLOGY RATIONALE
A.Y. 2017-2018

31. A 75-year-old male patient, known hypertensive and ***Answer: B


diabetic, admitted at the general ward for medical work-up
suddenly presented with acute onset of visual and auditory Antipsychotics
hallucinations. He became verbally and physically Typical/Classical (blocks D2 only) - Chlorpromazine,
assaultive. Which of the following psychopharmacologic Haloperidol
agents will be most suitable for this situation? Atypical (blocks both D2 and 5HT2A) - Quetiapine,
A. Chlorpromazine Olanzapine
B. Haloperidol
C. Quetiapine Atypical antipsychotics AE include
D. Olanzapine hyperglycemia/hyperlipidemia and inc risk of CV
events SO not recommended in this case because
our px is diabetic and hypertensive

Highest EPS - Haloperidol (IDK why this is the


answer)
Least EPS - Clozapine

32. If one will be asked to explain the rationale for the Answer: D
medical therapy for a 67-year-old woman diagnosed with
Alzheimer’s dementia, administration of donepezil, Don Riva Gala has Alzheimer’s
rivastigmine, and galantamine is used to increase which of Donepezil, Rivastigmine, Galantamine
the following neurotransmitters?
A. Dopamine -acetylcholinesterase inhibitor so ↑Ach
B. Norepinephrine
C. Glutamate
D. Acetylcholine

33. Which of the following general classification of Answer: D


antipsychotics medications will be clinically most
appropriate to control psychotic symptoms for patients High potency D2 antagonist - EPS
with Alzheimer’s dementia? Low potency D2 antagonist - low HAM
A. D2 antagonist — low potency
B. D2 antagonist — high potency Partial D2 agonist - won‟t worsen EPS symptoms
C. Combined D2/5-HT2A antagonist
D. Partial D2 agonist

34. Which of the following statements is true regarding Answer: C


antipsychotic drugs potencies?
A. Chlorpromazine > Perphenazine > Haloperidol They differ in potency, but NOT in effectivess
B. Perphenazine > Haloperidol > Chlorpromazine
C. Haloperidol > Perphenazine > Chlorpromazine
D. Haloperidol > Chlorpromazine > Perphenazine

Abraham, M. | Aquino, J. | Arada, M. | Aw, A. | Bagon, N. | Contreras, S. | Galapon, C.


Gison, R. | Obciana, D. | Palao, K. | Tangcueco, P. | Tumambing, M.
20
PHARMACOLOGY RATIONALE
A.Y. 2017-2018

35. Which of the following benzodiazepines will be most Answer: D


appropriate for the treatment of acute panic anxiety
disorders? Diazepam - longest effect
A. Diazepam Midazolam - shortest effect
B. Bromazepam
C. Midazolam Clonazepam - DOC for acute panic disorder
D. Clonazepam

36. Aripiprazole is an ideal atypical antipsychotic which Answer: D


has demonstrated efficacy for both positive and negative
symptom of schizophrenia due to which of the following D2 antagonism - decrease positive symptoms only
mechanism of action? Serotonergic antagonism - increase appetite
A. Dopamine 2 antagonism Fast/readily dissociating D2 antagonism -
B. Serotonergic antagonism Quetiapine
C. Fast dissociating dopamine 2 antagonism
D. Partial dopamine antagonism

37. The high ratio of 5-HT2a receptor to D2 receptor Answer: B


blockade characteristic of atypical antipsychotics will result
into which of the following? More 5HT2A-R to block = dopamine release =
A. Decrease in dopaminergic transmission in the mesocortical pathway = reduce negative and
nigrostriatal pathway cognitive symptoms
B. Improvement of negative symptoms More D2-R to block = dopamine blockade =
C. Cognitive impairments accentuated mesolimbic pathway = antipsychotic effect
D. Increase in cholinergic activity Equal = no change in dopamine = tubuloinfundibular
pathway = prevent increased prolactin

38. Which of the following pharmacologic properties is Answer: B


associated with Quetiapine?
A. Mechanism of action is tight-binding combined A - Clozapine
antagonist C - Typical D2 antagonists
B. Retains binding affinity to D2 receptors but D - Clozapine
dissociates readily
C. Does not have any affinity to all with serotonergic
receptors
D. Tight binding to D2 and 5-HT3 receptors

39. Which of the following D2/5HT antagonists has the Answer: D


best pharmacodynamics adverse effect profile compared to
the pure D2 antagonists? Aripiprazole
A. Clozapine -best partial D2 agonist because it can decrease both
B. Olanzapine positive and negative symptoms
C. Risperidone
D. Aripiprazole

Abraham, M. | Aquino, J. | Arada, M. | Aw, A. | Bagon, N. | Contreras, S. | Galapon, C.


Gison, R. | Obciana, D. | Palao, K. | Tangcueco, P. | Tumambing, M.
21
PHARMACOLOGY RATIONALE
A.Y. 2017-2018

40. Typical high-potency antipsychotics will strongly be Answer: A


contraindicated in which of the patients with the following
concomitant diseases? D2 blockade
A. Parkinson’s disease -produces antipsychotic effect in mesolimbic
B. Geriatric patients with cardiovascular disease pathway
C. Benign prostatic hypertrophy -worsens negative and cognitive symptoms in
D. Glaucoma mesocortical pathway
-causes EPS in nigrostriatal pathway so CI in PD
-causes increase in prolactin (gynecomastia) in
tuberoinfundibular

41. This atypical antipsychotic binds tightly to D2 receptor: Answer: A


A. Risperidone
B. Chlorpromazine Other tight D2 binding:
C. Quetiapine -done: Risperidone, Ziprasidone
D. Haloperidol -apine: Loxapine, Clozapine, Olanzapine
-Amisulpride, Sertindole, etc

B - typical
C - atypical readily dissociating
D - atypical partial D2 agonist

42. A 22 year old student on unrecalled anti-psychotic Answer: C


developed extrapyramidal symptoms. The unrecalled
medication is most likely: ↑potency = ↑EPS
A. Chlorpromazine
B. Clozapine Chlorpromazine - low potency
C. Haloperidol Haloperidol - high potency
D. Quetiapine
Clozapine, Quetiapine - atypical, no EPS

43. Which is NOT a pharmacokinetic characteristic of Answer: A


lithium carbonate?
A. Highly protein bound PK of Lithium carbonate:
B. Renal elimination -A: complete absorption
C. Complete absorption -D: NOT bound to plasma proteins
D. Not metabolized -M: NO metabolism
-E: 100% renal

44. Which of the following will increase lithium toxicity? Answer: B


A. Obesity
B. Hyponatremia High Lithium = Low Sodium
C. Hypokalemia
D. Metabolic acidosis

Abraham, M. | Aquino, J. | Arada, M. | Aw, A. | Bagon, N. | Contreras, S. | Galapon, C.


Gison, R. | Obciana, D. | Palao, K. | Tangcueco, P. | Tumambing, M.
22
PHARMACOLOGY RATIONALE
A.Y. 2017-2018

45. A 10 year old girl developed nightmares and Answer: D


depression few days after the attack of the Maute group.
She would benefit most with this drug: First-line for depression: FEDS
A. Diazepam Fluoxetine
B. Risperidone Escitalopram
C. Quetiapine Duloxetine
D. Sertraline Sertraline

46. Which of the following benzodiazepines will be most Answer: A


appropriate for the initiation treatment of status
epilepticus? Diazepam
A. Diazepam -effective agent for status epilepticus (given IV) but
B. Bromazepam has shorter DOA due to rapid distribution of drug
C. Triazolam from brain
D. Clonazepam -DA: NOT useful as an oral agent for tx of seizures
and NOT used as long term antiepileptic drug,
accumulates on repeated administration

47. Mematine, as a distinct class of dementia drugs, would Answer: C


have which of the following action?
A. Acetylcholinesterase inhibitor A - Don Riva Gala
B. Catechol-o-methyl transferase inhibitor B - Tolcapone, Entacapone
C. N-methyl-d-aspartate receptor blocker D-
D. Volatage gated calcium channel blocker

48. This drug is used in the treatment of Alzheimer’s Answer: A


disease has the beneficial effect of increasing cholinergic
activity: Don Riva has Alzheimer’s, kaya Gumala sa Tacrine
A. Rivastigmine Donepezil -BEST/DOC. inhibits acetylcholinesterase
B. Succinylcholine Rivastigmine - inhibits acetylcholinesterase +
C. Memantine butyrylcholinesterase; non hepatic
D. Gemfibrozil Galantamine - inhibits acetylcholinesterase +
allosteric N receptor
Tacrine - hepatotoxic

49. Which of the following when given in high doses can Answer: C
cause dry mouth, mydriasis, tachycardia, hot and flushed
skin and agitation? All mentioned are sympathetic effects so look for a
A. Neostigmine sympathetic / anticholinergic / adrenergic drug.
B. Nicotine
C. Atropine Neostigmine - Cholinergic (Acetylcholinesterase
D. Pralidoxime inhibitor)
Nicotine - Cholinergic
Atropine - Anticholinergic / Antimuscarinic
Pralidoxime - Cholinergic / Acetylcholinesterase
regenerator (Antidote for Atropine)

Abraham, M. | Aquino, J. | Arada, M. | Aw, A. | Bagon, N. | Contreras, S. | Galapon, C.


Gison, R. | Obciana, D. | Palao, K. | Tangcueco, P. | Tumambing, M.
23
PHARMACOLOGY RATIONALE
A.Y. 2017-2018

50. A 52 year old teacher was diagnosed to have Answer: C


myasthenia gravis. The most appropriate drug for long
term treatment of the disease is: Edrophonium - to diagnose myasthenia gravis
A. Edrophonium Pyridostigmine - to treat myasthenia gravis
B. Echothiophate Echothiophate, Pilocarpine - for closed/narrow
C. Pyridostigmine angle glaucoma
D. Pilocarpine

A 23 year old medical student has acute severe migraine Answer: B


attack:
“-tan” = 5HT1 (wan) = for migraine
51. The most appropriate drug is: [Sumatriptan]
A. Ergonovine
B. Sumatriptan “-tron” = 5HT3 (tree) = for chemo induced N/V
C. Propranolol [Ondansetron]
D. Amitryptiline

52. The answer to #51 should NOT be used in patients at Answer: D


risk for:
A. Asthma 5HT1D - cases vasoconstriction
B. Cancer
C. Glaucoma
D. Coronary artery disease

P.H., a 70 year old patient with BPH, has runny nose and Answer: B
was prescribed a decongestant with sympathomimetic
activity: Phenylephrine - Alpha 1 agonist = vasoconstricts
vessels to prevent runny nose
53. Which of the ff sympathomimetic drugs has
decongestant properties?
A. Amphetamines
B. Phenylephrine
C. Epinephrine
D. Metamphetamine

54. After three doses of the drug (#53), the patient Answer: B
experienced difficulty of urination which could be due to:
A. Contraction of the trigone of the urinary bladder Urinary retention = Sympathetic
B. Contraction of the urinary bladder sphincter - Relax bladder wall (B2)
C. Relaxation of the urinary bladder sphincter - Contract sphincter (a1)

Urinary emptying = Parasympathetic


- Contract bladder wall (M3)
- Relax sphincter (M3)

Abraham, M. | Aquino, J. | Arada, M. | Aw, A. | Bagon, N. | Contreras, S. | Galapon, C.


Gison, R. | Obciana, D. | Palao, K. | Tangcueco, P. | Tumambing, M.
24
PHARMACOLOGY RATIONALE
A.Y. 2017-2018

A 40 year old farmer, after spraying insecticide, presents Answer: D


with pinpoint pupils, salivation, wheezes and vomiting.
Organophosphate = irreversible acetylcholinesterase
55. He most likely has acute intoxication with: inhibitor = cholinergic drug = parasympathetic
A. Edrophonium [Malathion, Parathion]
B. Nicotine
C. Alcohol
D. Organophosphate

56. The mandatory antidote for the above (#55) Answer: B


intoxication is:
A. Epinephrine Atropine - anti-muscarinic, physiological antidote to
B. Atropine organophosphate toxicity, mandatory to be given
C. Pralidoxime immediately
D. Beta blocker
Pralidoxime - acetylcholinesterase regenerator,
pharmacologic antidote to organophosphate toxicity,
given within 24 hours

57. If this patient (#55) was brought to the ER within 24 Answer: B


hours of exposure, which of the ff should be given to
regenerate the active enzyme from the organophosphorus- Refer to #56
cholinesterase complex?
A. Atropine
B. Pralidoxime
C. Neostigmine
D. Scopolamine

A 30 year old male was diagnosed with Rheumatoid Answer: C


arthritis:
First line DMARD for RA [HSM] -
58. Which of the ff is the first line drug for his condition? Hydroxychloroquine, Sulfasalazine, Methotrexate
A. Rituximab
B. Azathioprine Methotrexate
C. Methotrexate - As DMARD: inhibits thymidylate synthase
D. Chloroquine and AICAR
- As Antineoplastic: inhibits dihydrofolate
reductase

59. Which of the ff should be given to reduce the incidence Answer: A


of GI and liver function abnormalities of the answer in #58?
A. Leucovorin Leucovorin (Folinic acid) - active form of folic acid,
B. Prednisone acts on normal cells
C. Cetirizine - “Leucovorin rescue” for Methotrexate
D. Ranitidine - Enhances effect of 5-FU by inhibiting
thymidylate synthase

Abraham, M. | Aquino, J. | Arada, M. | Aw, A. | Bagon, N. | Contreras, S. | Galapon, C.


Gison, R. | Obciana, D. | Palao, K. | Tangcueco, P. | Tumambing, M.
25
PHARMACOLOGY RATIONALE
A.Y. 2017-2018

A 40 year old presents with markedly swollen and painful Answer: B


left knee after binged alcohol drinking. His uric acid was
elevated and left knee aspirate revealed monosodium urate Aspirin, Tolmetin - NSAIDs contraindicated in gout
crystals: Colchicine
- DOC acute attacks of gout
60. The most appropriate drug that will relieve pain and - MOA: inhibits microtubule function &
inflammation within 24 hours is: synthesis
A. Aspirin - ADR: bloody diarrhea
B. Colchicine Allopurinol
C. Parecoxib - DOC chronic gout/maintenance
D. Allopurinol - MOA: xanthine oxidase inhibitor
- ADR: SJS, bone marrow suppression
Probenecid
- MOA: uricosuric
- Inhibit renal tubular reabsorption of weak
acids

61. The most appropriate drug to give between acute Answer: D


attacks of gout is:
A. Indomethacin Refer to #60
B. Prednisone
C. Ibuprofen
D. Allopurinol

62. The main adverse effect of the answer to #60 is: Answer: C
A. Sedation
B. Acidosis ADRs of Colchicine - GI disturbance/bloody diarrhea
C. Diarrhea (most common), aplastic anemia, thrombocytopenia,
D. Agranulocytosis agranulocytosis, paresthesia, vasomotor collapse

63. This agent is effective when given IV for seizure control Answer: B
but NOT when given orally:
A. Tiagabine Diazepam - anxiolytic/sedative-hypnotic
B. Diazepam - Potentiates GABA
C. Lamotrigine - Tendency to develop tolerance (also with
D. Topiramate Nitrates)

64. The answer to #63 is because of its propensity to Answer: C


develop:
A. Liver failure Tolerance - reduced response to a drug following
B. Cardiac arrhythmias repeated use [long term]
C. Tolerance
D. Allergic reaction Benzodiazepines must be prescribed for short
periods (2-4 weeks) with continued reevaluation

Abraham, M. | Aquino, J. | Arada, M. | Aw, A. | Bagon, N. | Contreras, S. | Galapon, C.


Gison, R. | Obciana, D. | Palao, K. | Tangcueco, P. | Tumambing, M.
26
PHARMACOLOGY RATIONALE
A.Y. 2017-2018

65. Which of the ff NSAIDs is recommended to be given to Answer: C


a post-MI 50 year old male?
A. Paracetamol Aspirin - TXA2 inhibitor anti-thrombotic, irreversible,
B. Indomethacin non-selective COX inhibitor
C. Aspirin - Taken soon after an acute MI, reduces
D. Ibuprofen mortality, reinfarction, and stroke

66. The beneficial effect of #65 is primarily due to its: Answer: A


A. Anti-platelet aggregation
B. Anti-inflammatory All mentioned are effects of aspirin, but its use post-
C. Analgesic MI is due to its anti-thrombotic effect.
D. Anti-pyretic

67. The BP of a hypertensive patient on maintenance Answer: C


medication with Drug X (which is metabolized by CYP450)
is poorly controlled. For better control, Drug C, a known Inducers = increase metabolism of substrate =
enzyme inhibitor is added. Which of the ff would be shorter half-life = decreased efficacy
expected?
A. Decreased efficacy of Drug X and X Inhibitors = decrease metabolism of substrate =
B. Lower plasma concentration of Drug C longer half-life = increased efficacy
C. Longer half-life of Drug X
D. Development of tolerance to drug C

MATCHING TYPE: Answer:


Indicate whether the anticonvulsants given below can 68. C
cause: 69. A
A. Enzyme induction 70. B
B. Enzyme inhibition
C. Both PURE INDUCERS: Carbamazepine Lamotrigine
D. Neither Phenytoin Phenobarbital [Kala mo pepe inducer pala]

68. Topiramate PURE INHIBITOR: Valproic acid [Ayaw ni Val]


69. Phenobarbital
70. Valproic acid BOTH: Topiramate, oxcarbazepine [Two-timer si
Topi & Oscar]

NEITHER: The rest

Given the ff drugs with their pharmacologic properties: Answer:


A. Drug R : t1/2 = 100 hours 71. A
B. Drug S : plasma protein binding = 99.1% Longer half-life = longer time (Tmax) to reach Cmax
C. Drug T : oral bioavailability = 30% so need loading dose
72. B
Which of the above drugs: Protein bound = vascular compartment
71. Would need a loading dose to promptly raise the 73. C
plasma drug concentration to the target concentration? Bioavailability = rate & extent of drug absorption into
72. Has lower concentrations in extravascular tissue than systemic circulation
in the vascular compartment?
73. Has incomplete extent of absorption across the gut
wall?

Abraham, M. | Aquino, J. | Arada, M. | Aw, A. | Bagon, N. | Contreras, S. | Galapon, C.


Gison, R. | Obciana, D. | Palao, K. | Tangcueco, P. | Tumambing, M.
27
PHARMACOLOGY RATIONALE
A.Y. 2017-2018

Match the elimination kinetics with their characteristic Answer:


description: 74. A
A. First order 75. A
B. Zero order 76. B

74. Increases in plasma drug concentration are matched by


First order (linear, Zero order (nonlinear,
increases in the rate of metabolism and excretion
nonsaturable) saturable)
75. The rate of drug loss is directly proportional to the
concentration of drug in the systemic circulation at that ● constant ● constant
time FRACTION AMOUNT
76. The clearance mechanisms become saturated at or near eliminated eliminated
the therapeutic concentration of the drug ● rate is ● rate is
concentration- concentration-
dependent INdependent
● half-life is ● half-life is
concentration- concentration-
INdependent dependent
● Most drugs

Match the mechanism of action of the ff drugs: Answer:


A. Prevents polymerization of intracellular protein 77. B
tubules to microtubule 78. C
B. Inhibits uric acid synthesis 79. A
C. Increases uric acid secretion
D. Inhibits uric acid migration into the joints Refer to #60

77. Allopurinol
78. Probenecid
79. Colchicine

Abraham, M. | Aquino, J. | Arada, M. | Aw, A. | Bagon, N. | Contreras, S. | Galapon, C.


Gison, R. | Obciana, D. | Palao, K. | Tangcueco, P. | Tumambing, M.
28
PHARMACOLOGY RATIONALE
A.Y. 2017-2018

Match the mechanisms of action of the ff antidepressants: Answer:


A. Fluoxetine 80. C
B. Mirtazapine 81. A
C. Duloxetine 82. D
D. Moclobemide
SSRI Paroxetine, Fluoxetine,
80. Selectively blocks serotonin and norepinephrine
Escitalo/Citalopram, Sertraline
reuptake
[Xet,Xet,Talo si Ser. SiSiRain ko sya]
81. Selectively blocks serotonin reuptake
82. Reversibly inhibits MOA-A SNRI Venlafaxine, Duloxetine
[SiNiRa ko yung fax machine sa dulo]

MAOI MAO-A: Moclobemide (RIMA)


MAO-B: Selegiline, Rasagiline
Both [PIT]: Phenelzine, Isocarboxazid,
Tranylcypromine

TCA Imipramine, Clomipramine, Nortriptyline,


Amitriptyline, Doxepin
[Take Care Ah, naPRAMI yung TRIP ni
DOC sepin]

NASA Mirtazapine [Nasa Morayta]

Match the adverse effects manifested by the following Answer:


patients who are on maintenance dose of anticonvulsant: 83. C
A. A 16 year old female with hirsutism and gingival 84. A
hyperplasia after several months of intake of this 85. B
anticonvulsant 86. D
B. A 3 year old boy presenting with hyperactivity and
irritability several months after intake of this drug Carbamazepine - spina bifida, SJS, rash
C. An infant with spina bifida born to a mother on Phenytoin - [HHH] Hirsutism, gingival Hyperplasia,
unrecalled anticonvulsant Hydantoin syndrome (craniofacial anomaly)
D. A 28 year old teacher developed peripheral visual Phenobarbital - sedation, irritability and
field defect after 2 years of treatment with this hyperactivity in children
anticonvulsant

83. Carbamazepine
84. Phenytoin
85. Phenobarbital
86. Vigabatrin

Abraham, M. | Aquino, J. | Arada, M. | Aw, A. | Bagon, N. | Contreras, S. | Galapon, C.


Gison, R. | Obciana, D. | Palao, K. | Tangcueco, P. | Tumambing, M.
29
PHARMACOLOGY RATIONALE
A.Y. 2017-2018

TRUE or FALSE: Write A if the statement is CORRECT and Answer:


B if statement is INCORRECT. 87. A
88. A
True regarding receptor antagonism: 89. A
87. A non-competitive antagonist acts by preventing the
receptor from being activated, even when the agonist is Refer to LE1
bound to the active site
88. A competitive antagonist reduces agonist potency
whereas a non-competitive antagonist reduces agonist
efficacy
89. A competitive antagonist blocks an agonist from
binding to the receptor, while maintaining the receptor in
the inactive form

TRUE statements regarding selective COX-2 inhibitors: Answer:


90. They have greater anti-inflammatory and analgesic 90. B
effects than non-selective COX inhibitors 91. B
91. They are very effective in preventing transient ischemic 92. A
attacks
92. They do not inhibit platelet aggregation

Regarding volume of distribution: Answer:


93. A drug with a higher degree of plasma protein binding 93. B
will have a higher volume of distribution 94. B
94. Volume of distribution increases as clearance decreases 95. B
95. The binding of drugs to tissue proteins has no
relationship to the overall distribution of drug in the body

Abraham, M. | Aquino, J. | Arada, M. | Aw, A. | Bagon, N. | Contreras, S. | Galapon, C.


Gison, R. | Obciana, D. | Palao, K. | Tangcueco, P. | Tumambing, M.
30
PHARMACOLOGY RATIONALE
A.Y. 2017-2018

2ND LONG EXAM

QUESTION ANSWER & RATIONALE

1. Which of the following diuretics would be most Answer: B.


useful in the acute treatment of a comatose patient
with brain injury and cerebral edema? Mannitol - prototype osmotic diuretic
Mechanism of Action: acts primarily in the proximal
A. Acetazolamide tubule and descending limb of Henle. Since Mannitol is
B. Mannitol an osmotically active agent, it causes water to be
C. Furosemide retained in these segments and promotes water
D. Ethacrynic Acid diuresis. It sets up an osmotic gradient causing the
water to seep out of the neurons and be eventually
excreted. (Kaya acute lang - meaning IV bolus. If given
as continuous infusion, it may exacerbate the ICP if
nagpunta sya sa interstitial space)

2. A 70 year old patient complains of paresthesias and Answer: C.


occasional nausea with one of her maintenance drugs.
She also has hyperchloremic metabolic acidosis. She Acetazolamide - prototype carbonic anhydrase inhibitor
is probably taking:
Large doses can cause drowsiness and paresthesias.
A. Furosemide
B. HCTZ MOA: inhibits carbonic anhydrase found in
C. Acetazolamide the epithelial cells of the PCT → decreased
D. Mannitol reabsorption of bicarbonate.

It‟s known for causing hyperchloremic metabolic


acidosis (Metabolic acidosis is due to decreased
bicarbonate stores in the body. Remember the concept
of anion gap, since nagdecrease ng concentration of
bicarbonate, another anion should fill its place, therefore
may increase Cl concentration, hence, hyperchloremic
metabolic acidosis.)

ACIDazolamide causes ACIDOSIS

Note: Diuretic efficacy of CAIs only last for 2-3 days but
the acidosis persists as long as you are taking the drug.

Others:
Loop diuretics - hypokalemic metabolic alkalosis
Thiazides - hypokalemic metabolic alkalosis
Potassium-sparing diuretics - hyperchloremic
metabolic acidosis

Abraham, M. | Aquino, J. | Arada, M. | Aw, A. | Bagon, N. | Contreras, S. | Galapon, C.


Gison, R. | Obciana, D. | Palao, K. | Tangcueco, P. | Tumambing, M.
31
PHARMACOLOGY RATIONALE
A.Y. 2017-2018

3. Which of the following is an important effect of Answer: D.


chronic use of loop diuretics?
Loop diuretics - dose-related ototoxicity - REVERSIBLE
A. Metabolic acidosis *most notorious for inducing ototoxicty - Ethacrynic
B. Elevation of pulmonary vascular pressure Acid
C. Decreased urinary excretion of Calcium *BUT advantage of Ethacrynic Acid is hindi sya
D. Ototoxicity sulfonamide derivative (phenoxyacetic acid derivative
sya) so lesser allergic reactions (SJS/TEN)/can be used in
patients with sulfa allergy

*Metabolic acidosis - K-sparing diuretics, CAIs


*Decreased urinary excretion of Calcium - Thiazides
*Elevation of pulmonary vascular pressure - can be
caused by osmotic diuretics like Mannitol (by causing an
osmotic gradient, water moves out causing an
expansion of the ECF leading to inc pulmonary pressure)

Additional:
Thiazide with carbonic anhydrase inhibitor property -
Chlorthalidone

4. Which of the following diuretics given in Answer: B.


combination with saline infusion would be most
useful in the treatment of severe hypercalcemia? Furosemide - loop diuretic
MOA: inhibits NKCC2 in the TAL of the loop
A. HCTZ of henle → decreased reabsorption of NaCl →
B. Furosemide decreased lumen-positive potential (under
C. Mannitol normal circumstances, the positive potential
D. Spironolactone promotes the reabsorption of divalent
cations like Mg and Ca) → increased urinary
excretion of these divalent cations.

NOTE: although it increases urinary excretion of


Calcium, it does not generally cause hypocalcemia due
to other factors like Vitamin D and PTH. It can, however,
cause significant hypomagnesemia in some cases.

Possible DDIs of Loop diuretics:


*avoid with Digoxin - increased risk of hypokalemia
*avoid with Aminoglycosides - ototoxicity

Abraham, M. | Aquino, J. | Arada, M. | Aw, A. | Bagon, N. | Contreras, S. | Galapon, C.


Gison, R. | Obciana, D. | Palao, K. | Tangcueco, P. | Tumambing, M.
32
PHARMACOLOGY RATIONALE
A.Y. 2017-2018

5. This diuretic has its major effect in the proximal Answer: C


tubule, descending limb of the loop of Henle, and
cortical collecting duct: Mannitol acts primarily in the proximal tubule and
descending limb of Henle. It also has actions on the
A. Furosemide cortical collecting duct (opposes action of ADH)
B. Acetazolamide
C. Mannitol Other choices:
D. HCTZ Furosemide - loop diuretic, TAL of the loop of Henle
Acetazolamide - acts on the carbonic anhydrase
enzyme in the epithelial cells of the PCT
HCTZ - inhibits the NaCl transport channel in the DCT
(Thiazide → Diazide HAHA)

6. This drug increases the formation of dilute urine in Answer: B


water-loaded subjects and is used to treat SIADH:
Conivaptan - ADH antagonist, against both V1a
A. Acetazolamide and V2 receptors. IV only → conIVaptan
B. Conivaptan
C. Amiloride Amiloride - K-sparing diuretic that acts by inhibiting
D. Desmopressin Na+ influx
Desmopressin - ADH analogue (similar effects with
Vasopressin)

7. The pharmacodynamics action of Methyldopa is: Answer: B

A. Blockade of presynaptic alpha 2 adrenergic BP - goal mo is to decrease CO or TPR. Remember that


neurons among the alpha and beta receptors, alpha 2 yung
B. Reduction of sympathetic outflow in the CNS inhibitory/taksil sa sympathetic (2 = 2nd choice lang sya
C. Blockade of reuptake of NE in the PNS so taksil sya hahaha)! So if you stimulate the presynaptic
D. Enhancement of postsynaptic alpha 2 alpha 2 receptor, you‟ll get parasympa effects)
adrenergic neurons
Methyldopa - presynaptic alpha 2 AGONIST not
antagonist (which makes choice A wrong). Also take
note that Methyldopa acts in the CNS (brainstem) to
reduce sympathetic outflow. Primarily used for HTN in
pregnancy

Other choices:
*Blockade of reuptake of NE in the PNS - can be
Cocaine. If blockade of vesicular storage - Reserpine.
*Choice D - should be enhancement of presynaptic
alpha 2 receptors (post-synaptic alpha-1 blockade is the
mechanism of alpha blockers such as Prazocin)

Abraham, M. | Aquino, J. | Arada, M. | Aw, A. | Bagon, N. | Contreras, S. | Galapon, C.


Gison, R. | Obciana, D. | Palao, K. | Tangcueco, P. | Tumambing, M.
33
PHARMACOLOGY RATIONALE
A.Y. 2017-2018

8. Lipid soluble beta blockers have the following Answer: C


pharmacokinetic profile:

A. Metabolized in the periphery


Lipid-soluble Water-soluble
B. Consistent bioavailability
C. Short half life Absorption Almost Incomplete
D. Incomplete oral absorption complete

Bioavailability Highly variable Less variable

Half-life Relatively short Longer than


average

Metabolism Hepatic (with No considerable


first pass effect) metabolism

Elimination GI, Kidney Unchanged in


the kidneys

Examples Propranolol Labetalol


Metoprolol Atenolol
Bisoprolol Nadolol
Carvedilol Timolol

WATER the
pLANT (except
P!!! Hehe)

*Note: Lipid soluble can cross the BBB → CNS


effects like confusion and lethargy

9. ACE Inhibitors are contraindicated in patients with: Answer: D

A. Heart failure
B. Mild renal impairment due to CKD
C. Hypokalemia
D. Bilateral renal artery stenosis

10. This antihypertensive inhibits vesicular Answer: C


catecholamine transport and storage:
*Other choices:
A. Methyldopa Methyldopa - see above
B. Guanabenz Guanabenz, Guanfacine - alpha-2 agonists in the
C. Reserpine brainstem resulting in reduced sympathetic outflow
D. Guanfacine (similar to Methyldopa)

Abraham, M. | Aquino, J. | Arada, M. | Aw, A. | Bagon, N. | Contreras, S. | Galapon, C.


Gison, R. | Obciana, D. | Palao, K. | Tangcueco, P. | Tumambing, M.
34
PHARMACOLOGY RATIONALE
A.Y. 2017-2018

11. Beta blocker with an antioxidant action: Answer: B

A. Propranolol Nebivolol - B1 selective with direct vasorelaxant effect


B. Nebivolol (due to generation of NO) and antioxidant effect
C. Telmisartan
D. Atenolol Beta blockers:

Selective: BEAM (Betaxolol, Bisoprolol, Esmolol,


Acebutolol, Atenolol, Metoprolol) + Nebivolol + Celiprolol
Non-selective: NSTP (Nadolol, Sotalol, Timolol,
Propranolol) + Carteolol
Vasodilatory Non-selective: Carve, Pin, and
Label → Carvedilol, Pindolol, Labetalol
Vasodilatory Selective: Celiprolol

Labetalol - HEPATOTOXICITY(L for liver)

12. A 65 year old male complains of dribbling urine Answer: A


with increased nighttime urination. BP is 150/80.
What is the most appropriate drug to give? Dribbling urine with increased nighttime urination in a
65 year old male is suggestive of BPH.
A. Terazosin
B. Metoprolol DOC for hypertensive patients with BPH - alpha
C. Spironolactone adrenergic antagonists (lahat ng may zocin, ie
D. Eplerenone Prazocin, Doxazocin). This is because the
urinary bladder has alpha receptors as well.
Inhibition of alpha receptors in the bladder
will promote bladder sphincter and prostate
relaxation → increase urinary outflow.
Hitting 2 birds with 1 stone! Haha

*Important beneficial effect of alpha blockers - it


decreases TG and LDL and increases HDL (as opposed
to beta blockers - decreases HDL)

*Doxazocin - discontinued because of cardiovascular


toxicity (increased risk for CHF)

Abraham, M. | Aquino, J. | Arada, M. | Aw, A. | Bagon, N. | Contreras, S. | Galapon, C.


Gison, R. | Obciana, D. | Palao, K. | Tangcueco, P. | Tumambing, M.
35
PHARMACOLOGY RATIONALE
A.Y. 2017-2018

13. A 50 year old male was recently prescribed with Answer: C


an antihypertensive drug due to uncontrolled BP. Two
weeks later, he developed malar rash with joint pains. Malar rash with joint pains → LUPUS-like
What is the most likely agent given to him? reaction

A. Clonidine Hydralazine causes drug-induced lupus reaction


B. Reserpine primarily due to its hydrazine derivative. Other drugs
C. Hydralazine that can cause lupus-like reactions:
D. Doxazocin
HIP
Hydralazine
Isoniazid
Procainamide

Also, Quinidine and Minocycline

14. A 50 year old female with hypertension and DM of Answer: D


5 years duration has ++ protein in the urine. GFR is 60
mL/min. The most appropriate drug to give is: ACEI/ARBs are the treatments of choice for
hypertension with comorbidities such as DM and CKD. It
A. Amlodipine slows down the progression of kidney disease due to
B. Clonidine hypertension and DM. May renal protective property sya
C. Metoprolol :)
D. Irbesartan
But never forget!!

Contraindicated in patients with bilateral renal artery


stenosis ang ACEI.

*If post-MI patient - give beta blockers

15. Which of the following must be avoided in a Answer: A.


patient with hypertension and clinical depression?
Anti-HTN drugs that can cause depression (usually mga
A. Reserpine centrally acting): RAMP
B. Verapamil
C. Enalapril Reserpine
D. Telmisartan A2 Adrenergic Antagonists (Guanabenz, Guanfacine,
Clonidine)
Methyldopa
Propranolol

Abraham, M. | Aquino, J. | Arada, M. | Aw, A. | Bagon, N. | Contreras, S. | Galapon, C.


Gison, R. | Obciana, D. | Palao, K. | Tangcueco, P. | Tumambing, M.
36
PHARMACOLOGY RATIONALE
A.Y. 2017-2018

16. A 30 year old G2P1 mother was noted to be Answer: A


hypertensive with BP 140/90 to 150/90 starting the
7th month AOG. The most appropriate anti- Anti-HTN drugs for pregnancy
hypertensive to prescribe is:
Methyldopa
A. Methyldopa Labetalol
B. Lisinopril Nifedipine
C. Losartan Hydralazine
D. Metoprolol

17. True of the antianginal property of Nitrates: Answer: C

A. They increase the level of cGMP in the arteries Review of concepts:


resulting to a decrease in diastolic wall stress
B. They increase the level of cGMP in the Preload - end diastolic volume, basically the amount of
myocardium resulting to a decrease in blood that returns to the heart from the systemic
contractility circulation (think of stretch)
C. They increase the level of cGMP in the veins
resulting to a decrease in preload Afterload - resistance that the heart has to overcome in
D. They decrease the level of cGMP in the order to pump blood to the systemic circulation (think of
coronary arteries resulting to vasoconstriction squeeze)

If you give a vasodilator (i.e. Nitrates) → may lesser


resistance to overcome therefore decrease in
the afterload. But at the same time, there
is decreased venous return to the heart
since dilated nga si blood vessels →
decreased end diastolic volume → decreased
preload (Aside from nitrates, diuretics can
also decrease the preload since it decreases
the overall circulating volume)

Other choices:
*choice A - nitrates will decrease systolic wall stress due
to its systemic vasodilatory property
*choice B - no direct effect in the contractility of the
heart. Its actions are mainly in smooth muscles. But
indirectly, it can cause tachycardia as compensatory
mechanism.
*choice D - coronary vasodilation but its systemic
vasodilation effect is still more prominent

Additional info: Nitrates decrease both preload and


afterload. Low dose nitrates cause venodilation (dec
preload) and high dose nitrates cause vasodilation (dec
afterload)

Abraham, M. | Aquino, J. | Arada, M. | Aw, A. | Bagon, N. | Contreras, S. | Galapon, C.


Gison, R. | Obciana, D. | Palao, K. | Tangcueco, P. | Tumambing, M.
37
PHARMACOLOGY RATIONALE
A.Y. 2017-2018

18. A 50 year old female, previously diagnosed to Answer: B


have chronic stable angina and hypertension, presents
with palpitation. The 12L ECG shows atrial fibrillation For CSAP, you usually give beta blockers and CCBs. For
with rapid ventricular response. Which of the Prinzmetal angina, usually CCBs ang DOC.
following antianginal drugs is the best one to give?
Patient has AF so you have to give a drug that can
A. Amlodipine manage both the angina and the arrhythmia of the
B. Verapamil patient. So this narrows down your choices into
C. Isosorbide mononitrate Diltiazem and Verapamil. Both Verapamil and Diltiazem
D. Diltiazem decrease the AV nodal conduction thereby reducing
ventricular response.

But Verapamil is more cardioselective (90%) so it‟s


more suitable. (Diltiazem = 50% BV, 50% cardio). Also,
Verapamil produces a more pronounced AV node
suppression (Diltiazem = more on SA node
suppression). Between the two, Verapamil also has a
better suppression of cardiac contractility (since more
cardioselective nga sya)..

*Avoid the nitrates since it can cause reflex tachycardia


which can aggravate the palpitations.

19. A 60 year old janitor has chest pains on walking 2 Answer: A


blocks. Which of the following beta blockers is the
most appropriate to give? Review of concepts:

A. Bisoprolol Remember beta blockers with ISA (intrinsic


B. Pindolol sympathomimetic activity) - these are beta blockers
C. Penbutolol which act as partial agonists. It partially activates the
D. Acebutolol receptor, but also inhibits the binding of NE to the
sympathetic receptors. Importance of this is its
usefulness in the treatment of hypertension since it can
decrease the blood pressure without causing major
changes in the cardiac output and heart rate (lesser
bradycardia).

However in angina, it is more useful to give


cardioselective beta blockers without ISA. By
decreasing the heart rate, you decrease the
cardiac workload/oxygen demand → reduced
anginal episodes.

Cardioselective beta blockers w/o ISA: MABB


Metoprolol
Atenolol
Bisoprolol
Betaxolol

Abraham, M. | Aquino, J. | Arada, M. | Aw, A. | Bagon, N. | Contreras, S. | Galapon, C.


Gison, R. | Obciana, D. | Palao, K. | Tangcueco, P. | Tumambing, M.
38
PHARMACOLOGY RATIONALE
A.Y. 2017-2018

20. This drug is an enzyme that converts plasminogen Answer: D


to plasmin:
RivaroXAban - direct factor XA inhibitor (all with XA
A. Rivaroxaban except for Enoxaparin)
B. Unfractionated heparin Enoxaparin - LMW heparin (inhibits factor Xa, lesser
C. Enoxaparin inhibition on antithrombin)
D. Streptokinase Unfractionated heparin - HMW heparin (inhibits both
factor XA and antithrombin)

21. A 70 year old male consults because of chest pain. Answer: C


His blood pressure in 90/60. Which of the following
antianginal medications is the safest drug to give? The patient is already hypotensive so avoid
anti-hypertensive drugs such as choices A
A. Verapamil and B. Nitrates cause vasodilation →
B. Diltiazem orthostatic hypotension.
C. Trimetazidine
D. Isosorbide mononitrate Trimetazidine MOA - inhibits fatty acid oxidation
therefore the heart uses more of glucose and less of
ATP (no effect on BP)

Another drug that does not affect BP - Ivabradine

22. Which of the the following acute myocardial Answer: B


infarction patients must be given a fibrinolytic agent?
Indications for thrombolytic therapy in MI
A. 58 year old male presenting with chest pain
for 2 hours duration with ECG of 3 mm ST 1. Chest pain suggestive of MI
depression in II, III, and avF 2. ST segment elevation of >0.1 mV (1 mm) in 2 or more
B. 60 year old male presenting with chest pain of contiguous ECG limb leads or > 0.2 mV (2 mm) in the
4 hours with ECG of 4 mm ST elevation in V1- chest leads or new or presumably new LBBB, strongly
5 suspicious for injury (LBBB obscure ST segment
C. 45 year old male presenting with chest pain of analysis)
3 hours duration with normal ECG 3. Symptom onset < 12 hours (Class I) or 12-24 hours
D. 50 year old male presenting with chest pain of with continuing pain (Class IIa)
3 hours duration the day before with 3 mm ST
elevation in II, III, and avF

23. This vasodilator provides myocardial protection Answer: C


via preconditioning by activation of cardiac Katp
channels: Other choices:
Ranolazine - reduces late sodium current (RanoLAzine
A. Diltiazem for LAte) that facilitates Ca entry via the
B. Bisoprolol Na-Ca exchanger → decreased intracellular Ca
C. Nicorandil → decreased diastolic wall stress
D. Ranolazine

Abraham, M. | Aquino, J. | Arada, M. | Aw, A. | Bagon, N. | Contreras, S. | Galapon, C.


Gison, R. | Obciana, D. | Palao, K. | Tangcueco, P. | Tumambing, M.
39
PHARMACOLOGY RATIONALE
A.Y. 2017-2018

24. Which of the following has the beneficial effect of Answer: B


delaying the progression of heart failure and decrease
sudden death and recurrent MI? ACEI → vasodilation → lesser cardiac
workload → delayed progression of HF
A. Amlodipine (handout)
B. Enalapril
C. Verapamil
D. Trimetazidine

25. A 50 year old female presents with tachycardia. Answer: A


PE findings are essentially normal except for CR of
110/min. ECG showed sinus tachycardia. This drug Ivabradine - selective If sodium channel
can decrease her heart rate without producing other blocker (funny current) → inhibits
hemodynamic effects: hyperpolarization in the SA node. It is a
bradycardic drug with NO hemodynamic effects (does
A. Ivabradine not affect the CO and BP)
B. Ranolazine
C. Allopurinol IVabradine = If sodium channel
D. Fasudil

26. Most angiotensin receptor blockers have no active Answer: D


metabolites EXCEPT:
WITH active metabolites: Losartan, Candesartan
A. Irbesartan cilexetil
B. Telmisartan NOT affected by food: Irbesartan, Candesartan cilexetil
C. Valsartan OD/BD dosing: Losartan (the rest are OD only)
D. Losartan

27. A 60 y/o male is admitted for peripheral edema Answer: D


and metabolic alkalosis. Which of the following drugs
is most appropriate for his treatment? Acetazolamide can be given for metabolic alkalosis since
it increases bicarbonate excretion.
A. HCTZ
B. Furosemide
C. Mannitol
D. Acetazolamide

28. A 40 y/o female has a history of frequent episodes Answer: A


of renal colic with calcium-containing renal stones.
What is the most appropriate diuretic to give? Thiazides can be given in nephrolithiasis due to
hypercalciuria by enhancing Ca reabsorption in the
A. HCTZ proximal tubules.
B. Furosemide
C. Mannitol *Other indications of thiazides:
D. Spironolactone `HTN
`HF
`Nephrogenic DI

Furosemide will increase urinary Ca excretion which will


aggravate the problem.

Abraham, M. | Aquino, J. | Arada, M. | Aw, A. | Bagon, N. | Contreras, S. | Galapon, C.


Gison, R. | Obciana, D. | Palao, K. | Tangcueco, P. | Tumambing, M.
40
PHARMACOLOGY RATIONALE
A.Y. 2017-2018

29. Hyponatremia due to thiazide diuretics is due to: Answer: C

A. Decreased thirst mechanism Thiazide-induced hyponatremia are caused by:


B. Hypovolemia-induced decrease in ADh 1. Hypovolemia-induced elevation of ADH
C. Reduction in the diluting capacity of the 2. Reduction in the diluting capacity of the kidney
kidney 3. Increased thirst
D. Excessive urinary losses of Na
Prevention: decrease water intake, reduce drug dose

30. A patient with long-standing diabetic Answer: B


nephropathy and hyperkalemia and recent-onset mild
heart failure requires a diuretic. Which of the Among the choices, HCTZ is the safest to
following agents would be the safest to give? give. You cannot give choice C and D → K-
sparing diuretics which will aggravate the
A. Acetazolamide hyperkalemia. Acetazolamide has limited
B. HCTZ diuretic activity but pronounced acidosis.
C. Triamterene
D. Amiloride *NOTE: Usually you give ACEI/ARBs in diabetics and
patients with HF pero avoid these drugs din if the
patient has hyperkalemia.

QUANTITATIVE COMPARISON: Answer:

Write A if 1>2, B if 1<2, or C if 1=2 31. B


HCTZ causes Ca reabsorption.
31. Urine Calcium: (1) HCTZ (2) Furosemide
32. Serum Magnesium: (1) HCTZ (2) Furosemide 32. A
33. Serum Uric Acid: (1) Furosemide (2) Furosemide causes increased urinary Mg excretion
Spironolactone leading to pronounced hypomagnesemia

33. A
Furosemide can cause hyperuricemia due to
hypovolemia-induced enhancement of uric acid
absorption in the proximal tubule

Abraham, M. | Aquino, J. | Arada, M. | Aw, A. | Bagon, N. | Contreras, S. | Galapon, C.


Gison, R. | Obciana, D. | Palao, K. | Tangcueco, P. | Tumambing, M.
41
PHARMACOLOGY RATIONALE
A.Y. 2017-2018

MATCHING TYPE: Answer:

Match the following anticoagulants with their 34. A


corresponding mechanism of action: 35. C
36. B
34. Blocks the carboxylation of several glutamate
residues in prothrombin and factors 7,9,10 Other choices:
35. Binds tightly to- and produces a conformational
change to- antithrombin RivaroXAban- inhibits Factor Xa. Used in prevention of
36. Binds directly to the active site of thrombin VTE following hip or knee surgery and stroke in patients
with AF
A. Warfarin
B. Dabigatran
C. Heparin
D. Rivaroxaban

Abraham, M. | Aquino, J. | Arada, M. | Aw, A. | Bagon, N. | Contreras, S. | Galapon, C.


Gison, R. | Obciana, D. | Palao, K. | Tangcueco, P. | Tumambing, M.
42
PHARMACOLOGY RATIONALE
A.Y. 2017-2018

Match the drug with its adverse effect: Answer:

37. Spironolactone 37. B


38. Hydrochlorothiazide 38. A
39. Mannitol 39. C [See #3]

A. Hyperglycemia Adverse effects:


B. Hyperkalemia
C. Pulmonary Edema Thiazides: hyperGLUC - hyperglycemia, lipidemia,
D. High risk of UTI uricemia, calcemia

Mechanism of hyperuricemia in thiazides → compete


w/ the tubular secretion of uric acid by the
organic acid secretory system in the
proximal tubule

Loop diuretics: hyperGLU - hyperglycemia, lipidemia,


uricemia (but if hyperglycemia and hyperlipidemia then
may thiazides and loop sa choices - choose thiazides!) +
ototoxicity, hypomagnesemia, and hypokalemia

Hyponatremia: Thiazides > Loop diuretics


Both thiazides and loop diuretics are sulfonamide
derivatives (EXCEPT ETHACRYNIC ACID) → think
of allergic reactions

Spironolactone → has activity on androgen and


progesterone receptors → GYNECOMASTIA,
IMPORTENCY, BPH
If you want a K-sparing diuretic with better
selectivity for the mineralocorticoid
receptor → EPLERENONE (slows progression of
albuminuria in diabetic patients)
TRIAMTERENE → kidney stones
TRIAMTERENE + INDOMETHACIN → acute renal
failure

Remember: Aldosterone antagonists depend on renal


PG production. Therefore, its effects can be reduced by
NSAIDs.

MANNITOL - sya lang kakaiba since HYPERNATREMIA


sya because you‟re losing free water. Kaya important to
hydrate!!. (EXCEPT: in patients with severe renal
impairment → hyponatremia)

Abraham, M. | Aquino, J. | Arada, M. | Aw, A. | Bagon, N. | Contreras, S. | Galapon, C.


Gison, R. | Obciana, D. | Palao, K. | Tangcueco, P. | Tumambing, M.
43
PHARMACOLOGY RATIONALE
A.Y. 2017-2018

Match these patients with Prinzmetal angina with the Answer:


best drug to give:
40. C
40. 50 years old with BP of 180/100, PR of 80/min, in Patient is hypertensive w/ normal PR so you can give an
sinus rhythm anti-hypertensive drug like Nifedipine.
41. 46 years old with BP of 140/90, PR of 105/min, in
sinus rhythm 41. B
42. 55 years old with BP of 130/90, PR of 115/min, in Patient is tachycardic so avoid choices C and D (both
atrial fibrillation can cause reflex tachycardia)

A. Verapamil 42. A
B. Diltiazem Verapamil is useful for AF.
C. Nifedipine
D. Isosorbide mononitrate

Match the following patients with the most Answer:


appropriate anti-platelet agent:
43. B
43. 70 year old male with intermittent claudication Cilostazol - PDE inhibitor that promotes vasodilation,
44. 65 year old female with CSAP and cholelithiasis primarily used to treat intermittent claudication.
with recent episode of colic
45. 50 year old male with non-ST elevation MI and (The other PDE inhibitor, Dipyridamole - “coronary
just had Percutaneous Coronary Intervention (PCI) steal” phenomenon because it promotes vasodilation of
normal arteries)
A. Clopidogrel
B. Cilostazol 44. C
C. Ticagrelor **not entirely sure** Ticagrelor is usually given to
D. Abciximab patients undergoing surgery since its ADP inhibition is
REVERSIBLE. (Patient has cholelithiasis with colic
episodes, meaning symptomatic na therefore, surgical
candidate).

45. D
Abciximab is a GIIb/IIIa inhibitor (receptor for platelet
aggregation). Used primarily in PCI and ACS.

Other GIIb/IIIa inhibitors:


Eptifibatide - carboxyl terminal similar with delta chain
of fibrinogen (EptifibatiDELTA)
Tirofiban - contains AA Arg-Gly-Asp (RGD) that
recognizes integrin receptors

Indications:
*For Non-Q MI - Eptifibatide, Tirofiban
*Unstable angina - Eptifibatide, Tirofiban
*Unstable angina/non-Q wave MI patients undergoing
PCI - Abciximab, Eptifibatide

<also take note of the contraindications sa handout>

Abraham, M. | Aquino, J. | Arada, M. | Aw, A. | Bagon, N. | Contreras, S. | Galapon, C.


Gison, R. | Obciana, D. | Palao, K. | Tangcueco, P. | Tumambing, M.
44
PHARMACOLOGY RATIONALE
A.Y. 2017-2018

Match the drug with its general mechanism in Answer:


decreasing blood pressure:
46. C
46. Carvedilol 47. B
47. Nicardipine 48. C
48. HCTZ 49. B
49. Losartan

A. Decrease in CO
Decreased CO Decreased TPR
B. Decrease in TPR
C. Both Diuretics Diuretics
D. Neither Beta blockers Alpha blockers
Adrenergic blockers
CCB
ACEI
ARBs

For #46: Remember the VASODILATORY beta blockers.

Remember: BP = CO x TPR
CO - influenced by preload and contractility
TPR - influenced by functional constriction and/or
structural hypertrophy

Any drug that decreases the preload or the contractility


will decrease the cardiac output (beta blockers - dec
contractility; diuretics - decrease circulating volume
thereby decreasing the preload)

TRUE OR FALSE. A if correct, B if incorrect. Answer:

True regarding diuretics: 50. B [see #4]


50. Loop diuretics generally cause hypocalcemia. 51. B [should be hypercalciuria]
51. Phosphaturia and hypercalcemia occur during the 52. A
bicarbonaturic response to inhibitors of carbonic Increased Na+ delivery to the collecting tubule will
anhydrase. promote K secretion. To counteract this, you can
52. Potassium wasting is theoretically a problem with simultaneously give K-sparing diuretics.
any diuretic that increases Na+ delivery to the
collecting tubule.

Abraham, M. | Aquino, J. | Arada, M. | Aw, A. | Bagon, N. | Contreras, S. | Galapon, C.


Gison, R. | Obciana, D. | Palao, K. | Tangcueco, P. | Tumambing, M.
45
PHARMACOLOGY RATIONALE
A.Y. 2017-2018

True regarding use of antihypertensive drugs: Answer:

53. If Atenolol and Verapamil are given together at 53. A


high doses, there is a risk of development of third 3rd degree AV block = complete AV block
degree AV block. Both Verapamil and Atenolol causes AV nodal block
54. If Felodipine and Metoprolol are given together,
the heart rate is expected to increase. 54. B
55. If Furosemide is given concomitantly with Felodipine - reflex tachycardia
Amlodipine, the serum K+ level is expected to Metoprolol - bradycardia
increase. *balanced*

55. A

Abraham, M. | Aquino, J. | Arada, M. | Aw, A. | Bagon, N. | Contreras, S. | Galapon, C.


Gison, R. | Obciana, D. | Palao, K. | Tangcueco, P. | Tumambing, M.
46
PHARMACOLOGY RATIONALE
A.Y. 2017-2018

2ND SHIFTING EXAM

QUESTION ANSWER & RATIONALE

1. The following structure, phenylethylamine, is Answer: D


the parent compound of most adrenergic drugs.
Increasing the alkyl substituents on this part of Structure activity relationships of sympathomimetic drugs
the phenylethylamine molecule increase B2 Substitution of the benzene ring
selectivity ● Substitution of hydroxyl groups at the 3- and 4-
A. alpha carbon positions of the benzene ring converts benzene to
B. beta carbon catechol = maximal α and β effects
C. on the benzene ring ● Absence of one or the other substituent at the 3-position =
D. terminal amino group decreased α effect 100-fold & negligible β effect
● Substitution of hydroxyl groups at the 3- and 5-
position = β2 selectivity

Unsubstituted benzene ring


● Loss either of the 2 hydroxyl groups enhances oral
effectiveness and duration of action because the drug
is no longer metabolized by COMT, also increases
CNS effects
● Catecholamines are metabolized by COMT
● Noncatecholamines are metabolized by MAO.

Substitution at the β carbon


● Important for storage of sympathomimetic amines in
neural vesicles

Substitution at the amine side chain


● Substitution of bulky structures (eg. alkyl group) to
amino group increases β2 selectivity, decreases affinity for
α receptors, and protects against metabolism by COMT
○ Epinephrine - 1 methyl
○ Isoproterenol - 2 methyl
○ Terbutaline - 3 methyl

[ Alkyl - terminal amino - B2 ]

2. B2-agonist inhalation may lead to a decrease Answer: D


in this electrolyte increasing the risk for muscle
cramps B2 agonists promote the entry of potassium into the cells
A. calcium which leads to hypokalemia and increases the risk for muscle
B. sodium cramps.
C. chloride
D. potassium

Abraham, M. | Aquino, J. | Arada, M. | Aw, A. | Bagon, N. | Contreras, S. | Galapon, C.


Gison, R. | Obciana, D. | Palao, K. | Tangcueco, P. | Tumambing, M.
47
PHARMACOLOGY RATIONALE
A.Y. 2017-2018

3. Which of the following B2 agonist have the Answer: B


longest duration of action and can be given
once daily as a controller Short acting: salbutamol, terbutaline
A. formoterol Long acting: formoterol, salmeterol
B. vilanterol Ultra long acting: indacaterol, olodaterol, vilanterol
C. terbutaline
D. salmeterol VILLAIN (vilanterol) has a very long duration of action
Masamang damo matagal mamatay
[ VILLAINterol - long DOA ]

4. This fixed dose formulation can be used both Answer: C


as a rescue drug for asthma exacerbation and
for long term asthma control Give LABA (formoterol) + ICS (budesonide)
A. salbutamol + ipratropium
B. montelukast + levocetirizine Formoterol has the best of both worlds. First, it is long acting.
C. formoterol + budesonide Second, its onset of action is faster than that of salmeterol.
D. tiotropium + indacaterol Thus, it is the only LABA that is approved for quick relief of
asthma exacerbations.

5. Rapidly acting anti-inflammatory effects of Answer: A


steroid is mediated by what mechanism(s)?
A. signaling membrane associated receptors Steroid-receptor complexes act in 3 ways:
and second messenger ● Direct genomic pathway: choice B
B. steroid receptor complex translocating to the ● Indirect genomic pathway: choice C
nucleus and binding to glucocorticoid response ● Nongenomic pathway: choice A
element (GRE) in the DNA
C. steroid receptor complex binding directly to
transcription factors like AP-1 or NFkB in the
cytoplasm preventing translocation of the
complex to the nucleus

6. Glucocorticoids enhance reabsorption of this Answer: B


electrolyte that may lead to hypertension:
A. calcium Glucocorticoid effects on electrolytes and water balance:
B. sodium ● Increases reabsorption of sodium from tubular fluid
C. potassium ● Increases urinary excretion of K+ Ca++ H+
D. bicarbonate
[ glucocorticoid - reabsorb Na - excrete KCaH ]

7. In the absence of glucocorticoids, end organ Answer: D


responses to this group of hormone, so called
permissive effect, is impaired: In the absence of glucocorticoids, end organ responses to
A. estrogens catecholamines is impaired. This leads to:
B. thyroid ● Decreased vascular and bronchial muscle response to
C. insulin catecholamines
D. catecholamines ● Lipolytic response of fat cells to catecholamines, ACTH,
GH

Simply put, [ cortisol enhances the body‟s response to


catecholamines ]

Abraham, M. | Aquino, J. | Arada, M. | Aw, A. | Bagon, N. | Contreras, S. | Galapon, C.


Gison, R. | Obciana, D. | Palao, K. | Tangcueco, P. | Tumambing, M.
48
PHARMACOLOGY RATIONALE
A.Y. 2017-2018

8. A child was given prednisone for acute Answer: B


asthma attack. If CBC was taken 6 hours later,
what result should you expect? Neutrophilia would be expected 6 hrs after giving prednisone.
A. lymphocytosis
B. neutrophilia Glucocorticoid effects on formed elements of the blood:
C. monocytosis ↑PMNs from ↑ influx from BM, ↑Hgb and RBC
D. mastocytosis ↓lymphocytes, eosinophils, monocytes & basophils
Effect peaks at 4-6 hrs, lasts about 24 hrs

9. A balikbayan patient with congenital adrenal Answer: A


hyperplasia is maintained on hydrocortisone
tablet daily. If he runs out of hydrocortisone Relative potencies & equivalent doses of corticosteroids
supply and it is unavailable here, what is the
best alternative for him?
Corticosteroid Anti-inflam. Na+ Duratio Equiv.
A. prednisolone
potency retaining n of dose
B. methylprednisolone
potency action
C. betamethasone
D. dexamethasone Cortisol 1 1 S 20

Cortisone 0.8 0.8 S 25

Prednisone 4 0.8 I 5

Prednisolone 4 0.8 I 5

Methylprednisol 5 0.5 I 4
one

Triamcinolone 5 0 I 4

Betamethasone 25 0 L 0.75

Dexamethason 25 0 L 0.75
e

Fludrocortisone 10 125 S -

10. For salt retaining effect in patients with Answer: C


congenital adrenal hyperplasia, the drug of
choice is See #9
A. hydrocortisone Among the steroids listed in the table, fludrocortisone has the
B. prednisone greatest sodium retaining activity.
C. fludrocortisone
D. betamethasone

Abraham, M. | Aquino, J. | Arada, M. | Aw, A. | Bagon, N. | Contreras, S. | Galapon, C.


Gison, R. | Obciana, D. | Palao, K. | Tangcueco, P. | Tumambing, M.
49
PHARMACOLOGY RATIONALE
A.Y. 2017-2018

11. Which of the following steroids has the Answer: D


least mineralocorticoid activity and most
potent glucocorticoid activity? See #9
A. hydrocortisone Among the steroids listed in the table, dexamethasone (along
B. prednisone with betamethasone) has the greatest anti-inflammatory
C. fludrocortisone (glucocorticoid) activity and the least sodium retaining
D. dexamethasone (mineralocorticoid) activity.

12. Elimination of mineralocorticoid effect is Answer: C


achieved by addition of methyl group at:
A. C9 Addition of a methyl group at C16 eliminates MC effect.
B. C11
C. C16
D. C17

13. For long term treatment of autoimmune Answer: B


diseases, which of the following regimens at
equivalent doses is the least suppressive to the In treating life threatening chronic disease,
HPA axis? ● Start with high daily split dose
A. prednisone 15 mg bid after breakfast and ● Use short or intermediate acting steroid (prednisone,
dinner prednisolone, methylprednisolone)
B. prednisolone 30 mg qd after breakfast ● Daily dose is then consolidated into a single morning
C. dexamethasone 500 mcg bid after breakfast dose preferably at breakfast to stimulate normal diurnal
and dinner rhythm of endogenous cortisol secretion
D. dexamethasone 3 mg once every 72 hrs ● Switch to alternate day regimen with gradual tapering
to the lowest dose possible to allow the HPA axis to
recover on the off day.

14. A 34 y/o female presents with Kussmaul‟s Answer: C


respiration and the ABG confirmed metabolic
acidosis. She is probably receiving this diuretic. Spironolactone & eplerenone side effects:
A. hydrochlorothiazide ● Hyperkalemia
B. bumetanide ● Metabolic acidosis
C. spironolactone ● Gynecomastia (for spironolactone only)
D. mannitol

15. A 50 y/o executive wants to go on a Answer: A


mountain climbing with his friends. He consults
to get a prescription to prevent high-altitude Acetazolamide quick facts:
sickness. You will give him ● Carbonic anhydrase inhibitor
A. acetazolamide ● Inhibits bicarbonate reabsorption in the proximal tubule
B. amiloride ● Reduces secretion of aqueous humor in glaucoma
C. demeclocycline ● Causes metabolic acidosis which increases respiration
D. desmopressin thus preventing high-altitude sickness

For high-altitude sickness, give acetazolamide


[ high altitude - acetazolamide ]

Abraham, M. | Aquino, J. | Arada, M. | Aw, A. | Bagon, N. | Contreras, S. | Galapon, C.


Gison, R. | Obciana, D. | Palao, K. | Tangcueco, P. | Tumambing, M.
50
PHARMACOLOGY RATIONALE
A.Y. 2017-2018

16. A 58 y/o female consults because of a Answer: C


gouty flare after another drug was added to
her antihypertensive regimen. Which of the Hydrochlorothiazide side effects:
following is the most likely drug added? ● Hyperuricemia
A. captopril ● Impaired CHO tolerance
B. carvedilol ● Hypokalemic metabolic alkalosis
C. hydrochlorothiazide ● Hyponatremia (more severe than loop diuretics)
D. felodipine ● Impotence, erectile dysfunction

17. A 65 y/o male is admitted in the ER Answer: B


because of a “fainting spell” at home. He has no
history of trauma from his fall. His BP is 120/60 Loop diuretics cause the greatest diuresis compared with other
when lying down and 60/20 when he sits up. diuretics and this can lead to hypotension!
Neurologic examination and ECG are within
normal limits. History reveals that he has been
taking diuretics for a heart condition. Which of
the following drugs is the most likely cause of
his fainting spell?
A. amiloride
B. furosemide
C. hydrochlorothiazide
D. spironolactone

18. If a patient presents with pulmonary Answer: C


edema, which of the following must NOT be
given? Before diuresis sets in, there is an expansion of the ECF
A. furosemide volume. This is detrimental to a patient who already has a
B. hydrochlorothiazide volume overload (eg, pulmonary edema).
C. mannitol
D. spironolactone

19. B.F. 60 y/o newscaster was noted to be Answer: B


hypertensive during an executive check-up. His
fasting blood glucose was noted to be 150 ACE inhibitors decrease the production of angiotensin II which
mg/dL. Urinalysis showed trace amount of reduces blood pressure. They also slow the progression of
proteins. The initial drug of choice for his kidney disease due to high blood pressure or diabetes.
hypertension is
A. amlodipine Salient features: hypertensive, diabetic, trace protein
B. enalapril In this case, give an ACE inhibitor.
C. clonidine If patient experiences chronic cough, switch to ARBs.
D. diltiazem

Abraham, M. | Aquino, J. | Arada, M. | Aw, A. | Bagon, N. | Contreras, S. | Galapon, C.


Gison, R. | Obciana, D. | Palao, K. | Tangcueco, P. | Tumambing, M.
51
PHARMACOLOGY RATIONALE
A.Y. 2017-2018

20. B.F. a 20 y/o medical student consulted at Answer: A


the health clinic for palpitation. The 2D echo
revealed mitral valve prolapse. B.F. has #MustKnowConcept
frequent attacks of asthma. BP 130/85 Beta blockers (eg. propranolol) is a NO NO for asthmatics.
The drug that should NOT be given is
A. propranolol
B. diltiazem
C. amiodarone
D. esmolol

22. The main indication of this agent is Answer: C


hypertension during pregnancy
A. reserpine Methyldopa‟s main indication is hypertension in pregnancy
B. prazosin It is a centrally acting α2 agonist. It acts as an agonist at
C. methyldopa presynaptic α2 adrenergic receptors. Because it is centrally
D. clonidine acting, watch out for CNS depression. Also, watch out for side
effects such as hemolytic anemia and lupus-like syndrome.

23. This antianginal drug exerts its beneficial Answer: B


effect through inhibition of the late sodium
channel which is active during episodes of Quick review of MOA
ischemia Trimetazidine - pFOX inhibitor
A. trimetazidine Allopurinol - xanthine oxidase inhibitor
B. ranolazine Ranolazine - inhibits late sodium current
C. verapamil Ivabradine - selective If Na channel blocker in the SA node
D. ivabradine
[ Trime - pFOX ]
[ ranOLA - ola ola im LATE! ]
[ Iva - IfNa ]

24. A 68 y/o diabetic complains of substernal Answer: D


chest pain radiating to the left arm and jaw
which started 2 hours earlier. Which of the Indications for thrombolytic therapy in MI
following ECG changes is an indication to ● Chest pain suggestive of MI
giving a thrombolytic agent? ● ST segment elevation >0.1 mV (1 mm) in 2 or more
A. 2 mm ST depression in II, III, and avF contiguous ECG limb leads or >0.2 mV (2 mm) in the
B. diffuse T wave inversion in the chest leads chest leads or a new or presumably new LBBB,
C. 1 mm ST elevation V1-3 strongly suspicious for injury
D. 3 mm ST elevation in V3-6 ● Symptom onset <12 hours (Class I) or 12-24 hours
with continuing pain (Class IIa)

Absolute contraindications
● Previous hemorrhagic stroke
● Other stroke or CVA within 1 year
● Active internal bleeding (menses excluded)
● Suspected aortic dissection

Abraham, M. | Aquino, J. | Arada, M. | Aw, A. | Bagon, N. | Contreras, S. | Galapon, C.


Gison, R. | Obciana, D. | Palao, K. | Tangcueco, P. | Tumambing, M.
52
PHARMACOLOGY RATIONALE
A.Y. 2017-2018

25. A 50 y/o male on warfarin for atrial Answer: A


fibrillation wants to know which of the
following drugs can be safely taken without Drugs without effect on warfarin
any need to alter the warfarin‟s dose ● Ethanol
A. paracetamol ● Phenothiazines
B. rifampicin ● Benzodiazepines
C. amiodarone ● Acetaminophen/Paracetamol
D. cholestyramine ● Opioids
● Indomethacin
● Most antibiotics

Additional:
Buzz word for warfarin: valvular A-fib!
Buzz word for dabigatran: nonvalvular A-fib!

26. Which of the following antianginal drugs is Answer: D


the most appropriate to give to decrease heart
rate, without lowering the BP? #MustKnowConcept
A. bisoprolol Ivabradine can cause bradycardia, without causing other
B. diltiazem hemodynamic effects
C. verapamil
D. ivabradine

27. A 55 y/o male complains of chest Answer: D


discomfort when walking 2 blocks. Which of
the following drugs will likely enable him to Diltiazem and verapamil are calcium channel blockers.
walk longer without having an angina attack? Metoprolol is a beta blocker. All these drugs are anti-anginal
A. diltiazem drugs.
B. metoprolol
C. verapamil
D. all of the above

28. A 45 y/o male had a myocardial infarction Answer: A


a month ago. He consults because of
palpitation. The 24 hour Holter recordings Beta blockers prevent reinfarction, decrease mortality in
showed frequent premature ventricular patients with recent MI due to its antiarrhythmic effect and
contractions. Which of the following is the prevent ventricular remodeling
most appropriate to give?
A. bisoprolol Buzz word for beta blockers: post-MI patient
B. diltiazem
C. flecainide
D. lidocaine

Abraham, M. | Aquino, J. | Arada, M. | Aw, A. | Bagon, N. | Contreras, S. | Galapon, C.


Gison, R. | Obciana, D. | Palao, K. | Tangcueco, P. | Tumambing, M.
53
PHARMACOLOGY RATIONALE
A.Y. 2017-2018

29. For a drug to terminate an atrial fibrillation, Answer: A


which of the following must be its
pharmacodynamic property? Re-entry is the most common mechanism for arrhythmia.
A. it must terminate reentry circuits in the Multiple reentry results in A-fib. Now, if we want to terminate
atrium an A-fib, we would want to block re-entry.
B. it must increase the refractory period of the
AV node What are the 2 endpoints of A-fib?
C. it must decrease the spontaneous 1. Rate control
depolarization of the SA node 2. Medical cardioversion
D. all of the above

30. A 68 y/o female is brought in the ER Answer: D


because of loss of consciousness. BP=0, PR=0,
RR=0. The monitor shows ventricular This is pulseless V-tach, so choose defibrillation!
tachycardia. You will immediately perform
chest compression and Defib vs Cardioversion
A. give IV amiodarone ● Defibrillate in case of a pulseless V-tach or V-fib
B. give IV lidocaine ● Cardiovert in case of a tachyarrhythmia that is
C. do electrical cardioversion hemodynamically compromising
D. do defibrillation
[ pulseless v-tach/v-fib - defib ]
[ tachy hemo compromising - cardiovert ]

31. A 20 y/o male is known to have WPW. He Answer: D


presents in the clinic because of palpitation and
he has with him a 24 hr Holter recording that #MustKnowConcept
shows paroxysmal atrial fibrillation. Which of Digoxin is a NO NO for Wolff-Parkinson-White!
the following must NOT be given to him?
A. amiodarone
B. flecainide
C. sotalol
D. digoxin

32. A 35 y/o female presents in the ER because Answer: A


of supraventricular tachycardia (SVT). It was
rapidly terminated after she was given a drug Adenosine can be used to convert paroxysmal SVT to sinus
but the arrhythmia recurred very soon after. rhythm.
What is the most probable drug that was
given?
A. adenosine
B. verapamil
C. amiodarone
D. procainamide

Abraham, M. | Aquino, J. | Arada, M. | Aw, A. | Bagon, N. | Contreras, S. | Galapon, C.


Gison, R. | Obciana, D. | Palao, K. | Tangcueco, P. | Tumambing, M.
54
PHARMACOLOGY RATIONALE
A.Y. 2017-2018

33. This antiarrhythmic drug has a very long Answer: C


half-life
A. vernakalant Amiodarone is a broad spectrum antiarrhythmic agent. After
B. labetalol discontinuation, effects are still maintained for 1-3 months. Its
C. amiodarone hepatic metabolite, desethylamiodarone, is also bioactive. The
D. sotalol drug accumulates in the heart (10-50x more than the plasma),
lung, liver, skin and tears (this is where we‟d expect the side
effects)

#MustKnowConcept
Side effects of amiodarone:
● GI: NV, anorexia, esophageal reflux, ↑LFTs
● Pulmo: chronic interstitial fibrosis, acute ARDS
● Thyroid: reduces peripheral conversion of T4 to T3,
hypothyroidism, hyperthyroidism
● Cutaneous: blue gray discoloration of sun exposed skin,
photosensitivity
● Neurologic: ataxia, tremor, sleep disturbances,
peripheral neuropathy
● Ocular: corneal microdeposits, visual disturbances

34. Which of the following drugs has the Answer: D


slowest binding kinetic?
A. lidocaine #MustKnowConcept
B. encainide Amiodarone is used in medical cardioversion.
C. disopyramide
D. amiodarone

35. Recombinant human B type natriuretic Answer: A


peptide used to treat acute heart failure (AHF)
A. nesiritide Nesiritide is a recombinant BNP that causes potent vasodilation
B. dopamine in the venous and arterial vasculatures, resulting in significant
C. phenylephrine reductions in venous and ventricular filling pressures and mild
D. ezetimibe increases in cardiac output.. It exerts its activity by means of
guanylyl cyclase-linked NPR A and NPR B, causing cGMP-
mediated vasodilation.

Side effects of nesiritide:


● Hypotension
● Headache

Abraham, M. | Aquino, J. | Arada, M. | Aw, A. | Bagon, N. | Contreras, S. | Galapon, C.


Gison, R. | Obciana, D. | Palao, K. | Tangcueco, P. | Tumambing, M.
55
PHARMACOLOGY RATIONALE
A.Y. 2017-2018

36. Which dose of dopamine is used to provide Answer: A


better renal perfusion?
A. low Counterintuitive answer. But yes, low dose dopamine provides
B. intermediate better renal perfusion.
C. high
D. very high Dopamine effects according to dose:
● Low dose (≤ 2 ug/kg/min) - dilation of
renal, splanchnic and cerebral arteries;
increases RBF and natriuresis
● Intermediate dose (2-10 ug/kg/min) - enhanced NE
release, increases inotropy, mild peripheral
vasoconstriction
● High dose (≥ 10 ug/kg/min) - significant
vasoconstriction via α1 receptor stimulation, precipitates
heart failure and poor perfusion, limb and end organ
ischemia

37. Most common extracardiac site of digitalis Answer: C


toxicity
A. CNS #MustKnowConcept
D. endocrine glands The gastrointestinal tract is the most common extracardiac site
C. GIT of digitalis toxicity. This is caused both by direct effects on the
D. musculoskeletal GIT and indirectly by digitalis effect on the CNS.

Remember DIGOC-N
D - digoxin level ≥ 2 ng/mL is toxic
I - inhibits Na-K-ATPase
G - GI signs (nausea, vomiting, diarrhea, anorexia)
O - output, intake and weight should be monitored
C - CNS signs (disorientation, hallucination, visual disturbances,
color aberrations)
N - narrow therapeutic index

38. Reason behind proarrhythmic effect of Answer: A


digitalis: delayed afterdepolarization due to
increased intracellular Digoxin inhibits the activity of the Na-K-ATPase. This
A. Ca++ indirectly increases intracellular calcium.
B. Na+
C. K+
D. Mg++

Abraham, M. | Aquino, J. | Arada, M. | Aw, A. | Bagon, N. | Contreras, S. | Galapon, C.


Gison, R. | Obciana, D. | Palao, K. | Tangcueco, P. | Tumambing, M.
56
PHARMACOLOGY RATIONALE
A.Y. 2017-2018

39. Class of drugs with Class I indication for Answer: C


heart failure
A. Ca channel blockers Treatments recommended in ALL symptomatic patients with
B. centrally acting hypertensives heart failure with reduced ejection fraction (Class I indication)
C. ACE inhibitors ● ACE inhibitors
D. PDE inhibitors ● Beta-blockers
● Diuretics
● ARBs (alternative for ACEI intolerance)

Treatment recommended in SELECTED symptomatic patients


with heart failure with reduced ejection fraction
● Angiotensin receptor neprilysin inhibitor (ARNI) -
combination of valsartan and sacubitril (neprilysin
inhibitor)
○ Sacubitril slows the degradation of NPs,
bradykinin, and other peptides by inhibiting
neprilysin
○ Valsartan reduces vasoconstriction, sodium
and water retention, and myocardial
hypertrophy

40. With what drug is hydralazine combined Answer: C


and given to heart failure?
A. amlodipine A combination of hydralazine and isosorbide dinitrate may be
B. sacubitril considered in symptomatic patients with heart failure with
C. isosorbide dinitrate reduced ejection fraction who cannot tolerate ACEI nor ARB (or
D. metoprolol are contraindicated) to reduce mortality.

Abraham, M. | Aquino, J. | Arada, M. | Aw, A. | Bagon, N. | Contreras, S. | Galapon, C.


Gison, R. | Obciana, D. | Palao, K. | Tangcueco, P. | Tumambing, M.
57
PHARMACOLOGY RATIONALE
A.Y. 2017-2018

41. Lipid lowering agents which act on PPAR α Answer: D


in nucleus
A. atorvastatin #MustKnowConcept
B. ezetimibe Quick Review of Lipid Lowering Agents
C. nicotinic acid
Class Examples MOA Side Effects
D. fenofibrate
Statins Atorvastatin Inhibit HMG- Myopathy,
SImvastatin CoA reductase hepatic
dysfunction

Fibrates Gemfibrozil PPAR-α Myopathy,


Fenofibrate agonists hepatic
dysfunction,
cholestasis

Bile acid- Colestipol Prevents Constipation,


binding reabsorption of bloating
resins bile acids from
the GIT

Sterol Ezetimibe Reduces Hepatic


absorption intestinal dysfunction,
inhibitor uptake of myositis
cholesterol by
inhibiting sterol
transporter
NPC1L1

Niacin Decreases GI irritation,


VLDL flushing,
synthesis and hepatic
LDL toxicity,
cholesterol, hyperuricemi
increases HDL a, may reduce
cholesterol glucose
tolerance

Abraham, M. | Aquino, J. | Arada, M. | Aw, A. | Bagon, N. | Contreras, S. | Galapon, C.


Gison, R. | Obciana, D. | Palao, K. | Tangcueco, P. | Tumambing, M.
58
PHARMACOLOGY RATIONALE
A.Y. 2017-2018

42. This selective B1 agonist plays a role in Answer: B


heart failure by increasing cardiac output and
decreasing ventricular filling pressure #MustKnowConcept
A. dopamine Quick Facts on Dobutamine
B. dobutamine ● Dobutamine does NOT stimulate dopamine receptors!
C. nitroglycerine ● It is a β1 and β2 agonist with variable effects on α
D. nesiritide receptors.
● At low doses, it causes vasodilation, resulting in
decreased aortic impedance and systemic vascular
resistance with reduction in afterload and indirect
increases in cardiac output
● At high doses, it causes vasoconstriction, with
decreased venous capacitance and increased right
atrial pressure

[ dobutamine - low dose vasodilate - high dose vasoconstrict ]

43. A 60 y/o male is on maintenance digitalis Answer: A


for his heart failure. Which of the following will
increase the risk of digitalis toxicity? #MustKnowConcept
A. hypokalemia Electrolyte abnormalities that promote digoxin toxicity:
B. hyponatremia ● Hypokalemia
B. hypocalcemia ● Hypomagnesemia
D. hypermagnesemia ● Hypercalcemia

Remember!
HyPOkalemia facilitates digoxin toxicity
HyPERkalemia reduces digoxin toxicity

[ hyPOkalemia - digoxin toxicity ]

44. Which of the following diuretics reduces Answer: D


mortality in patients with heart failure by
preventing ventricular remodeling? #MustKnowConcept
A. furosemide Drugs can prevent ventricular remodeling by preventing the
B. mannitol abnormal proliferation of connective tissue (emphasized by Doc
C. hydrochlorothiazide Olonan)
D. spironolactone

45. This agent, which inhibits the 1st committed Answer: C


step in sterol synthesis, has marked first pass
hepatic extraction Statins inhibit HMG-CoA reductase, which prevent the formation
A. ezetimibe of mevalonate, the rate limiting step of sterol synthesis. Statins also
B. niacin increase HDL-C in part by preventing the prenylation of Rho A
C. simvastatin and phosphorylation of PPAR-α, a factor that regulated apo A-I
D. gemfibrozil transcription.

Abraham, M. | Aquino, J. | Arada, M. | Aw, A. | Bagon, N. | Contreras, S. | Galapon, C.


Gison, R. | Obciana, D. | Palao, K. | Tangcueco, P. | Tumambing, M.
59
PHARMACOLOGY RATIONALE
A.Y. 2017-2018

46. This agent inhibits the intracellular lipase Answer: D


of adipose tissue through receptor mediated
signaling leading to reduced VLDL production #MustKnowConcept
A. lovastatin Niacin (nicotinic acid) is one of the oldest drugs for
B. fish oil dyslipidemia. It is the only lipid-lowering drug that reduces
C. ezetimibe Lp(a) levels significantly. It inhibits adipocyte adenylyl cyclase,
D. nicotinic acid triglyceride lipolysis, and release of free fatty acids. It reduces
VLDL and LDL, and increases HDL.

Side effects of niacin:


● Hepatotoxicity most common medically serious side
effect
● Flushing and pruritus alleviated by aspirin; aggravated
by coffee, tea, alcoholic drinks
● Contraindicated in PUD
● Reductions in LDL-C ≥50% may indicate
niacin toxicity

47. Abby, a 10 y/o known asthmatic on low Answer: C


dose inhaled steroid (ICS) presents to the ER
with acute moderate to severe exacerbation.
The initial reliever for him that is most cost Salbutamol (SABA) MDI initial reliever is the most cost effective
effective is initial reliever
A. salbutamol nebulization
B. salbutamol + ipratropium nebulization
C. salbutamol metered dose inhaler
D. formoterol + budesonide metered dose
inhaler

48. After 3 doses of bronchodilator of choice Answer: B


20 minutes apart, Abby improved with better
air exchange and no more retractions, but she Prednisone has anti-inflammatory effects due to inhibition of
still has mild wheezing. Peak flow improved pro-inflammaotry cytokines and chemokines. It potentiates β2
form 40% to 75%. Which reliever with anti- agonist effect by increasing the synthesis of β2 receptors. When
inflammatory action is best given for faster used regularly, it can decrease airway hyperreactivity, decrease
resolution of asthma attack upon discharge frequency of asthma exacerbations and improve lung function.
from the ER?
A. theophylline Theophylline is a phosphodiesterase inhibitor with a narrow
B. prednisone therapeutic index.
C. montelukast
D. ICS at double the maintenance dose Montelukast is a leukotriene antagonist that inhibits binding of
LTD4 to cys-LT1 receptors on target tissues. Its anti
inflammatory actions are less marked than inhaled steroids.

Abraham, M. | Aquino, J. | Arada, M. | Aw, A. | Bagon, N. | Contreras, S. | Galapon, C.


Gison, R. | Obciana, D. | Palao, K. | Tangcueco, P. | Tumambing, M.
60
PHARMACOLOGY RATIONALE
A.Y. 2017-2018

49. On further follow up after 1 week, Abby‟s Answer: B


symptoms are well controlled. Which of the
following is most cost effective for Most cost effective for maintenance therapy in this patient
maintenance therapy considering she was would be FDC of low dose ICS + LABA
previously on low dose ICS?
A. double the current dose of ICS
B. low dose ICS + LABA in fixed dose
combination
C. theophylline sustained release preparation
D. montelukast

50. Which of the following can restore Answer: B


responsiveness to B2 agonist?
A. ipratropium bromide Corticosteroids potentiate β2 agonist effect by increasing the
B. glucocorticoid synthesis of β2 receptors.
C. double dose of salbutamol
D. theophylline

51. A 4 y/o has persistent mild asthma and mild allergic Answer: D
rhinitis. Which of the following drugs can be given singly
for the control of both conditions? Montelukast:
A. non-sedating H1 blocker -Leukotriene Antagonist
B. ICS by oral inhalat -inhibits binding of LTD4 to cys-LT1 receptors
C. Nasal steroid -Indications: Alternative controller medications for mild
D. Montelukast asthma and allergic rhinitis

52. Which of the following anti-asthma inhalational drug


administration will require proper hand-mouth Answer: C
coordination for excellent deposition of inhaled drug to
the distal airways? Breather out. Press canister once. Breathe in slowly
A. Nebulization through mouth. Hold breath for 10 seconds to allow
B. Dry powder inhaler medication to reach the airways.
C. Metered Dose Inhaler

53. Inhibition of this isoform of phosphodiesterase leads Answer: C


to airway smooth muscle relaxation?
A. PDE1 Methylxanthines
B. PDE2 Inhibition of PDE3: inc cAMP
C. PDE3 inhibition of PDE4: reduces release of
D. PDE4 cytokines and chemokines→ dec in immune cell
migration and activation

Abraham, M. | Aquino, J. | Arada, M. | Aw, A. | Bagon, N. | Contreras, S. | Galapon, C.


Gison, R. | Obciana, D. | Palao, K. | Tangcueco, P. | Tumambing, M.
61
PHARMACOLOGY RATIONALE
A.Y. 2017-2018

54. Which of the following drugs will inhibit CYP450 Answer: C


leading to increased theophylline level?
A. Beta lactams Enzyme inducers: PCRABS
B. Phenobarbital Phenobarbital, Carbamazepine, Rifampin,Alcohol (chronic
C. Macrolides use), Barbiturates, St. John Wort
D. Aminoglycosides
Enzyme inhibitors: GPACMAN
Grapefruit, Protease inhibitors, Azole antifungals,
Cimetidine, Macrolides, Amiodarone, Non-DHP CCBs
(Diltiazem, Verapamil)

55. Some methylxanthines may block these receptors Answer: D


that mediate bronchoconstriction:
A. LTD4 Methylxanthines:
B. B2 -inhibits PDE → smooth muscle relaxation
C. H1 -competitive antagonists at adenosine receptor
D. Adenosine -activate histone deacetylase in the nucleus
-low level theophylline → inhibits cytokine
and chemokine release

56. Both theophylline and corticosteroids act on the Answer: D


following molecular pathway in inhibiting inflammation:
A. Stimulation of adenylyl cyclase Methylxanthines: bronchodilator and anti-inflammatory
B. Binding to glucocorticoid receptors Corticosteroids: anti-inflammatory; potentiates beta-2
C. Inducing transfer of glucocorticoid receptor agonist effect
complex to the nucleus Activation of histone deacetylase in the
D. Histone deacetylation nucleus → DEC transcription of pro-
inflammatory genes

57. A major adverse effect with Magnesium sulfate Answer: B


infusion in severe status asthmaticus that should be
anticipated is: Magnesium sulfate: bronchodilator
A. Tingling sensation over infusion site In smooth muscles: DEC intracellular Ca by blocking its
B. Respiratory depression entry and release from SR and by activating Na-Ca
C. Convulsion pumps
D. headache In cholinergic nerve terminals: DEC ACh release

Abraham, M. | Aquino, J. | Arada, M. | Aw, A. | Bagon, N. | Contreras, S. | Galapon, C.


Gison, R. | Obciana, D. | Palao, K. | Tangcueco, P. | Tumambing, M.
62
PHARMACOLOGY RATIONALE
A.Y. 2017-2018

58. This fixed dose formulation can be given once daily Answer: A
to patients with COPD or emphysema for sustained
bronchodilation: Salbutamol + ipratropium bromide: relief of asthma
A. Salbutamol + ipratropium attacks and COPD; sustained bronchodilation
B. Motelukast + levocetirizine
C. Formoterol + budesonide LABA + ICS: for asthma control
D. Tiotropium + indacaterol
ICS prevents downregulation of beta-2 agonist receptor

59 - 62 59-62
A. Decreased myocardial oxygen demand Drugs that DECREASE CO:
B. Increased myocardial oxygen supply Diuretics
C. Both A and B Beta blockers
D. Decreased infarct size
DRUGS THAT DECREASE TPR:
Diuretics
Alpha-blockers
Adrenergic blockers
Ca channel blockers
ACE inhibitors
Angiotensin-II receptor blockers

CCBs:
DHP: high vascular selectivity more on peripheral and
coronary vasodilation
59. Beta blocker Non-DHP: greater inhibitory effect on myocardial cells

3 beta-blockers with anti-oxidant effects:


Nebivolol, Celiprolol, Carvedilol

Answer: A

60. Nitroglycerin Answer: C

61. Non-dihydropyridine calcium channel blocker Answer: A

62. Dihydropyridine calcium channel blocker Answer: B

Abraham, M. | Aquino, J. | Arada, M. | Aw, A. | Bagon, N. | Contreras, S. | Galapon, C.


Gison, R. | Obciana, D. | Palao, K. | Tangcueco, P. | Tumambing, M.
63
PHARMACOLOGY RATIONALE
A.Y. 2017-2018

63 - 65 Major Adverse Effects


A. Gemfibrozil + cholestyramine Bile acid sequestrants (Colesevelam, Cholestyramine,
B. Simvastatin + fenofibrate Colestipol): GI, DEC absorption of other drugs, INC liver
C. Niacin + Simvastatin enzymes
D. Ezetimibe + Fish oil Nicotinic acid: Flushing, Pruritus, Headache,
Hyperpigmentation, GI upset, INC in liver enzymes, Uric
acid, Serum glucose
Statins: Myositis and rhabdomyolysis (esp with
gemfibrozil or cyclosporine), INC liver enzymes, Mild GI
upset
Gemfibrozil: INC incidence of cholelithiasis, potentiates
effect of warfarin, diarrhea, nausea, skin rash, myositis
(rare)
Probucol: QT prolongation, serious arrhythmias,
eosinophilia, diarrhea, nausea, flatulence
Ezetimibe: GI complaints, myalgias,
myopathy/rhabdomyolysis, INC liver enzymes, headache,
sinus inflammation, arthralgias

63. Increases the risk of cholelithiasis Answer: A

64. Increases the risk of myopathy Answer: B

65. Elevation in aminotransferases Answer: C

Abraham, M. | Aquino, J. | Arada, M. | Aw, A. | Bagon, N. | Contreras, S. | Galapon, C.


Gison, R. | Obciana, D. | Palao, K. | Tangcueco, P. | Tumambing, M.
64
PHARMACOLOGY RATIONALE
A.Y. 2017-2018

66 - 69 CCBs:
A. Acts on imidazoline receptors Dihydropyridines (-dipines): blocks DHP receptors,
B. Decreases cardiac output by decreasing heart peripheral and coronary vasodilation; AE: headache and
rate ankle edema
C. Direct arterial dilation Non-dihydropyridine: therapeutic activity resemble those
D. Blocks dihydropyridine receptors of beta-blockers; AE: LV depression and bradycardia

Beta-blockers: inhibition of sympathetic


effect on the heart → DEC HR and force of
contraction

Clonidine:alpha 2 adrenergic agonist; DEC CO


and TPR → DEC BP; rebound hypertension is
common

Hydralazine: direct arteriolar smooth muscle relaxation


66. Amlodipine due to fall in intracellular Ca concentration

Answer: D

67. Metoprolol Answer: B

68. Clonidine Answer: A

69. Hydralazine Answer: C

70 - 73 Warfarin (anticoagulant-indirect thrombin inhibitor):


A. Warfarin valvular atrial fibrillation; prevention of embolic infarct in
B. Eptifibatide patients with artificial heart valves
C. Enoxaparin
D. Clopidogrel Eptifibatide (antiplatelet GPIIb/IIIa inhibitor)/Abciximab/
Tirofiban: for PCI and acute coronary syndromes

Enoxaparin (low molecular weight heparin): pregnant


patient with metallic prosthetic valve

Clopidogrel (antiplatelet-ADP inhibitor): Unstable angina,


NSTEMI in combi with ASA, STEMI, Recent MI, stroke or
70. A 28 year old male with mitral stenosis and atrial PAD
fibrillation Answer: A

71. 55 year old, post myocardial infarction patient Answer: D

72. A 30 year old pregnant rheumatic heart disease Answer: C


(Rhd) patient with prosthetic mitral valve

73. A 40 year old male, who just had percutaneous Answer: B


intervention (PCI) for a 2-vessel coronary artery disease

Abraham, M. | Aquino, J. | Arada, M. | Aw, A. | Bagon, N. | Contreras, S. | Galapon, C.


Gison, R. | Obciana, D. | Palao, K. | Tangcueco, P. | Tumambing, M.
65
PHARMACOLOGY RATIONALE
A.Y. 2017-2018

74-77 Nitroglycerin/ISDN sublingual: rapid relief of anginal pain


A. Prevent recurrent angina IV ISDN: first 24-48 hrs in patients with AMI, and CHF
B. Immediately terminate the angina attack ISMN: cont‟d beyond 48 hrs, prevention of recurrent
C. Increase blood supply through reperfusion angina
D. Prevent re-occlusion
Fibrinolytic drugs
(streptokinase/urokinase/Aristeplase/tPA/-kinase/-plase):
Create a generalized lytic state when administered IV; for
acute STEMI

Clopidogrel with ASA for NSTEMI

ASA: long term prophylaxis to prevent MI in patients with


chronic stable angina

74. streptokinase Answer: C

75. Sublingual nitroglycerin Answer: B

76. Clopidogrel and aspirin Answer: D

77. Isosorbide mononitrate and metoprolol Answer: A

Abraham, M. | Aquino, J. | Arada, M. | Aw, A. | Bagon, N. | Contreras, S. | Galapon, C.


Gison, R. | Obciana, D. | Palao, K. | Tangcueco, P. | Tumambing, M.
66
PHARMACOLOGY RATIONALE
A.Y. 2017-2018

78-81 Amiodarone: class III antiarrhythmics; AFibm AVN


A. Amiodarone reentry, macroreentry, PVCs, VF/VFib
B. Verapamil
C. Adenosine Verapamil: blocks both activated and inactivated L-type
D. Magnesium sulfate Ca channels; supraventricular arrhythmia, Afib/Aflutter

Adenosine: activation of inward rectifier K


current and inhibition of Ca current →
hyperpolarization and suppression Ca-
dependent action potentials; Use: conversion of
paroxysmal SVT to sinus

Magnesium sulfate: influence Na/K ATPase; Na, K, Ca


channels; Indications: Digitalis-induced arrhythmias if
with HYPOmagnesemia & Torsades de Pointes even if
serum Mg is normal
78. A 30 year old male presenting in the clinic because
of palpitation since 2 months ago and the ECG shows Answer: B
atrial fibrillation with rapid ventricular response

79. A 28 year old male presenting in the ER with Answer: C


palpitation and supraventricular tachycardia (SVT) with
a rate of 150/min and BP of 100/70

80. A 59 year old female with new onset palpitation and Answer: A
a 24 hour Holter monitoring with frequent premature
ventricular contractions

81. A 40 year old male presenting in the ER with loss of Answer: D


consciousness and Torsades de Pointes on the cardiac
monitor

82-84 Answer: D
A. Furosemide
B. Eplerenone
C. Hydrochlorothiazide
D. Acetazolamide
82. 28 year old male, mountain climber, was given
unrecalled diuretic developed paresthesia and loss of
appetite

83. 45 year old male on maintenance unrecalled diuretic Answer: B


developed hyperkalemia

84. 20 year old male on this diuretic for almost 2 weeks Answer: A
developed vertigo and hearing problems

Abraham, M. | Aquino, J. | Arada, M. | Aw, A. | Bagon, N. | Contreras, S. | Galapon, C.


Gison, R. | Obciana, D. | Palao, K. | Tangcueco, P. | Tumambing, M.
67
PHARMACOLOGY RATIONALE
A.Y. 2017-2018

85 - 88 Beta-blockers:
A. Labetalol Non-cardioselective: Propano,Carteo,Nado,Sota,Timo
B. Prazosin Cardioselective: Acebuto, Ateno, Betaxo, Bisopro,
C. Esmolol Metopro, Esmo
D. Carteolol Vasodilatory, non-selective: Labeta, Pindo, Carvedi
Vasodilatory, selective: Celipro, Nebivo

Labetalol:IV prep for hypertensive emergencies


Carvedilol: for HPN and CHF; reduces mortality in CHF
Esmolol: shortest acting beta-blocker; DOC for intra- and
postoperative HPN; „breviblock‟

Aortic dissection: Labetalol and Nitroprusside

Alpha blockers (-sin): arteriolar and smooth muscle


relaxation, CO unchanges, DEC peripheral resistance

85. Non-selective beta receptor antagonist that is Answer: D


excreted largely in the urine necessitating dose
adjustment in patients with reduced renal function

86. Has beta blocking and vasodilating property, useful Answer: A


in the treatment of hypertensive emergency

87. Beta 1 selective blocker that is rapidly metabolized Answer: C


via hydrolysis by RBC esterases; used for management
of intraoperative hypertension

88. Alpha blocker with beneficial effect in men with Answer: B


bladder obstruction symptoms

Abraham, M. | Aquino, J. | Arada, M. | Aw, A. | Bagon, N. | Contreras, S. | Galapon, C.


Gison, R. | Obciana, D. | Palao, K. | Tangcueco, P. | Tumambing, M.
68
PHARMACOLOGY RATIONALE
A.Y. 2017-2018

89 - 92 Risk for Torsades de Pointes: Class IA and III; Moricizine


A. Torsades de Pointes (IC)
B. Re-entry
C. Both Risk for Re-entry: Class IC
D. None
Class IA: Double Quarter Pounder: Disopyramide,
Quinidine, Procainamide
Class IB: with Lettuce, Mayo & Tomato: Lidocaine,
Mexiletine, Tocainide
Class IC: More Fries Please: Flecainide, Moricizine,
Propafenone
Class II: beta blockers
Class II: SAD: Sotalol, Amiodarone, Dofetilide
Class IV: Diltiazem, Verapamil

89. Lidocaine Answer: D

90. Flecainide Answer: C

91. Sotalol Answer: A

92. Amiodarone Answer: D

93-95
A. If item #1 is greater than item #2
B. If item #2 is greater than item #1
C. If item #1 is equal to item #2
93. INC serum potassium Answer: C
1. Triamterene K sparing diuretics:
2. Amiloride 1. Block Na channels: Amiloride and Triamterene
2. MR antagonist: Spironolactone and Eplerenone

94. DEC serum sodium Answer: B


1. Thiazide Furosemide: most effective diuretic; inhibits NaCl
2. Furosemide reabsorption in TAHL thru inhibition of NKCC2
“High ceiling diuretics”: large absorption capacity in the
segment; NOT limited by acidosis

95. Gynecomastia Answer: A


1. Spironolactone Spironolactone stimulate androgen receptors →
2. Eplerenone gynecomastia (M), menstrual irregularities
(F)
Eplerenone has very low affinity for progesterone and
androgen receptor

Abraham, M. | Aquino, J. | Arada, M. | Aw, A. | Bagon, N. | Contreras, S. | Galapon, C.


Gison, R. | Obciana, D. | Palao, K. | Tangcueco, P. | Tumambing, M.
69
PHARMACOLOGY RATIONALE
A.Y. 2017-2018
3RD LONG EXAM

QUESTION ANSWER & RATIONALE

1. In the treatment of ulcerative colitis, the amount Answer: B


of 5- aminosalisylic acid that must locally act on the
diseased colonic mucosa is markedly lessened because 5-ASA is readily absorbed in the small intestine therefore
of: it doesn‟t reach the distal small intestine and colon.
However, bacterial azoreductase enzyme in the terminal
a. Inactivation of the drug by the acidity of the ileum and colon release the active form of 5-ASA.
stomach
b. High absorption in the small intestine MOA: inhibits NF-kb and NK cells
c. Destruction of the active compound in the
alkaline environment of the small intestine
d. Prolonged intestinal transit brought about by the
disease

2 A 20 year old student was diagnosed with Answer: B


bacterial meningitis probably due to Streptococcus
pneumonia. The most appropriate empiric antibiotic A. Penicillin G: for susceptible S. Pneumoniae but
therapy considering its efficacy against this pathogen only penetrates BBB w/inflammation (it is the
and good penetration into the BBB achieving DOC for N. meningitidis meningitis but will not
therapeutic concentration in the CSF? eliminate carrier state) rifampicin,
Ciprofloxacin, to eliminate carrier state
a. Penicillin G B. Ceftriaxone: 3rd generation ceph, crosses BBB
b. Ceftriaxone C. Vancomycin: DOC for MRSA, crosses BBB only
c. Vancomycin during inflammation
d. Cefuroxime D. Cefuroxime: 2nd generation ceph, crosses BBB
in small amounts WONT REACH
THERAPEUTIC LEVELS IN CSF

To treat meningitis, drug must be able to cross even


without inflammation.

Ceftriaxone is the most active oral antibiotic against


MENINGITIS caused by penicillin resistant S. pneumonia.
It can also be used for both the treatment and elimination
of carrier state of meningitis caused by Neisseria
meningitidis.

Abraham, M. | Aquino, J. | Arada, M. | Aw, A. | Bagon, N. | Contreras, S. | Galapon, C.


Gison, R. | Obciana, D. | Palao, K. | Tangcueco, P. | Tumambing, M.
70
PHARMACOLOGY RATIONALE
A.Y. 2017-2018

Answer: D
3. If the patient has liver failure, which should be
avoided? All aminoglycosides are excreted renally
a. Amikacin ALL penicillins are excreted renally except ampicillin and
b. Cefuroxime oxacillin (rb) nafcillin (hb) ANO
c. Ampicillin ALL cephs are excreted renally except cefixime (rb),
d. Chloramphenicol cefoperazone ceftriaxone (rb)

Chloramphenicol and Clindamycin- HB

4. If this patient has a history of anaphylactic reaction Answer: D


to penicillins, which should be given?
a. Ceftriaxone Chloramphenicol is the DOC for meningitis as a
substitute for beta lactams
b. Ampicillin
c. Meropenem
d. Chloramphenicol

5. If this patient has renal insufficiency, which should Answer: A


be given with caution?
a. Penicillin Penicillin is the only one among the choices that is
b. Ampicillin excreted renally therefore it should be given with caution
c. Chloramphenicol in a patient with renal insufficiency
d. Ceftriaxone

All aminoglycosides are excreted renally


ALL penicillins are excreted renally except ampicillin and
oxacillin (rb) nafcillin (hb) ANO
ALL cephs are excreted renally except cefixime (rb),
cefoperazone ceftriaxone (rb)

Chloramphenicol and Clindamycin- HB

Abraham, M. | Aquino, J. | Arada, M. | Aw, A. | Bagon, N. | Contreras, S. | Galapon, C.


Gison, R. | Obciana, D. | Palao, K. | Tangcueco, P. | Tumambing, M.
71
PHARMACOLOGY RATIONALE
A.Y. 2017-2018

6. Which of the following antibiotics would have more Answer: A


rapid rate of bacterial killing when given as a single
large daily dose? Concentration dependent: single large daily dose
a. Gentamicin SAD MFKr - sad muderfuker?
b. Ceftriaxone Streptogramins (dalfopristin quinupristin),
c. Erythromycin aminoglycosides, daptomycin, metronidazole,
d. Aztreonam fluoroquinolones, ketolides (telithromycin)

Time dependent: continuous IV infusion


CLAM TV

Clindamycin, linezolid, All beta lactams, macrolides,


tetracycline, vancomycin

7. The most cost effective manner of administering the Answer: B


following antibiotic is through continuous IV infusion:
a. Amikacin Continuous IV= Time dependent
b. Ampicillin
c. Metronidazole Ampicillin is the only time dependent among the choices
d. Daptomycin

8. This is responsible for the greater bactericidal Answer: C


efficacy of aminoglycosides if given at a less frequent
dosing: PAE has been used to support the lengthening of the
a. Significant cell membrane penetration dosing interval for aminoglycosides since it is assumed
that the PAE would significantly delay bacterial regrowth
b. Significant growth inhibition after the antibiotic concentration falls below the MIC
c. Prolonged post antibiotic effect
d. Extensive tissue binding

9. A 20 year old has been taking an unrecalled Answer: D


antibiotic or his acne. He later developed nausea,
diarrhea and mild abdominal pain. Which of the GI disturbance is a common side effect of almost all
antibiotics.
following antibiotics could have caused his symptoms?
a. Erythromycin
Erythromycin is a prokinetic agent through stimulation of
b. Clindamycin motilin receptors. Most patients who take this complain
c. Co-amoxiclav of increased gastric motility. Its adverse effects are
d. AOTA related to the stimulation of ,
Bile sludge
infantile hypertrophic pyloric stenosis

Clindamycin is the most common cause of


pseudomembranous colitis. Patients with the condition
commonly present with abdominal pain, diarrhea, fever,
and leukocytosis. Fever and leukocytosis due to the
bacteria Clostridium difficile.

Abraham, M. | Aquino, J. | Arada, M. | Aw, A. | Bagon, N. | Contreras, S. | Galapon, C.


Gison, R. | Obciana, D. | Palao, K. | Tangcueco, P. | Tumambing, M.
72
PHARMACOLOGY RATIONALE
A.Y. 2017-2018

10. Which of the following antibiotics arrest the Answer: A


growth and multiplication of the bacteria without
killing them? All antibiotics are bactericidal except protein synthesis
A. linezolid inhibitors which are bacteriostatic. But there are always
B. daptomycin exceptions.
C. vancomycin
D. quinupristin-dalfopristin Bactericidal protein synthesis inhibitors: ACTS

Aminoglycosides due to inhibition of 3 sequential steps


Formation of initiation complex, translocation,
misreading of mrna therefore wrong AA is
incorporated
Chloramphenicol is cidal against SHN (Streptococcus
pneumoniae, Haemophilus influenzae, Neisseria
meningitidis)
Telithromycin (ketolide)
Streptogramin (dalfu-quino) blockade of 2 steps so
synergistic din

11. Which of the following antibiotics that inhibit Answer: C


protein synthesis is bactericidal against susceptible
pathogens? ACTS

A. Tetracycline
B. Macrolides
C. Aminoglycosides
D. lincosamide

12. The bactericidal activity of these antibiotics Answer: D


increases as their serum concentration increases above
the MIC of the infecting pathogen: Concentration dependent: single large daily dose
SAD MFKr - sad muderfuker?
A. Macrolides Streptogramins (dalfopristin quinupristin),
B. Beta lactams aminoglycosides, daptomycin, metronidazole,
C. Tetracyclines fluoroquinolones, ketolides (telithromycin)
D. Fluoroquinolones
Time dependent: continuous IV infusion
CLAM TV
Clindamycin, linezolid, All beta lactams, macrolides,
tetracycline, vancomycin

Abraham, M. | Aquino, J. | Arada, M. | Aw, A. | Bagon, N. | Contreras, S. | Galapon, C.


Gison, R. | Obciana, D. | Palao, K. | Tangcueco, P. | Tumambing, M.
73
PHARMACOLOGY RATIONALE
A.Y. 2017-2018

13. A 25 year old smear (+) was started on quadruple Answer: B


anti-kochs. Which among the first line agents cause a
rapid reduction in bacillary load? Isoniazid reduces bacillary load during the first 2 days by
90%
A. Rifampicin Rifampicin is a sterilizing agent
B. Isoniazid Ethambutol prevents emergence of resistance
C. Ethambutol Pyrazinamide is also a sterilizing agent
D. Pyrazinamide

14. Isoniazid resistance is the results of: Answer: C

A. Point mutations in the rpoB gene A. rifampicin


B. Point mutations in rpsL gene B. Streptomycin
C. Overexpression of ahpC gene C. Isoniazid
D. Overexpression of pncA gene D. pyrazinamide

Isoniazid resistance
inhA: low level resistance (cross resistance with
ethionamide)
katG: high level resistance (can still use
ethionamide)
ahpC: protects from free radicals; encodes
something that sequesters free radicals
kasA:
Rifampicin resistance
rpoB: reduces affinity for drug
Pyrazinamide resistance
Impaired uptake of the drug
pncA: impair conversion of pyrazinamide to its
active form
Ethambutol
emB: encodes arabinosyl transferase
Streptomycin
rpsL: mutation in s12 protein
Rrs: alters binding site in 16s rRNA (ribosomal
binding site)

Abraham, M. | Aquino, J. | Arada, M. | Aw, A. | Bagon, N. | Contreras, S. | Galapon, C.


Gison, R. | Obciana, D. | Palao, K. | Tangcueco, P. | Tumambing, M.
74
PHARMACOLOGY RATIONALE
A.Y. 2017-2018

15. True of the pharmacokinetic property of isoniazid: Answer: B

A. Better absorption if taken with meals HR: affected by foods


B. Metabolism via acetylation is genetically ZE: not affected by food and assoc with
determined HYPERURICEMIA
C. It diffuses in all body fluids and tissues except
CSF HRZ: good CSF penetration w/o inflammation
D. It is highly protein bound E: passes BBB only in inflammation
E.
It is not protein bound kaya feeling ko it can kill caseous
bacilli HAHA.

Filipinos are fast acetylators of isoniazid. No correlation


between acetylation rate and either efficacy or
adverse reactions.

16. A 30 year old patient with latent TB was started in Answer: B


isoniazid. The antibacterial effect of this drug most
likely is mediated by the inhibition of the synthesis of Mycolic acid inhibition: DIE
which of the following molecules?
Delamanid, isoniazid, ethionamide
A. Peptidoglycan
B. Mycolic acid Arabinogalactan- ethambutol
C. Arabinoglycan DNA dependent RNA polymerase- rifampicin
D. DNA dependent RNA polymerase

17. Which subpopulation of Mycobacterium TB is Answer: C


rifampicin not effective?

A. Rapidly growing extracellular organism


B. Slowly growing intracellular organism
C. Rapidly growing intracellular organism
D. Slowly growing extracellular organism

18. Inhibition of DNA dependent RNA polymerase is Answer: C


the MOA of which anti-TB drug?

A. Ethambutol
B. Isoniazid
C. Rifampicin
D. streptomycin

Abraham, M. | Aquino, J. | Arada, M. | Aw, A. | Bagon, N. | Contreras, S. | Galapon, C.


Gison, R. | Obciana, D. | Palao, K. | Tangcueco, P. | Tumambing, M.
75
PHARMACOLOGY RATIONALE
A.Y. 2017-2018

19. A patient developed flu like symptoms 2 weeks Answer: A


after starting quadruple anti-koch’s therapy. CBC
showed: Hgb: 12, hct: 35, wbc: 7,000 with Flu- like symptoms: rifampicin and PAS
predominance of lymphocytes, plt ct: 80,000. Which PLT NV 200,000- 400,000
among the first line drugs is responsible for the
laboratory abnormality? Adverse reaction of rifampicin
1. Discoloration of body fluids
A. Rifampicin 2. Cholestatic jaundice
B. Isoniazid 3. Flu like syndrome- >anemia and
C. Ethambutol thrombocytopenia so must monitor CBCw. plt
D. Pyrazinamide

20. A 45 year old male with known Tb infection and on Answer: D


medication for 2 months, comes to the UST Eye Center
with complaint of decreased visual acuity. If these EYE-thambutol! (OPTIC NEURITIS that is DOSE
symptoms are due to his drug treatment, which drug is RELATED AND DURATION RELATED) It is not to be
most likely the cause of his symptoms? given to children below 6 years old due to red and green
vision problems. Get perimetry study before treatment.
A. Rifampicin
B. Ciprofloxacin Optic neuritis is also an adverse effect in linezolid
C. Isoniazid treatment. (irreversible)
D. Ethambutol

21. True of the PK property of ethambutol: Answer: C

A. It is bacteriostatic at higher doses Ethambutol is bacteriostatic at usual doses (


B. Absorption is delayed by food and antacids 15mg/kg/day) and bactericidal at higher doses (>25
C. Half of its unchanged form is excreted in the mg/kg/day). Absorption is not affected by food. It is
urine excreted unchanged in the urine. It crosses the BBB
D. Distributed to most tissues but does not cross when inflamed.
the BBB
HR: affected by food
ZE: not affected by food

Answer: B
22. A 55 year old male newly diagnosed smear (+) TB
was started on medication. After 3 weeks, he ZE: associated with hyperuricemia. GOUT FLARE!
consulted at UST OPD with severe pain in the big toe. It is metabolized by xanthine oxidase. The one that
Blood tests show very high uric acid level. Which of makes uric acid.. Related to that pathway.. Xanthine
the following drugs most likely caused his symptoms? oxidase is inhibited by allopurinol.

A. Streptomycin
B. Pyrazinamide
C. Isoniazid
D. Rifampicin

Abraham, M. | Aquino, J. | Arada, M. | Aw, A. | Bagon, N. | Contreras, S. | Galapon, C.


Gison, R. | Obciana, D. | Palao, K. | Tangcueco, P. | Tumambing, M.
76
PHARMACOLOGY RATIONALE
A.Y. 2017-2018

23. A 40 year old male on medication for pulmonary Answer: A


TB or the last 2 months comes to the END OPD wit
complaint of hearing problems and ringing in the ears. The drug involved is an aminoglycoside, particularly
The drug most likely involved has this MOA: streptomycin since it is used as an anti TB. adverse
effects of AGs to remember: NON!!
A. Binds to 30s ribosomal subunit
B. Inhibits DNA dependent RNA polymerase Nephrotoxicity, ototoxicity, neuromuscular blockade
C. Inhibits synthesis of arabiniglyvan subunits AGs bind to the 30s ribosomal subunit
D. Binds to DNA gyrase
30s: STAT the rest bind to 50s
Spectinomycin (ketolides),Tigecycline,
Aminoglycosides, Tetracycline

B: rifampicin
C: Ethambutol
D. FQ

24. The anti-leprosy drug, clofazimine, has the Answer: A


following PK profile?
Clofazimine half life is 2 months; discontinuous therapy, 4
A. Longer serum half-life weeks part. It has good oral absorption, mainly excreted
B. Poorly absorbed from the gut in the shit. Highly lipophilic but doesn't cross the BBB.
C. Mainly excreted in the urine
D. Good tissue penetration including the CSF

25. A 40 year old G6PD deficient male with leprosy is Answer: A


at risk of developing hemolysis if this drug is given:
Dapsone is a sulfa drug! Usually sulfa drugs cause the
A. Dapsone hemolysis in g6pd deficient peeps
B. Rifampicin
C. Clofazimine
D. Thalidomide

26. A 70 y/o patient in the ICU with sepsis has growth Answer: D
of Enterobacter sp. In the blood culture. Which of the
following antibiotics has good activity against the said Enterobacter drugs: 4-AC
organism? 1. 4th gen cephalosporins
2. Aztreonam
A. Ceftazidime 3. Carbapenems
B. Vancomycin
C. Ampicillin-sulbactam A: 3rd gen ceph
D. Ciprofloxacin B: only covers gram positive aerobes and anaerobes (only
c. difficile)
C. wala among pens active
D. FQ

Abraham, M. | Aquino, J. | Arada, M. | Aw, A. | Bagon, N. | Contreras, S. | Galapon, C.


Gison, R. | Obciana, D. | Palao, K. | Tangcueco, P. | Tumambing, M.
77
PHARMACOLOGY RATIONALE
A.Y. 2017-2018

27. Which of the following antibiotics has good Answer: C


coverage for pseudomonas aeruginosa?
Among the carbapenems, ertapenem has no activity
A. Ertapenem against PEA (pseudomonas, enterococcus, acinetobacter)
B. Vancomycin Vancomycin gram positive only
C. Cefepime Chloramphenicol wala
D. chloramphenicol
Anti pseudomonads: CAMPFIRE, let’s build a pseudo
campfire cos poor lang us. HAHAHAHAH

Carbapenems not ertapenem


Aminoglycosides
Monobactams
Polymyxins (polymyxin B, colistin)
FQ (cipro and levo)
thIRd gen and 4th gen cephs (ceftazidime, cefepime
Extended spectrum penicillins (piperacillin, ticarcillin)

28. A 13 y/o taking unrecalled antibiotic presents with Answer: A


respiratory paralysis. This patient may have been given
very high doses of: Gentamicin is an aminoglycoside.
Must remember adverse effects: NON
A. Gentamicin
B. Cotrimoxazole Nephrotoxicity, Ototoxicity, Neuromuscular blockade
C. Penicillin G
D. Imipenem Most nephrotoxic: NTG
Most ototoxic: aNAK!
Most vestibulotoxic: SG (cant balance in singapore)

NMB: risk factors: succinlycholine, hypocalcemia


Antidote: calcium gluconate or neostigmine

Neostigmine is an acetylcholinesterase inhibitor to bring


an anesthetized person back to life HAHA.
Pralidoxime is an acetylcholinesterase regenerator used
as an antidote for organophosphate poisoning (aside
from atropine)

NMB is also seen in treatment with polymyxins: 3N‟s!


1. Neurotoxic
2. Nephrotoxic
3. NMB

Abraham, M. | Aquino, J. | Arada, M. | Aw, A. | Bagon, N. | Contreras, S. | Galapon, C.


Gison, R. | Obciana, D. | Palao, K. | Tangcueco, P. | Tumambing, M.
78
PHARMACOLOGY RATIONALE
A.Y. 2017-2018

29. Which of the following antibiotics is inactivated by Answer: C


renal dehydropeptidase resulting in low urinary
concentrations? Imipenem is formulated with cilastatin which prolongs its
half life.
A. Penicillin G
B. Ceftriaxone
C. Imipenem
D. Piperacillin

30. A 24 y/o female is diagnosed to have pelvic Answer: C


inflammatory disease. Which of the following is the
most appropriate treatment if anaerobic infection is Anaerobic coverage:
suspected?
2nd gen cephs (cephamycins TAN FOX only)
A. Cefuroxime 3rd gen cephs (ceftizoxime)
B. Amikacin Carbapenems
C. Clindamycin Vancomycin
D. Aztreonam Tetracycline
Clindamycin
Linezolid
Metronidazole
Fosfomycin

Gram positive anaerobe: Clostridium difficile


Gram negative anaerobe: Bacteroides fragilis

Abraham, M. | Aquino, J. | Arada, M. | Aw, A. | Bagon, N. | Contreras, S. | Galapon, C.


Gison, R. | Obciana, D. | Palao, K. | Tangcueco, P. | Tumambing, M.
79
PHARMACOLOGY RATIONALE
A.Y. 2017-2018

31. The antibiotic is NOT associated with drug Answer: A


interactions as it does not inhibit nor induce CYP 450
enzymes Clarithromycin and Chloramphenicol are inhibitors
Linezolid is a relatively clean drug. No drug interactions.
A. Linezolid
B. Clarithromycin Inhibitors: SICKFACES.COM
C. Chloramphenicol
D. Doxycycline Sodium valproate
Isoniazid
Chloramphenicol
Ketoconazole
Fluconazole
Alcohol
Cimetidine
Erythromycin/Clarithromycin
Sulfonamides
Ciprofloxacin
Omeprazole
Metronidazole

Inducers: SCRAP GP
Sulfonylureas
Carbamazepine
Rifampin
Alcohol
Phenytoin
Griseofulvin
Phenobarbital

32. Which of the following statements about the Answer: D


disposition of beta lactam antibiotics is correct?
A: only 3rd and 4th gen cross the BBB
A. All second gen cephs cross the BBB B: Nafcillin and ceftriaxone are excreted HB
B. Nafcillin and ceftriaxone are both eliminated C: renal tubular secretion is inhibited by probenecid
via renal excretion
C. The renal tubular reabsorption of amoxicillin is
inhibited by probenecid
D. The instability of penicillins in gastric acid can
limit theri oral absorption

Abraham, M. | Aquino, J. | Arada, M. | Aw, A. | Bagon, N. | Contreras, S. | Galapon, C.


Gison, R. | Obciana, D. | Palao, K. | Tangcueco, P. | Tumambing, M.
80
PHARMACOLOGY RATIONALE
A.Y. 2017-2018

33. A 34 year old female with Rheumatic Heart Answer: B


Disease was admitted for treatment of infective
enterococcal endocarditis/ In the past she had severe Enterococcal endocarditis: vancomycin plus gentamicin
anaphylactic reaction to Penicillin G. The best drug to
give is:

A. Aztreonam
B. Vancomycin
C. Meropenem
D. Piperacillin-tazobactam

34. Which of the following drugs has good activity Answer: B


against Bacteroides fragilis and may cause antibiotic-
associated colitis? Clindamycin is the most associated drug with colitis. It
also has anaerobic coverage.
A. Chloramphenicol
B. Clindamycin Anaerobic coverage:
C. Clarithromycin
D. Cotrimoxazole 2nd gen cephs (cephamycins TAN FOX only)
3rd gen cephs (ceftizoxime)
Carbapenems
Vancomycin
Tetracycline
Clindamycin
Linezolid
Metronidazole
Fosfomycin

Gram positive anaerobe: Clostridium difficile


Gram negative anaerobe: Bacteroides fragilis

35. Which of the following statements is correct Answer: A


regarding the clinical use of aminoglycosides?
Aminoglycosides have no anaerobic coverage. They are
A. They can not be used to treat infections given IV due to large size, polycationic charge and lipid
caused by Bacteroides fragilis insolubility. They can cause renal toxicity. In combination
B. They are highly lipophilic and are not absorbed with beta lactams for serious infections? lol
after oral administration
C. They are often used in combination with
cephalosporins in the empiric treatment of life
threatening infections
D. They can cause hepatotoxicity characterized
by elevated liver function tests

Abraham, M. | Aquino, J. | Arada, M. | Aw, A. | Bagon, N. | Contreras, S. | Galapon, C.


Gison, R. | Obciana, D. | Palao, K. | Tangcueco, P. | Tumambing, M.
81
PHARMACOLOGY RATIONALE
A.Y. 2017-2018

36. Which statements is true about sulfonamides? Answer: B

A. Cross-allergenicity occurs between No cross allergenicity cause different class of antibiotics.


sulfonamides and penicillins They inhibit dihydropteroate synthase (SS). Crystalluria
B. Dysfunction of the basal ganglia may occur in occurs at acidic pH, alkalinize the urine (sodium
the newborn if administered in pregnancy bicarbonate to dissolve crystals)
C. They inhibit dihydrofolate reductase
D. Crystalluria occurs at high urinary pH Cotrimoxazole is a highly protein bound drug. It can
displace bilirubin from albumin and cause kernicterus

37. Which is a correct statement about Answer: D


fluoroquinolones?
Antacids decrease the bioavailability of FQs. Cannot give
A. Antacids increase the bioavailability of FQ to children below 18 years old cause it damages the
B. They are the most appropriate drugs for the cartilage cos it chelates magnesium. All FQs are excreted
treatment of uncomplicated UTI in a 7 year old renally except moxifloxacin. Will cause tendon rupture.
girl
C. They are contraindicated in patients with
hepatic dysfunction
D. They can cause rupture of achilles tendon

38. Th intake of this agent is dependent on the acid Answer: B


secretion stimulated by food intake.
A: PPI: take before meals
A. Esomeprazole B: Antacid: taken with meals
B. Aluminum magnesium hydroxide B: H2R blocker: not affected by food
C. Ranitidine
D. Rebamipide

39. Which of the following describes the PD of Answer: C


histamine 2 receptor antagonists?
H2R blockers block only 60-70%.
A. Blocks >90% of total 24-hr acid secretion They inhibits only H2R
B. Competitive inhibitors of both H2 and H1 They maintain >50% acid inhibition for only 10 hours
receptors kaya given sila BID twice a day.
C. Suppress both basal and meal stimulated acid
secretion PPI‟s block 90 % of total 24-hr acid secretion.
D. Maintains >50% acid inhibition for 16 hours

Abraham, M. | Aquino, J. | Arada, M. | Aw, A. | Bagon, N. | Contreras, S. | Galapon, C.


Gison, R. | Obciana, D. | Palao, K. | Tangcueco, P. | Tumambing, M.
82
PHARMACOLOGY RATIONALE
A.Y. 2017-2018

40. Although PPIs have a very short half life, their acid Answer: D
suppression effect lasts for 24 hours because of this
reason:

A. Reabsorption of the active substance from the


renal tubules is very efficient
B. 98% of the parietal cells are affected
C. Cholinergic stimulation is also suppressed
D. Inactivation of the HK atpase is irreversible

41. Sucralfate heals ulcers through the following Answer: D


mechanism:

A. Acid suppression and selective binding to


necrotic tissue
B. Has antimicrobial property and inhibition of
gastion release
C. Increase prostaglandin synthesis and increase
in gastric emptying time
D. Absorbs biles salts and acts as a barrier to
pepsin

42. For a prokinetic agent to effect a good peristaltic Answer: B


movement, one of the following events must happen:

A. Stimulation of motilin receptor proximally and


stimulation of dopaminergic receptor distally
B. Stimulation of motilin receptor proximally and
inhibition of dopaminergic receptor distally
C. Inhibition of motilin receptor proximally and
stimulation of dopaminergic receptor distally
D. Inhibition of motilin receptor proximally and
inhibition of dopaminergic receptor distally

43. Somatostatin as a key regulatory peptide has the Answer: C


following physiologic effects as an anti diarrheal
agent:

A. Reduce pancreatic secretion


B. Slows intestinal motility
C. Both
D. Neither
E.

Abraham, M. | Aquino, J. | Arada, M. | Aw, A. | Bagon, N. | Contreras, S. | Galapon, C.


Gison, R. | Obciana, D. | Palao, K. | Tangcueco, P. | Tumambing, M.
83
PHARMACOLOGY RATIONALE
A.Y. 2017-2018

Match th ff antiemetics with their MOA: C


D
A. Histamine 1 receptor antagonist B
B. Centrally acting D2 antagonist
C. 5HT3 receptor antagonist
D. Muscarinic receptor antagonist

44. Ondansetron
45. Scopolamine
46. Metoclopramide

Match the effect/ influence of food on the bioavail of C


the ff: A
B
A. Decreased
B. Increased Erythromycin and Azithromycin: taken before meals
C. No change Clarithromycin: before or after

47. Clarithromycin 1. Azithro/Erythro: affected by food, excreted HB


48. Azithromycin 2. Clarithro: not affected by food, excreted renally
49. Cefuroxime axetil 3. Erythro and clarithro both inhibit CYP450

Match the primary route of elimination of the ff: A


C
A. Renal B
B. Hepatobiliary
C. Renal biliary
50. Vancomycin RB: TACO HZ
51. Ampicillin Telithromycin
52. Erythromycin Ampicillin
Cefixime
Oxacillin
Isoniazid
Pyrazinamide

HB: alphabetical lol


Cefoperazone
Chloramphenicol
Clindamycin
Doxycycline/Tigecycline
Erythro/Azithro
Linezolid
Metronidazole
Moxifloxacin
NAfcillin
Quino-Dalfu
Rifampicin

Abraham, M. | Aquino, J. | Arada, M. | Aw, A. | Bagon, N. | Contreras, S. | Galapon, C.


Gison, R. | Obciana, D. | Palao, K. | Tangcueco, P. | Tumambing, M.
84
PHARMACOLOGY RATIONALE
A.Y. 2017-2018

Match the laxatives to their specific side effects: C


A
A. Blorating and flatus formation D
B. Colonic atony
C. Melanosis coli Anthraquinones: senna, cascara, aloe-> melanosis coli
D. Vitamin D and E deficiency Mineral oil: coats the bowel mucosa, impaired
absorption.. ADEK deficiency
53. Senna
54. Lactulose
55. Mineral oil

Abraham, M. | Aquino, J. | Arada, M. | Aw, A. | Bagon, N. | Contreras, S. | Galapon, C.


Gison, R. | Obciana, D. | Palao, K. | Tangcueco, P. | Tumambing, M.
85
PHARMACOLOGY RATIONALE
A.Y. 2017-2018
3RD SHIFTING EXAM

QUESTION ANSWER & RATIONALE

1. A patient diagnosed with malaria complains of tinnitus Answer: C


and visual disturbance. Which of the following drugs
must have been given? Quinine is associated with cinchonism which manifests
A. Chloroquine as tinnitus and visual disturbances (if to treat malaria tas
B. Mefloquine may tinnitus, Quinine agad)
C. Quinine
D. Artemether

2. A male patient was diagnosed with severe malaria. Answer: B


Which of the following must not be given because of its
slow onset? Mefloquine is a SLOW-ACTING schizonticide as
A. Quinine compared to Chloroquine and Quinine/Quinidine which
B. Mefloquine are rapid acting; note that all 4 are active schizonticides
C. Chloroquine for all 4 species of malaria
D. Quinidine

3. A 38 year old male presents with headache and Answer: D


seizures. He was found to have neurocysticercosis. He
should be given: Albendazole is the DOC for cysticercosis or pork
A. Ivermectin tapeworm (T. solium) infection and Echinococcus
B. Pyrantel Pamoate (hydatid disease) if Tapeworm (Cestode), usual DOC din
C. Mebendazole siya for Roundworms except for Enterobius and
D. Albendazole Strongyloides
Ivermectin is indicated for infections with Strongyloides
(threadworm) and Onchocerca
Pyrantel Pamoate is used for Enterobius (pinworm)

Mebendazole is similar to Albendazole but in terms of


being the DOC, Albendazole is more often indicated, so
go with Albendazole

4. A 50 year old female is brought to the ER because of Answer: B


fever, urticaria, swollen and tender lymph nodes,
tachycardia and hypotension (Mazzotti reaction). A week Note the infection treated, since Schistosoma ang agent
ago, she was treated for schistosomiasis. She could have alam mo na flukes, and if flukes ALMOST ALWAYS
been given: Praziquantel and DOC.
A. Ivermectin Mazzotti reaction is only (+) in Diethylcarbamazine,
B. Praziquantel Ivermectin and Praziquantel
C. Albendazole
D. Pyrantel Pamoate

Abraham, M. | Aquino, J. | Arada, M. | Aw, A. | Bagon, N. | Contreras, S. | Galapon, C.


Gison, R. | Obciana, D. | Palao, K. | Tangcueco, P. | Tumambing, M.
86
PHARMACOLOGY RATIONALE
A.Y. 2017-2018

5. Which of the following drugs does NOT require Answer: C


phosphorylation for activation?
A. Acyclovir Acyclovir, Ganciclovir and Cidofovir share similarities in
B. Ganciclovir that they all need kinases to activate their mechanism
C. Nevirapine (if same kinase ang nagact for different antivirals most
D. Cidofovir probably may cross resistance)
Nevirapine is a NNRTI which don‟t require
phosphorylation to become activated (pero kapag NRTI
need padin magphosphorylate)

6. Which of the following antiviral agents does NOT Answer: A


cause termination of chain elongation because of the
presence of 3-OH group? The lack of the 3-OH group is necessary for the chain
A. Penciclovir terminating property of anti-herpes virus agents
B. Acyclovir (Aciclovir, Valaciclovir, Famciclovir); Out of anti-herpes,
C. Didanosine Penciclovir does not cause chain termination b/c it lacks a
D. Cidofovir 3-OH group

7. The oral preparation (as palmitate) of this antibiotic Answer: D


has higher bioavailability than its intravenous (as
succinate) preparation: Take note of palmitate, drugs with salt preparations
A. Linezolid from lecture recall are Erythromycin and
B. Ampicillin Chloramphenicol lang.
C. Azithromycin For Erythro (Estolate - best absorption), for Chloram
D. Chloramphenicol (Succinate - IV, Palmitate - oral); notable lang na greater
bioavailability (b/a) ng palmitate prep VS IV preps of
Chloramphenicol

8. What drug should given in an uncomplicated Answer: A


chloroquine-resistant falciparum malaria during the first
trimester of pregnancy? For Malaria in pregnancy:
A. Quinine + Clindamycin Treatment:
B. Chloroquine + Doxycycline 1. Uncomplicated chloroquine-sensitive;
C. Mefloquine + Lumefantrine Chloroquine
D. Quinidine + Tetracycline 2. Uncomplicated chloroquine-resistant: Quinine
(oral) + Clindamycin
3. Uncomplicated falciparum (2nd and 3rd trim):
ACT (Arthemter Based Combination Therapy)
4. Uncomplicated P. vivax and ovale: Weekly
Chloroquine prophylaxis until delivery and
breastfeeding, then if G6PD levels are normal
switch to Primaquine
5. Severe malaria (1st trim) IV Quinine + Clinda
6. Severe malaria (2nd & 3rd trim): IV/IM
Artemether
Prophylaxis:
1. Chloroquine-sensitive: Chloroquine
2. Chloroquine-resistant: Mefloquine
3. Malaria-endemic areas: FDC Sulfadoxine-
Pyrimethamine

Abraham, M. | Aquino, J. | Arada, M. | Aw, A. | Bagon, N. | Contreras, S. | Galapon, C.


Gison, R. | Obciana, D. | Palao, K. | Tangcueco, P. | Tumambing, M.
87
PHARMACOLOGY RATIONALE
A.Y. 2017-2018

9. To achieve radical cure for P. vivax and P. ovale, this Answer: B


regimen should be given:
A. Artemether + Primaquine Since radical cure (elimination of both erythrocytic and
B. Chloroquine + Primaquine exoerythrocytic stages) dapat may Primaquine (since
C. Primaquine + FDC Sulfadoxine Pyrimethamine Primaquine is the only drug active for dormant hepatic
D. Quinidine + Clindamycin stages of P. vivax & ovale)

Although not sure why Chloroquine VS Artemether, pero


sa h/o kasi may table tas ang indicated dun would be
Chloroquine then if normal G6PG + Primaquine

10. A traveler consulted you for drug prophylaxis prior to Answer: A


going to Thailand, known to be HIGHLY endemic for
multi-drug resistant falciparum malaria. The most Kapagprophylaxis for multi-drug resistant falciparum,
appropriate drug is: always Doxycycline
A. Doxycycline
B. Chloroquine
C. Malarone
D. Primaquine

11. A 55-year old man, who was about to leave for ac Answer: C
vacation trip to Central Africa started a treatment with
mefloquine. Which of the following plasmodial cells can Mefloquine is only active for blood schizonts
be effectively killed by the drug? (erythrocytic stages) of all 4 species of malaria with no
A. P. vivax hypnozoites activity against hypnozoites and gametocytes
B. P. malariae tissue schizonts
C. P. malariae blood schizonts
D. P. falciparum gametocytes

12. Which is a true statement regarding Metronidazole? Answer: C


A. Excreted mainly in the feces
B. Lipophilic and highly protein bound Letter C corresponds to Metronidazole‟s MOA;
C. Acts by producing nitro radical that causes Metro is excreted both in urine and feces and has low
protein and DNA damage plasma protein binding
D. Kills trophozoites and cysts that cause intestinal Only active for trophozoites and not cysts
and extraintestinal infection Effectively eradicates intestinal and extraintestinal tissue
infections but ineffectIve for intraluminal parasites

13. A totally asymptomatic patient consulted because of Answer: A


a finding in the fecalysis of Entamoeba histolytica cysts.
The most appropriate drug is: Diloxanide furoate is the DOC for asymptomatic cyst
A. Diloxanide passers and asymptomatic intestinal infections for
B. Tetracycline children >2 y/o
C. Azithromycin
D. Metronidazole

Abraham, M. | Aquino, J. | Arada, M. | Aw, A. | Bagon, N. | Contreras, S. | Galapon, C.


Gison, R. | Obciana, D. | Palao, K. | Tangcueco, P. | Tumambing, M.
88
PHARMACOLOGY RATIONALE
A.Y. 2017-2018

14. Which drug selectively inhibits viral DNA synthesis? Answer: B


A. Zanamivir
B. Acyclovir Zanamivir (anti-influenza) acts by inhibiting
C. Abacavir neuraminidase leading to clumping of newly released
D. ZIdovudine influenza virus (same with Oseltamivir)

Abacavir (NRTI) and Zidovudine (NNRTI) act more via


DNA termination rather than inhibiting synthesis by
being incorporated to a growing chain leading to
termination as well as by inhibiting HIV-1 reverse
transcriptase

15. A 20 y/o female consulted because of pain at the Answer: A


upper R part of the abdomen accompanied by fever.
Physical exam findings and UTZ of the liver are For hepatic abscess, ameboma and other extraintestinal
suggestive of hepatic abscess probably amoebic in disease the DOC is Metronidazole/Tinidazole + Luminal
nature. The most appropriate first line treatment regime agent
is:
A. Metronidazole + Luminal agent
B. Metronidazole + Tetracycline
C. Dehydroemetine + Luminal agent
D. Chloroquine + Luminal agent

16. Which of the ff is used in the treatment of CMV Answer: D


retinitis and acyclovir resistant HSV and VZV in
immunocompromised patients? Foscarnet is indicated for acyclovir resistant HSV and
A. Trifluridine VZV in immunocompromised individuals
B. Rimantadine Rimantadine and Amantadine are used for influenza and
C. Amantadine Trifluridine is more indicated for HSV for primary
D. Foscarnet keratoconjunctivitis and recurrent epithelial keratitis

17. A 23 y/o medical student, known asthmatic Answer: B


consulted b/c of 1 day history of fever, malaise, nasal
congestion and anorexia suggestive of influenza. Which Oseltamivir is the DOC for Influenza (previously
of the ff drugs should be administered with extreme Amantadine/Rimantadine but not anymore due to the
caution? high rates of resistant strains)
A. Amantadine
B. Oseltamivir Amantadine and Rimantadine also act by inhibiting the
C. Rimantadine viral uncoating process while Oseltamivir and Zanamivir
D. Zanamivir are neuraminidase inhibitors

Zanamivir is given by inhalation and is not recommended


for patients with underlying airway disease

Abraham, M. | Aquino, J. | Arada, M. | Aw, A. | Bagon, N. | Contreras, S. | Galapon, C.


Gison, R. | Obciana, D. | Palao, K. | Tangcueco, P. | Tumambing, M.
89
PHARMACOLOGY RATIONALE
A.Y. 2017-2018

18. An HIV (+) pregnant patient is about to deliver her Answer: A


first baby. What antiretroviral should be administered to
prevent vertical transmission? Antiviral agents that prevent and/or reduce vertical
A. Nevirapine transmission include:
B. Tenofovir Zidovudine (reduce rates; 1st line for pregnant patients)
C. Efavirenz Nevirapine (prevent vertical transmission at the onset of
D. Cidofovir labor)
*Note that Zidovudine and Nevirapine prevent the
transmission of HIV
Lamivudine (prevents vertical transmission of HBV when
given in that last 4 weeks of gestation)

19. Other than Tenofovir, which NRTI is co-administered Answer: B


as pre-exposure prophylaxis to reduce HIV-acquisition
among sexually active heterosexual male and females? Tenofovir + Emtricitabine is given for pre-exposure
A. Abacavir prophylaxis to reduce HIV acquisition in homosexuals,
B. Emtricitabine heterosexuals and IV drug users
C. Lamivudine
D. Stavudine

20. A 28 y/o HIV (+) patient who is being treated with Answer: A
multiple drugs, develops breast hypertrophy, central
adiposity, and insulin resistance. If these changes are NRTIs can produce a potentially fatal syndrome of lactic
related to his drug treatment, this drug belongs to which acidosis, severe hepatomegaly with hepatic necrosis
group of antiretroviral drugs?
A. NRTIs *Not entirely sure why NRTIs ang sagot
B. NNRTIs
C. Entry Inhibitors
D. Protease Inhibitors

21. A 35 y/o HIV (+) male patient presents with anorexia, Answer: C
nausea and vomiting, abdominal pain and epigastric
tenderness suggestive of acute pancreatitis. Which of Didanosine is associated with dose dependent
the following antiretroviral drugs has the the patient pancreatitis (especially when given with Stavudine,
most likely been taking? Zalcitabine, Ribavirin and Hydroxyurea - avoid)
A. Saquinavir
B. Emtricitabine Efavirenz - neural tube defects and CNS toxicity
C. Didanosine Saquinavir (and Ritonavir) - prolongation of QT interval
D. Efavirenz Emtricitabine - hyperpigmentation; *with propylene
glycol - avoid in young children and women)

Abraham, M. | Aquino, J. | Arada, M. | Aw, A. | Bagon, N. | Contreras, S. | Galapon, C.


Gison, R. | Obciana, D. | Palao, K. | Tangcueco, P. | Tumambing, M.
90
PHARMACOLOGY RATIONALE
A.Y. 2017-2018

22. A 35 y/o woman is diagnosed with chronic hepatitis Answer: B


B infection and therapy is initiated. Just after a few hours
she comes to the ER complaining of fever, chills and Note treatment for HepB
muscle aches. Which one of the ff drugs most likely
cause this symptoms? All four are indicated for HepB infection however
A. Lamivudine interferons are the ones which presents with a flu-like
B. Adefovir syndrome (fever, chills, and muscle aches)
C. Entecavir
D. Interferon alfa

23. First pass metabolism of this drug results in the Answer: B


active from penciclovir?
A. Acyclovir
B. Famciclovir
C. Valacyclovir
D. Ganciclovir

24. Monophosphorylation of this antiviral drug is Answer: A


catalyzed by virus specific protein kinase
phosphotransferase in CMV infected cells and by viral
thymidine kinases in HSV-infected cells
A. Ganciclovir
B. Foscarnet
C. Cidofovir
D. Valacyclovir

25. Nephrotoxicity, the major dose-limiting toxicity Answer: C


associated with IV use of this drug can be prevented by
probenecid Cidofovir is excreted by active tubular secretion and
A. Foscarnet must be administered with high dose probenecid to
B. Famciclovir block secretion decreasing nephrotoxicity
C. Cidofovir
D. Ganciclovir Foscarnet - nephrotoxicity and electrolyte imbalances
(excretion via kidney directly proportional to creatinine
clearance hence dose is dependent on CrCl)

Ganciclovir - excretion is linearly related to CrCl;


increased levels when given w/ Probenecid

Famciclovir - testicular toxicity with no changes in sperm


morphology

26. A patient with Hepatitis B and co-infected with HIV Answer: A


is best treated with:
A. Lamivudine Entecavir has weak anti-HIV activity; Zidovudine has no
B. Zidovudine activity against HepB and Telbivudine has no activity
C. Entecavir against HIV
D. Telbivudine

Abraham, M. | Aquino, J. | Arada, M. | Aw, A. | Bagon, N. | Contreras, S. | Galapon, C.


Gison, R. | Obciana, D. | Palao, K. | Tangcueco, P. | Tumambing, M.
91
PHARMACOLOGY RATIONALE
A.Y. 2017-2018

27. Antiretroviral agent not administered in young Answer: BONUS


children, pregnant women, and those with renal and
hepatic insufficiency b/c of its propylene glycol content Emtricitabine and Fosamprenavir/Amprenavir contain
A. Lopinavir propylene glycol and are avoided in children, pregnant
B. Ritonavir women and those with renal and/or hepatic insufficiency
C. Raltegravir
D. Maraviroc

28. Which of the following statements correctly pairs the Answer: D


antifungal drug with its typical adverse effect?
A. Amphotericin B - anemia Amphotericin B - chills and fever (Shake and Bake
B. Griseofulvin - nephrotoxicity syndrome); nephrotoxicity
C. Nystatin - mental confusion Nystatin - extremely toxic for systemic use (given
D. Ketoconazole - menstrual irregularities topically)
Griseofulvin - madami sobra see h/o nalang

29. Which of the following statements best explain the Answer: B


mechanism of antifungal action of Amphotericin B?
A. Inhibition of topoisomerase II Amphotericin B binds to fungal cell membrane ergosterol
B. Impairment of the normal permeability of fungal creating pores leading to increased cell permeability
cell membrane (normal permeability is impaired) and loss of cellular
C. Inhibition of fungal cytochrome P450 enzymes contents
D. Blockade of fungal mitosis

30. Which of the following statements best explains the Answer: D


mechanism of antifungal action of azoles?
A. Inhibition of conversion of squalene to lanosterol Azoles bind and inhibit CYP450 enzyme lanosterol
B. Formation of artificial pores in the fungal demethylase responsible for demethylation of lanosterol
membrane to ergosterol thus leading to decreased ergosterol
C. Inhibition of fungal mitosis synthesis
D. Inhibition of conversion of lanosterol to
ergosterol

31. A 32 y/o female consulted b/c of 4 day history of Answer: C


cheesy vaginal discharge and pruritus. She is currently
taking an oral contraceptive drug. Vaginal smear shows Cheesy vaginal discharge and presence of budding yeast
budding yeast cells with pseudohyphae. Which of the ff cells with pseudohyphae suggests candidiasis
drugs, given topically, would be appropriate for this
patient? If walang Nystatin one can use other topical antifungal
A. Griseofulvin agents. Tolnaftate is ineffective against Candida and like
B. Tolnaftate Naftifine, are only effective for dermatophytes
C. Miconazole
D. Naftifine Miconazole can be given for Candidiasis

Abraham, M. | Aquino, J. | Arada, M. | Aw, A. | Bagon, N. | Contreras, S. | Galapon, C.


Gison, R. | Obciana, D. | Palao, K. | Tangcueco, P. | Tumambing, M.
92
PHARMACOLOGY RATIONALE
A.Y. 2017-2018

32. The most common cause of Clostridium difficile Answer: C


toxin-mediated diarrhea is:
A. Metronidazole Clindamycin is the most common cause of Clostridium
B. Chloramphenicol difficile toxin-mediated diarrhea (Pseudomembranous
C. Clindamycin colitis or toxic megacolon)
D. Erythromycin
Metronidazole - side effects include GI disturbances and
metallic taste and disulfiram-like effect

Chloramphenicol - Gray baby syndrome; dose related


anemia (reversible), aplastic anemia (irreversible)

Erythromycin - Gi disturbances (diarrhea d/t motilin)

33. The most appropriate drug to give for this patient‟s Answer: C
Clostridium difficile colitis is:
A. Daptomycin Metronidazole - DOC for Clostridium difficile colitis
B. Linezolid Vancomycin can also be given but must be given orally
C. Metronidazole
D. Co-trimoxazole Daptomycin - VRSA and enterococci; MRSA (cannot be
given for pneumonia b/c of inactivation by lung
surfactants)

Co-trimoxazole - DOC for Pneumocystis jiroveci


pneumonia; alternative for toxoplasma encephalitis and
may also be used for prostatitis

34. An unimmunized household contact of a patient with Answer: A


diphtheria should receive this antibiotic as prophylaxis:
A. Erythromycin Erythromycin is the DOC for atypical causes of
B. Doxycycline pneumonia (Legionella, Mycoplasma, Chlamydophila) as
C. Rifampicin well as Corynebacterium (diphtheriae) and Bordetella
D. Chloramphenicol pertussis (whooping cough)

Doxycycline - DOC for Rickettsia and Vibrio spp.


(cholera); also used for spirochetes (Leptospira and
Syphilis)

Chloramphenicol - alternative for B-lactams for


treatment of bacterial meningitis

35. A 50 y/o man is suspected of having Mycoplasma Answer: B


pneumonia. The most appropriate antibiotic to give is:
A. Ceftriaxone
B. Azithromycin
C. Co-amoxiclav
D. Amikacin

Abraham, M. | Aquino, J. | Arada, M. | Aw, A. | Bagon, N. | Contreras, S. | Galapon, C.


Gison, R. | Obciana, D. | Palao, K. | Tangcueco, P. | Tumambing, M.
93
PHARMACOLOGY RATIONALE
A.Y. 2017-2018

36. T.L., 20 y/o with leukemia and on her 3rd cycle of Answer: C
chemotherapy is suspected of having febrile
neutropenia. The most appropriate monotherapy for her
is:
A. Aztreonam
B. Ceftriaxone
C. Meropenem
D. Vancomycin

37. A.M. has MRSA bacteremia which of the following is Answer: C


not recommended?
A. Vancomycin Linezolid is not recommended VS Streptogramins which
B. Daptomycin can be used for MRSA bacteremia
C. Linezolid
D. Quinupristin-Dalfopristin Vancomycin - DOC for MRSA (including pneumonia,
bacteremia and endocarditis)

38. Which of the ff anti-MRSA antibiotics act by Answer: B


disrupting the cell membrane function?
A. Linezolid Vancomycin acts by inhibiting cell wall synthesis by
B. Daptomycin binding to the D-ala-D-ala terminus
C. Vancomycin Linezolid and Quinupristin and Dalfopristin are protein
D. Quinupristin-Dalfopristin synthesis inhibitors

AT (30s) CELLS (50s) - protein synthesis inhibitors


Aminoglycosides
Tetracyclines
Chloramphenicol
Erythromycin (Macrolides)
Lincosamides
Linezolid
Streptogramins

39. Which of the ff antibiotics CAN be given EITHER in Answer: B


an empty or full stomach without change in its
bioavailability?
A. Cloxacillin
B. Cephalexin
C. Azithromycin
D. Isoniazid

40. The primary mechanism of resistance of Strep. Answer: B


Pneumoniae to beta lactam antibiotics is:
A. Enzymatic inactivation Beta-lactams act at PBP (penicillin binding protein) to
B. Modification at target site inhibit transpeptidation/cross linking of peptidoglycan
C. Decreased beta lactam uptake units. Resistance occurs when PBP of the bacteria
D. Increased beta lactam efflux evolve/transform to PBP2a wherein B-lactams can no
longer bind

Abraham, M. | Aquino, J. | Arada, M. | Aw, A. | Bagon, N. | Contreras, S. | Galapon, C.


Gison, R. | Obciana, D. | Palao, K. | Tangcueco, P. | Tumambing, M.
94
PHARMACOLOGY RATIONALE
A.Y. 2017-2018

41. A 20 y/o diabetic patient is suspected to have gram(- Answer: A


) anaerobic infection probably due to Bacteroides fragilis.
Which of the following has good activity against the 2nd Gen Cephs which have anaerobic activity include:
organism? Cefoxitin, Cefotetan, Cefmetazole
A. Cefoxitin Cefazolin is a 1st Gen Ceph which has no coverage for
B. Aztreonam anaerobic microorganisms
C. Cefazolin Aztreonam only has activity for Gram(-) bacteria and
D. Gentamicin does not cover for Gram (+) and anaerobes
Aminoglycosides generally have no activity against
anaerobes

42. In which of the ff infections is Doxycycline NOT Answer: A


clinically useful?
A. Gonococcal urethritis
B. Leptospirosis
C. Cholera
D. Chlamydial Cervicitis

43. A 20 y/o with prosthetic heart valve is diagnosed Answer: C


with endocarditis secondary to coagulase negative
staphylococci. The most appropriate antibiotic to give is: Vancomycin is the DOC for coagulase negative
A. Co-amoxiclav staphylococci seen in prosthetic device infections, CSF
B. Co-trimoxazole shunt and catheter related infections
C. Vancomycin
D. Ceftriaxone Ceftriaxone - DOC for meningitis (Penicillin-resistant S.
pneumoniae), N. gonorrhoeae and MDR Salmonella typhi

44. Hypersensitivity, jaundice, GI intolerance and Answer: B


thrombocytopenia are side effects of which first line anti-
TB agent?
A. Isoniazid
B. Rifampicin
C. Pyrazinamide
D. Ethambutol

45. The anti-leprosy drug used in the prevention or Answer: B


treatment of Pneumocystis jiroveci pneumonia in AIDS
patients is? Dapsone shares the mechanism of the DOC for PCP (Co-
A. Clofazimine trimoxazole) in that they both inhibit dihydropteroate
B. Dapsone synthase leading to inhibition of folate synthesis
C. Thalidomide
D. Ethionamide

Abraham, M. | Aquino, J. | Arada, M. | Aw, A. | Bagon, N. | Contreras, S. | Galapon, C.


Gison, R. | Obciana, D. | Palao, K. | Tangcueco, P. | Tumambing, M.
95
PHARMACOLOGY RATIONALE
A.Y. 2017-2018

46. A 30 y/o woman of childbearing potential, currently Answer: B


on contraceptive pills was discovered to have sputum (+)
PTB. You are about to start anti-TB treatment but wary Rifampicin is an enzyme inducer hence it catalyzes the
of possible drug interaction. Which of the anti-TB drugs metabolism of other drugs which are metabolized by the
is most likely to interact with the oral medication and CYP450 system. Here, contraceptive pills might be
pose a risk of decreased protective efficacy against metabolized much more rapidly leading to decreased
pregnancy? protective efficacy
A. Isoniazid
B. Rifampicin In contrast, Isoniazid is an enzyme inhibitor, with
C. Ethambutol opposite effects as to Rifampicin which might predispose
D. Pyrazinamide an individual to prolonged therapeutic effect of drugs or
toxicity

47. True statement regarding Pyrazinamide: Answer: D


A. Can cause vitamin B6 deficiency as an adverse
effect Isoniazid is the one associated with Vitamin B6
B. Acts by inhibiting an essential component in the deficiency leading to peripheral neuropathy
bacterial cell wall Rifampicin acts by inhibiting DNA-dependent RNA
C. Can cause a flu-like syndrome as an adverse polymerase; causes flu-like syndromes (only one among
effect 1st line anti-TB)
D. Distributed throughout the body fluids including Ethambutol inhibits arabinosyl transferase involved in
CSF arabinoglycan (component of mycobacterial cell wall)

48. True of the pharmacokinetic profile of Dapsone: Answer: D


A. Variable absorption
B. Widely distributed in body fluids and tissues
C. Eliminated thru feces
D. Very short serum half-life

49. A 24 y/o consulted b/c of chest x-ray findings of Answer: D


PTB. He is asymptomatic with normal physical
examination. Sputum AFB smear is negative. TST shows See table on anti-TB drug regimen sa h/o
10 mm induration. The most appropriate anti-TB
regimen to start is:
A. Isoniazid
B. Rifampicin
C. Isoniazid, Rifampicin and Pyrazinamide
D. 4 drug fixed dose combination

50. Antacids as acid neutralizers have local effects on Answer: B


the following gastric secretions:
A. Gastrin Antacids only act locally in the stomach by neutralization
B. Hydrogen ion (so gastric HCl lang walang effect on gastrin)
C. Both
D. Neither

Abraham, M. | Aquino, J. | Arada, M. | Aw, A. | Bagon, N. | Contreras, S. | Galapon, C.


Gison, R. | Obciana, D. | Palao, K. | Tangcueco, P. | Tumambing, M.
96
PHARMACOLOGY RATIONALE
A.Y. 2017-2018

51. One of the following drugs crosses the blood brain Answer: D
barrier thus causing CNS manifestations:
A. Rabeprazole GIT drugs that cross BBB:
B. Domperidone Diphenoxylate (anti-diarrheal: opioid agonist)
C. Colloidal bismuth Metoclopramide (prokinetic: dopamine receptor
D. Diphenoxylate antagonist)

52. In the eradication of H. pylori, one of these agents Answer: A


has NO role and therefore should not be prescribed in
addition to at least two antibiotics: Standard “triple therapy” (BID for 14 days):
A. Famotidine Proton Pump Inhibitors
B. Lansoprazole Clarithromycin
C. Colloidal bismuth Amoxicillin or Metronidazole

Second-line therapy
Proton Pump Inhibitors
Bismuth subsalicylate
Tetracycline
Metronidazole

53. A pharmacokinetic characteristic common to both Answer: A


omeprazole and ranitidine is:
A. Undergo rapid first pass hepatic metabolism Omeprazole (PPI)
B. Require activation and absorption in the Undergo rapid first pass hepatic metabolism
stomach Require activation and absorption in the stomach
C. Bioavailability is decreased to 50% by food Bioavailability is decreased to 50% by food
D. Drug clearance is reduced in elderly
Ranitidine (H2 Blocker)
Undergo rapid first pass hepatic metabolism
Not affected by food
Drug clearance is reduced in elderly

Abraham, M. | Aquino, J. | Arada, M. | Aw, A. | Bagon, N. | Contreras, S. | Galapon, C.


Gison, R. | Obciana, D. | Palao, K. | Tangcueco, P. | Tumambing, M.
97
PHARMACOLOGY RATIONALE
A.Y. 2017-2018

54. Either omeprazole or ranitidine is a good drug in one Answer: C


of the following clinical findings:
A. Duodenal ulcer with H. pylori infection Omeprazole (PPI)
B. Gastroesophageal reflux disease with GERD with esophagitis
esophagitis NSAID-induced ulcers (despite continued use)
C. Non-ulcer dyspepsia due to NSAID Actively bleeding ulcers
D. Bleeding gastric ulcer H. pylori-associated ulcers
NUD
Stress-related bleeding
Gastrinoma

Ranitidine (H2 blocker)


GERD
NSAID-induced ulcers (with discontinuation)
NUD
Stress-related bleeding

55. Which adverse effect is common to both Answer: C


domperidone and metoclopramide?
A. Dystonia Both of them are DOPAMINE RECEPTOR
B. Drowsiness ANTAGONISTS but ONLY METOCLOPRAMIDE
C. Gynecomastia CROSSES THE BBB ---> CNS manifestations (dystonia,
D. Parkinsonian symptoms drowsiness, parkinsonian symptoms)

Adverse effects common to both:


Prolactinemia, galactorrhea, gynecomastia, impotence,
menstrual disorders

56. Metoclopramide as dopamine 2 receptor Answer: D


antagonists have one of the following effects as a
prokinetic agent: Motilin = Motility
A. Increased small intestinal motility Dopamine = Distention
B. No effect on LES pressure
C. Decreased colonic transit time Metoclopramide = blocks distention (inhibitory effect of
D. Increased esophageal peristaltic amplitude dopamine):
- Increase esophageal peristaltic amplitude
- Increase LES pressure
- Enhance gastric emptying
NOTE: NO EFFECT ON SI AND COLONIC MOTILITY

Abraham, M. | Aquino, J. | Arada, M. | Aw, A. | Bagon, N. | Contreras, S. | Galapon, C.


Gison, R. | Obciana, D. | Palao, K. | Tangcueco, P. | Tumambing, M.
98
PHARMACOLOGY RATIONALE
A.Y. 2017-2018

57. Which of the following laxatives is the best used as a Answer: A


preparation for a GI procedure to be done in 4 hours?
A. Sodium phosphate Consider the stool evacuation time
B. Bisacodyl
C. Lactulose 1 to 3 hours
D. Psyllium Osmotic laxatives except Lactulose
Castor oil

6 to 8 hours
Stimulant laxatives

1 to 3 days
Bulk-forming laxatives
Surfactant laxatives
Lactulose

58. Small amounts of atropine is added to one of these Answer: D


antidiarrheal agents to discourage overdosage thus
preventing opioid dependence: Opioid Agonists
A. Loperamide Diphenoxylate - crosses the BBB ---> CNS effects and
B. Colestipol opioid dependence
C. Octreotide Loperamide - does not cross the BBB
D. Diphenoxylate

59. Opioid agonist as antidiarrheals has the following Answer: A


constipating action:
A. Decreased mass colonic movements
B. Increased gastrocolic reflex
C. Both
D. Neither

60. Clinical use/s of somatostatin in variceal bleeding Answer: D


include/s:
A. Primary prophylaxis Somatostatin -> ACTIVE BLEEDING
B. Secondary prophylaxis Beta Blocker -> PROPHYLAXIS
C. Both
D. Neither

61. A known HIV (+) patient on the maintenance NRTI Answer: B


and protease inhibitor was recently diagnosed with PTB,
active, cavitary. Which of the following anti-TB drugs Rifampicin - Inducer
should be avoided because of its potent CYP450 Isoniazid - inhibitor
enzyme inducing effect:
A. Isoniazid
B. Rifampicin
C. Pyrazinamide
D. ethambutol

Abraham, M. | Aquino, J. | Arada, M. | Aw, A. | Bagon, N. | Contreras, S. | Galapon, C.


Gison, R. | Obciana, D. | Palao, K. | Tangcueco, P. | Tumambing, M.
99
PHARMACOLOGY RATIONALE
A.Y. 2017-2018

62. Which of the following anti-TB drugs act by Answer: C


inhibiting cell wall synthesis?
A. Isoniazid, ethambutol, pyrazinamide
B. Isoniazid, capreomycin and para-aminosalicylic
acid (PAS)
C. Isoniazid, ethambutol, ethionamide
D. Isoniazid, ethambutol, linezolid

63. This anti-TB drug should not be formulated as fixed Answer: D


dose combination with other anti-TB drugs because of
stability problems:
A. Ethambutol
B. Pyrazinamide
C. Isoniazid
D. Rifampicin

64. The half-life of this anti-malarial drug is longer in Answer: C


patients with severe malaria than in healthy individual:
A. Chloroquine Plasma level, protein binding, half-life of quinine
B. Mefloquine depend on severity of malaria (severe = higher: due to
C. Quinine increased alpha-1-acid glycoprotein)
D. Primaquine

65. The reason for no. 64 is the higher level of the Answer: B
following patients with severe malaria:
A. Plasma albumins
B. Alpha-1-acid glycoprotein
C. Presence of active metabolite
D. Slow hepatobiliary elimination

66. This drug‟s plasma half-life varies with the urinary Answer: D
pH being longer in alkaline urine:
A. Ivermectin
B. Albendazole
C. Praziquantel
D. Diethylcarbamazine

67. A 28 year old male was diagnosed with lung Answer: A


abscess most likely caused by anaerobes. The most
appropriate antibiotic for him is: For anaerobic infections:
A. Clindamycin Clindamycin - ABOVE diaphragm (except CNS) -
B. Metronidazole CLINDABOVE
C. Ceftazidime Metronidazole - BELOW diaphragm + CNS -
D. Vancomycin METROLOW

Abraham, M. | Aquino, J. | Arada, M. | Aw, A. | Bagon, N. | Contreras, S. | Galapon, C.


Gison, R. | Obciana, D. | Palao, K. | Tangcueco, P. | Tumambing, M.
100
PHARMACOLOGY RATIONALE
A.Y. 2017-2018

68. In which of the following organisms is ciprofloxacin Answer: D


NOT clinically useful/beneficial?
A. Mycoplasma pneumoniae Fluoroquinolones are NOT recommended for Neisseria
B. Enterobacter sp gonorrheae
C. Pseudomonas spp.
D. Neisseria gonorrheae Only Ciprofloxacin and Levofloxacin are active against
Pseudomonas aeruginosa

69. A pregnant woman on her 4th month of gestation is Answer: D


positive for P. ovale malaria. She is NOT G6PD
deficient, which of the following would you recommend? Clue: P. ovale and Not G6PD deficient
A. Chloroquine followed by primaquine Give: Primaquine but should be avoided during
B. Quinine plus doxycycline pregnancy
C. Artemether plus mefloquine
D. Chloroquine throughout pregnancy then
primaquine after delivery

Match the drug to its mechanism of action Answer:


A. Causes paralysis of the parasite by depolarizing 70. B
neuromuscular synapse causing the release of Mebendazole = inhibit tubule polymerization
acetylcholine and inhibiting acetylcholinesterase (meBENDAzole TUBe)
B. Inhibit microtubule polymerization by binding to 71. A
the parasite‟s beta-tubulin
C. Increase permeability of trematode and cestode 72. D
cell membranes to calcium causing spastic Ivermectin = Intensify GABA (or IG = Ivermectin-GABA)
paralysis 73. C
D. Paralysis of parasite by intensifying GABA- PraziCAntel = CAlcium permeability
mediated transmission of signals in the
parasite‟s peripheral nerves

70. mebendazole
71. Pyrantel pamoate
72. Ivermectin
73. praziquantel

Match the following patients with adverse effect of their Answer:


maintenance anti-TB drugs: 74. A
A. Isoniazid Isoniazid: peripheral neuropathy, hepatitis
B. Rifampicin 75. C
C. Pyrazinamide Pyrazinamide: hyperuricemia, hepatitis
D. Ethambutol 76. B
Rifampicin: discoloration of body fluids
74. A 35 year old antibiotic male was found to have 77. D
elevated liver enzymes and tingling sensation of hands Ethambutol: retrobulbar neuritis
75. A 40 year old female developed jaundice and joint
pains
76. A 19 year old noted to have red-orange discoloration
of urine
77. A 25 year old male failed to discriminate red from
green color of traffic light

Abraham, M. | Aquino, J. | Arada, M. | Aw, A. | Bagon, N. | Contreras, S. | Galapon, C.


Gison, R. | Obciana, D. | Palao, K. | Tangcueco, P. | Tumambing, M.
101
PHARMACOLOGY RATIONALE
A.Y. 2017-2018

Match the mechanism of action with the antibiotic Answer:


selective toxicity: 78. A
A. Selectively toxic 79. B
B. Relatively selectively toxic 80. A

78. Inhibitors of cell wall synthesis


79. Inhibitors of protein synthesis
Cell wall synthesis Selectively cidal
80. Inhibitors of folic acid synthesis
inhibitors toxic

Cell membrane Not cidal


function inhibitors selectively
toxic

Protein synthesis Relatively Static except


inhibitors selectively aminoglycosides,
toxic streptogramins

Direct inhibitor of Selectively cidal


nucleic acid toxic
synthesis

Folic acid synthesis Selectively Individual drug -


inhibitor toxic static
Combination -
cidal

Match the blood brain barrier penetration of the Answer:


following antibiotics in the presence of inflamed 81. C
meninges: 82. D
A. Excellent 83. A
B. Good 84. B
C. Minimal
D. No passage

81. macrolides
82. cephalexin
83. chloramphenicol
84. ceftriaxone

Abraham, M. | Aquino, J. | Arada, M. | Aw, A. | Bagon, N. | Contreras, S. | Galapon, C.


Gison, R. | Obciana, D. | Palao, K. | Tangcueco, P. | Tumambing, M.
102
PHARMACOLOGY RATIONALE
A.Y. 2017-2018

Match the mechanism of acquired resistance to the Answer:


antibiotics given below: 85. A
A. Increased drug efflux 86. B
B. Enzymatic inactivation of the antibiotic 87. C
C. Decreased affinity of receptor for the antibiotic 88. A
D. Increased concentration of metabolite
antagonizing the antibiotic action mechanisms of resistance (most common)
Beta-lactams: enzymatic inactivation
85. tetracycline Aminoglycosides: enzymatic inactivation (transferase)
86. chloramphenicol Chloramphenicol: enzymatic inactivation
87. vancomycin (chloramphenicol acetyltransferase)
88. macrolides Macrolides: Efflux
Vancomycin: reduced affinity (vanA gene)
Tetracycline: efflux
Sulfonamide: overproduce target

Match the following laxatives to its mechanism of action: Answer:


A. Coats the bowel and decrease colonic 89. D
absorption of water 90. C
B. Direct stimulation of the ENS leading to colonic 91. B
fluid and electrolyte secretion
C. Non-absorbable compound causes obligate Know the classification of laxatives
increase in fecal fluid Castor oil - irritant
D. Hydrolyzed in the upper small intestine to a Lactulose - osmotic laxative
colonic irritant that stimulates intestinal motility Senna - stimulant laxative

89. Castor oil


90. lactulose
91. senna

Abraham, M. | Aquino, J. | Arada, M. | Aw, A. | Bagon, N. | Contreras, S. | Galapon, C.


Gison, R. | Obciana, D. | Palao, K. | Tangcueco, P. | Tumambing, M.
103
PHARMACOLOGY RATIONALE
A.Y. 2017-2018

Match the adverse effect with the corresponding Answer:


antibiotic: 92. B
A. Chloramphenicol Levofloxacin - Arthropathy, Tendinitis, Tendon rupture,
B. Levofloxacin QTc prolongation
C. Linezolid
D. Vancomycin QTc prolong
- erythromycin
92.prolongation of QTc interval - 3rd gen fluoroquinolone (Levo, Moxi,Gati, Gemi)
93. Histamine-related flushing
94. Aplastic anemia 93. D
95. Optic neuropathy Vancomycin - Red man syndrome (due to histamine
release), Ototoxicity, Nephrotoxicity

94. A
Chloramphenicol - Dose-related anemia, Gray baby
syndrome, Aplastic anemia

95. C
Linezolid - thrombocytopenia, optic and peripheral
neuropathy, lactic acidosis, serotonin syndrome

Abraham, M. | Aquino, J. | Arada, M. | Aw, A. | Bagon, N. | Contreras, S. | Galapon, C.


Gison, R. | Obciana, D. | Palao, K. | Tangcueco, P. | Tumambing, M.
104
PHARMACOLOGY RATIONALE
A.Y. 2017-2018
4TH LONG EXAM

QUESTION ANSWER & RATIONALE

1. A 45 year old female with breast cancer is on her Answer: B


4th cycle of chemotherapy with anthracycline. Which
of the following is the expected adverse effect? HIGH YIELD! Refer to TOXICITY BEAR (summary of
A. Cirrhosis the most common adverse effects of specific
B. Dilated Cardiomyopathy antineoplastics) after the last number/before 4th SE
C. Retinal Detachment ratio :)
D. Diabetic Nephrotoxicity
Drug referred to in here is Doxorubicin/Mitoxantrone.
Notable S/E include: CARDIAC TOXICITY (Dilated CM,
Arrhythmias, etc), Radiation recall

Antidote for Cardiotoxicity: Dexrazoxane

*Also note the S/E, Adriamycin Flare (streaking at site


of injection)

2. Which of these anthracyclines has a less toxic Answer: C/D


effect to the heart?
A. Doxorubicin Epirubicin ang na-discuss ni Doc sa lecture to be the
B. Daunorubicin ONLY HEART FRIENDLY anthracycline, so it would
C. Epirubicin probably be the SAFEST answer if kailangan pumili
D. Idarubicin between the two.

“EPIc si IDA kasi SAFE ka sa kanya”

*Cumulative dose toxicity of Dauno/Doxo: 450-550


mg/m2

3. Which of the following drugs causes Answer: B


myelosuppression as its dose-limiting toxicity?
A. Bleomycin DLTs:
B. Busulfan 1. Bleomycin - Pulmonary Fibrosis
C. Oxaliplatin 2. Oxaliplatin - Neurotoxicity (vs. Cisplatin -
D. Dacarbazine Nephrotoxicity; vs Carboplatin -
Myelosuppression)
3. Dacarbazine - Nausea and Vomiting (vs.
Procarbazine - Myelosuppression); “Nahihilo
sila inside DACAR”

TOXICITY BEAR

Abraham, M. | Aquino, J. | Arada, M. | Aw, A. | Bagon, N. | Contreras, S. | Galapon, C.


Gison, R. | Obciana, D. | Palao, K. | Tangcueco, P. | Tumambing, M.
105
PHARMACOLOGY RATIONALE
A.Y. 2017-2018

4. A patient with testicular cancer underwent Answer: A


chemotherapy. He had complete response to
chemotherapy. After 5 years, he developed acute 2 Leukemogenic Antineoplastics discussed:
myelogenous leukemia. What leukemogenic 1. Alkylating Agents - after 4-5 YEARS
chemotherapy agent most likely caused this? - Yung A mukhang 4
A. Alkylating agent 2. Topoisomerase II Inhibitors - after 1-3 YEARS
B. Topoisomerase II inhibitor - 1-II-3
C. Methotrexate
D. Pyrimidine antagonist

5. Which pyrimidine analog gives negative feedback Answer: BONUS


and inhibits de novo synthesis of purine by inhibiting
phosphoribosyl synthetase? Recall:
A. Cytarabine - PYRIMIDINE Antagonists: 5-FU, Floxuridine,
B. Hydroxyurea Cytarabine/Ara-C
C. 5-FU - PURINE Antagonists: 6-Mercaptopurine, 6-
D. Fludarabine Thioguanine, Fludarabine

6. A 15 year old teen was diagnosed with Acute Answer: A


Myelogenous Leukemia. The most appropriate anti-
neoplastic to give is: Other notable uses of other drugs:
A. Cytarabine - Fludarabine - CLL; “CaLL the doctor, may FLU”
B. Hydroxyurea - 5-FU - Colorectal cancer; “FU sa pwet”
C. 5-FU - w/ Oxaliplatin or Irinotecan (UmiIRI)
D. Fludarabine - Cytarabine - AML; “C-AMeL

7. A cancer patient was started on immunotherapy. Answer: D


The most common adverse effect that he would
expect is: One of the dose limiting toxicities of immunotherapies is
A. Hypotension the development of autoimmune diseases.
B. Myelosuppression
C. Hypersensitivity reaction Important Antibody Medications emphasized during the
D. Activation of autoimmune diseases lecture:
- Rituximab - Non-Hodgkin‟s Lymphoma
- Brentuximab - CD30 positive lymphoma
- Nivolumab - PD1/PDL1 inhibitors
- Ipilimumab - CTLA-4 inhibitor
Also note:
- Imatinib - Tyrosine Kinase Inhibitor (BCR-ABL);
CML

Abraham, M. | Aquino, J. | Arada, M. | Aw, A. | Bagon, N. | Contreras, S. | Galapon, C.


Gison, R. | Obciana, D. | Palao, K. | Tangcueco, P. | Tumambing, M.
106
PHARMACOLOGY RATIONALE
A.Y. 2017-2018

8. A 50 y/o female with Graves’ disease developed Answer: B


tonsillopharyngitis. On PE, she is tachycardia, with
fever, chills, and diarrhea. What is the best single Top two choices would be Methimazole and PTU, but
drug to give for more rapid control of her symptoms? take note of the keywords: RAPID CONTROL!
A. Methimazole
B. Propylthiouracil Remember:
C. Iodine - PTU has RAPID dose response
D. Prednisone - Methimazole has SLOW dose response

Other important parameters to compare:


PTU Methimazole

Potency LOWER HIGHER

Absorption RAPID VARIABLE but


COMPLETE

Thyroid storm YES


use?

Pregnancy use? YES (strong No (crosses


protein placenta)
binding)

Duration of LONGER so
action and Half- LESS
life FREQUENCY
OF ADMIN

Breast milk LOW LOW


secretion

9. The reason for your answer above is: Answer: B


A. Decreases vascularity of the gland
B. Rapid inhibition of T4 to T3 conversion Drugs that can inhibit T4 to T3 conversion: 3P’s
C. Causes permanent destruction of thyroid - Prednisone
gland - PTU
D. Blocks alpha adrenergic activity - Propranolol

Abraham, M. | Aquino, J. | Arada, M. | Aw, A. | Bagon, N. | Contreras, S. | Galapon, C.


Gison, R. | Obciana, D. | Palao, K. | Tangcueco, P. | Tumambing, M.
107
PHARMACOLOGY RATIONALE
A.Y. 2017-2018

10. A community of 50 healthy people was exposed Answer: C


to radioactive material due to accidental explosion of
a nuclear facility. What drug can be given to them to Indications of Iodides:
prevent adverse effect on their thyroid? 1. Thyroid storm
A. PTU 2. Pre-op for thyroid surgery
B. Lithium 3. Nuclear accidents (prevent entry of nuclear
C. Iodine tablet or solution iodine)
D. Glucocorticoid
PTU: Thyroid storm, Hyperthyroidism in Pregnancy -
1st Trim (SAFEST)

Lithium/Glucocorticoid: Usually adjunct or last resort

11. A 30 y/o seaman underwent radioactive iodine for Answer: C


Graves’ disease 2 weeks ago. His agency repeated his
thyroid function which showed that his free thyroid Remember that the destruction of the thyroid
hormone levels are still significantly elevated. The parenchyma occurs within 6-12 weeks so EFFECT IS
most logical explanation for this is: DELAYED
A. Radioactive iodine increases thyroid binding
globulin
B. Radioactive iodine has a short half-life
requiring a repeat dose
C. Radioactive iodine takes 6-12 weeks to take
effect
D. He should have been pretreated with
antithyroid first to control his T4 and T3

12. A hypothyroid patient was started on Answer: B


levothyroxine at a dose of 1.6 mcg/kg body weight?
Her thyroid function test should be repeated after 5 Remember: STEADY STATE CONCENTRATION IS
weeks because levothyroxine: REACHED ONLY AFTER 4-5 HALF-LIVES!
A. Has high tissue protein binding
B. Has half-life of 7 days In this case, 5 weeks pa yun. So doon pa lang kukunin.
C. Has wide volume of distribution
D. Undergoes enterohepatic recirculation Also note, T4 has LONGER half-life vs T3 (7 days vs 1
day).

13. A patient with low bone mineral density Answer: D


(osteopenia) should receive this treatment:
A. Calcium Activated Vitamin D enhances calcium absorption in
B. Denosumab the gut so it would be the BETTER choice vs A.
C. Teriparatide
D. Calcium and Vitamin D Denosumab and Teriparatide are treatment
choices/alternatives for OSTEOPOROSIS. So pag nag-
develop na yun compared sa A/D na for low bone
mineral density pa lang.

Abraham, M. | Aquino, J. | Arada, M. | Aw, A. | Bagon, N. | Contreras, S. | Galapon, C.


Gison, R. | Obciana, D. | Palao, K. | Tangcueco, P. | Tumambing, M.
108
PHARMACOLOGY RATIONALE
A.Y. 2017-2018

14. A 40 y/o female experienced carpopedal spasm Answer: D


after undergoing thyroid therapy. Her PTH level is
low. Which is the best preparation of Vit D to give, Activated Vit D3 which is Calcitriol - 1,25
together with calcium? dihydroxycholecalciferol!
A. Ergocalciferol
B. 25-OH cholecalciferol Other choices:
C. 7-dehydrocholesterol - 7-dehydrocholesterol - Precursor to
D. 1,25 dihydroxycholecalciferol Cholecalciferol (Vit D3 inactive)
- 25-OH cholecalciferol - Calcidiol (intermediate
converted to Active Vit D3 by 1alpha-
hydroxylase)
- Ergocalciferol - Vit D2 (plant derived)

15. This drug can falsely increase total thyroid Answer: A


hormone levels because it can increase the binding of
thyroid hormone to thyroid binding globulin: Notable Drugs which INCREASE Binding:
A. Estrogen - Methadone
B. Danazol - Clofibrate
C. Glucocorticoids - SERMs (Raloxifene, Tamoxifen)
D. L-asparaginase - Heroin
- 5-FU

“MSC 5H”

Notable Drugs which DECREASE Binding:


- Glucocorticoids
- Androgens
- Danazol
- L-asparaginase
- Aspirin
- Anticonvulsants
- Furosemide
- Nicotinic Acid

Yung wala sa MSC 5H, nagdedecrease na :) Walang


overlapping letters.

For 16-21: A pregnant woman on her 8th week of Answer: D


gestation came in for prenatal check-up. She
disclosed that she took several medications for her Drugs causing Neural Tube Defects: Valproic Acid,
medical problems. Carbamazepine (Remember CNS drugs sila, so
Neurologic ang teratogenic effect)
16. If she took carbamazepine, which of the following
adverse effects on the fetus would you expect? Oligohydramnios - ACEIs (because they cause renal
A. No effect dysgenesis)
B. Hypotonia
C. Oligohydramnios
D. Spina bifida

Abraham, M. | Aquino, J. | Arada, M. | Aw, A. | Bagon, N. | Contreras, S. | Galapon, C.


Gison, R. | Obciana, D. | Palao, K. | Tangcueco, P. | Tumambing, M.
109
PHARMACOLOGY RATIONALE
A.Y. 2017-2018

17. The above drug utilizes this mechanism in Answer: C


producing teratogenicity:
A. Oxidative stress - Oxidative Stress - Thalidomide, Phenytoin,
B. Vascular disruption Valproic Acid
C. Neural crest disruption - Folate Antagonism - Methotrexate,
D. Mediated teratogenicity Aminopterin, Phenytoin, Valproic Acid
- Specific receptor or enzyme mediated
teratogenicity - ACEIs, Warfarin, Valproic Acid
- Neural Crest Disruption - Valproic Acid,
Carbamazepine, Phthalates
- Vascular Disruption - Ergotamine, Cocaine,
Aspirin, Misoprostol
- Endocrine Disruption - DES, OCPs, Bisphenol A,
Phthalates

Notice how their actual MOAs contribute to their


teratogenic effect? Ex. Folate antagonism and
methotrexate. Sundan lang yung pattern.

Also, note na paulit-ulit si Valproic Acid sa first 4


mechanisms

Abraham, M. | Aquino, J. | Arada, M. | Aw, A. | Bagon, N. | Contreras, S. | Galapon, C.


Gison, R. | Obciana, D. | Palao, K. | Tangcueco, P. | Tumambing, M.
110
PHARMACOLOGY RATIONALE
A.Y. 2017-2018

18. Which of the following physiologic changes is Answer: A


expected during pregnancy?
A. Reduced gastrointestinal motility Effect of decreased GET and motility: Decreased Cmax,
B. Increased gastric acid secretion Increased Tmax
C. Decreased glomerular filtration rate
D. Increased permeability of the blood-brain Other changes:
barrier - Increased gastric pH
- Increased total body of water
- Increased fat accumulation
- Decreased plasma albumin concentration
- Altered activity of hepatic enzymes
- Increased renal blood flow

Physiologic Change PK/PD Effect

↓ GET and Motility ↓Cmax, ↑Tmax

↑ Total Body Water ↑ Vd

↑Fat accumulation ↑tissue binding of


LIPOphilic drugs,
prolonged effect

↑Renal blood flow ↑elimination, ↓t1/2

↓Plasma albumin ↑concentration of


concentration free drug, ↑effect

Altered activity of hepatic ↓metabolism, ↑length


enzymes of time in
concentration

19. Which of the following drugs, if given to her this Answer: B


visit, will NOT cause adverse effect on the fetus?
A. Phenytoin Patient is on her FIRST trimester and Clomipramine
B. Clomipramine causes adverse effects on the THIRD trimester.
C. Lithium
D. Warfarin Androgens - Second and Third

Drugs that cause AEs on the First Trimester only:


- Thalidomide
- Antineoplastics
- Lithium
- Metronidazole
- Misoprostol
- Penicillamine
- Carbamazepine

Abraham, M. | Aquino, J. | Arada, M. | Aw, A. | Bagon, N. | Contreras, S. | Galapon, C.


Gison, R. | Obciana, D. | Palao, K. | Tangcueco, P. | Tumambing, M.
111
PHARMACOLOGY RATIONALE
A.Y. 2017-2018

20. History revealed that she is a chronic smoker and Answer: A


continues to smoke until the time of interview. Which
of the following adverse effects on the fetus is Smoking also causes IUGR and Sudden infant death
expected? syndrome.
A. Prematurity
B. Microcephaly Microcephaly - Cocaine
C. Cutis laxa Cutis laxa - Penicillamine
D. Pulmonary hypoplasia Pulmonary hypoplasia - ACEIs

21. She also disclosed that she is a chronic alcohol Answer: B


drinker. This maternal alcohol consumption is directly
related to the severity of fetal alcohol syndrome: 2g/kg/day OR 6oz/kg/day or 8 drinks/day
A. 2 grams/day
B. 2 grams/Kg/day
C. 2 oz/Kg/day
D. Any amount

For 22-25: A nursing mother came to your clinic Answer: D


asking for medications that will increase her milk
production: INCREASE milk prod:
22. You would giver her: - Reserpine
A. Pyridoxine - Clonidine
B. Bromocriptine - Methyldopa
C. Phenylephrine - Metoclopramide
D. Metoclopramide
DECREASE milk prod:
- Nicotine
- Bromocriptine
- Pyridoxine
- MAO Inhibitors

23. Which of the following will increase the transfer Answer: C


of drugs in the breastmilk?
A. Increased maternal plasma protein binding Other factors which INCREASE transfer:
B. Drug undergoes metabolism in the breast - Low drug MW
tissue - Decreased ionization/more lipid soluble drug
C. Increased blood flow in the breast tissue - Decreased metabolism w/in breast tissue
D. Volume of milk consumed

24. Which of the following is absolutely Answer: D


contraindicated to her?
A. Chloramphenicol ABSOLUTELY CONTRAINDICATED: RAI,
B. Phenobarbital Antineoplastics!
C. Methyldopa
D. Radioactive iodine Breastfeeding:
Chloramphenicol causes MYELOSUPPRESSION
Phenobarbital causes SEDATION

Abraham, M. | Aquino, J. | Arada, M. | Aw, A. | Bagon, N. | Contreras, S. | Galapon, C.


Gison, R. | Obciana, D. | Palao, K. | Tangcueco, P. | Tumambing, M.
112
PHARMACOLOGY RATIONALE
A.Y. 2017-2018

25. Reason for the answer in no. 24: Answer: D


A. Sedation
B. Grey-baby syndrome
C. Sluggish moro reflex
D. Suppression of thyroid function

For 26-28: A 30 year old, 34 week pregnant woman is Answer: D


in active labor with regular and frequent uterine
contractions. Ester anesthetics RARELY CROSS PLASMA, Amide
26. Which of the following anesthetics, if given to her, anesthetics DO!
can cause neonatal cardiorespiratory depression and
flaccidity? Depolarizing NMBs DO NOT CROSS, Non-Depolarizing
A. Ester type local anesthetics NMBs DO!
B. Depolarizing neuromuscular blockers
C. Non-depolarizing neuromuscular blockers General anesthetics also cause neonatal anesthesia
D. General Anesthetics - Compare with BZD which cause hypotonia and
hypothermia
- BRBs which cause global depression

27. To lessen her labor pains, you gave single IV dose Answer: C
of meperidine. Which of the following effects on the
neonate is expected?
A. Seizures
B. Hypotension
C. Respiratory depression
D. Beat-to-beat variability

28. Meperidine’s adverse effect on the neonate is due Answer: B


to its:
A. Low protein binding
B. Metabolite, normeperidine
C. Redistribution to peripheral tissues
D. Long elimination half-life

29. The best time for the baby to be delivered to Answer: A


avoid the above adverse effects (answer to #27) is
___ after administration of this drug: Neonatal respiratory depression occurs 2-3 hours
A. Within one hour AFTER administration so you have to deliver the baby
B. 2-3 hours before this happens.
C. At any time

30. Which of the following drug adverse effect Answer: A


pairing is correct?
A. Tetracycline: enamel hypoplasia Tetracyclines - also cause tooth discoloration, maternal
B. Chloramphenicol: arthropathy liver toxicity
C. Levofloxacin: hearing loss Chloramphenicol - Grey baby syndrome
D. Streptomycin: mental retardation Fluoroquinolones (-floxacin) - Arthropathy
Aminoglycosides (Streptomycin, Amikacin, etc) -
Ototoxicity
Sulfonamides - neonatal jaundice/hemolytic anemia

Abraham, M. | Aquino, J. | Arada, M. | Aw, A. | Bagon, N. | Contreras, S. | Galapon, C.


Gison, R. | Obciana, D. | Palao, K. | Tangcueco, P. | Tumambing, M.
113
PHARMACOLOGY RATIONALE
A.Y. 2017-2018

31. Which of the following maternal medications can Answer: C


cause neural tube defect?
A. Phenytoin Refer to #17
B. Pthalates
C. Valproate Warfarin causes:
D. Warfarin 1st Trimester - Hypoplastic nasal bridge,
chondrodysplasia
2nd Trimester - CNS malformation
3rd Trimester - Bleeding

32. A 40 year old female was diagnosed with ovarian Answer: A


cancer. Which of the following agents, which is active
against cells in all stages of the cell cycle, can be As far as the handouts are concerned, Cisplatin is the
given? only drug which “kills all cells in all stages of the cell
A. Cisplatin cycle”
B. Methotrexate
C. Etoposide CELL CYCLE SPECIFIC AGENTS:
D. Ifosfamide S-Phase:
Topoisomerase I Inhibitor
Topoisomerase II Inhibitor
Anti-metabolites
Hydroxyurea

“HATTS”

G2-M Phase:
Vincas
Taxanes
Bleomycin

Abraham, M. | Aquino, J. | Arada, M. | Aw, A. | Bagon, N. | Contreras, S. | Galapon, C.


Gison, R. | Obciana, D. | Palao, K. | Tangcueco, P. | Tumambing, M.
114
PHARMACOLOGY RATIONALE
A.Y. 2017-2018

33. This antineoplastic agent causes accumulation of Answer: D


acrolein in the urine and bladder:
A. Methotrexate REMEMBER TOXICITY BEAR!
B. Carmustine
C. Chlorambucil - Acrolein causes Hemorrhagic Cystitis
D. Cyclophosphamide - Also in Ifosfamide (Bladder Toxcity)
- Antidote: MESNA (given
SIMULTANEOUSLY with the drug)
- Given with vigorous
HYDRATION
- Methotrexate Toxicity: Myelosuppression
- Antidote: Folinic Acid (given 24-26
hrs AFTER drug administration)

Other drugs + antidotes:


- Cisplatin - Nephrotoxicity
- Antidote: AMIFOSTINE
- Given with vigorous
HYDRATION
- Doxorubicin - Cardiotoxicity
- Antidote: Dexrazoxane

34. Acute hypersensitivity reaction occurring within a Answer: B


few minutes of paclitaxel infusion is due to:
A. Production of free radicals
B. its vehicle castor oil and alcohol
Paclitaxel Docetaxel
C. Its active ingredient polyethylene glycol
D. Presence of sulfonamide side chain in its Water insoluble Water soluble
molecule
Uses Camphor-EL Does not use Camphor-
(castor oil and EL
alcohol) →
HYPERSENSITIVITY

A/E: Neutropenia, Cardiac Neutropenia


Conduction N/V
Abnormalities, Peripheral
Neuropathy (stocking
and gloving pattern)

Imagine si Pacman kumakain ng neutrophils


(neutropenia) habang nakagloves and stockings at
broken hearted siya

Doce (12) nalang ang neutrophils niya (neutropenia)

Abraham, M. | Aquino, J. | Arada, M. | Aw, A. | Bagon, N. | Contreras, S. | Galapon, C.


Gison, R. | Obciana, D. | Palao, K. | Tangcueco, P. | Tumambing, M.
115
PHARMACOLOGY RATIONALE
A.Y. 2017-2018

35. This antineoplastic agent is used to treat Answer: C/D


superficial bladder cancer when administered by local
instillation:
A. Dacarbazine
B. Carboplatin
C. Thiotepa
D. Mitomycin C

MATCHING TYPE: Answer: D

For 36-38: *REDUCED plasma protein binding capacity =


Given the following neonatal physiologic properties: INCREASED free drug = INCREASED effect
A. Erratic gastric emptying
B. Reduced plasma protein binding capacity
C. Increased total body water content
D. Reduced glomerular filtration rate

Match the above with the following effects on PKs


and PDs of drugs:

36. longer half-life

37. Increased Vd Answer: C

38. Delay in therapeutic effects Answer: A

For 39-42: Answer: C

Given the following medications taken by the Ebstein‟s Anomaly


pregnant mother:
A. Isotretinoin
B. Phenytoin
C. Lithium
D. Misoprostol

Which of the above could have caused the following


adverse effects on the neonate?

39. Malformation of the tricuspid valve

40. Bilateral facial paralysis Answer: D

Moebius Sequence - affects CN 6, 7

41. Cardiac, ear and clefting defects Answer: A

42. Microcephaly, mental retardation and hypoplastic Answer: B


nails and distal phalanges
Fetal Hydantoin Syndrome

Abraham, M. | Aquino, J. | Arada, M. | Aw, A. | Bagon, N. | Contreras, S. | Galapon, C.


Gison, R. | Obciana, D. | Palao, K. | Tangcueco, P. | Tumambing, M.
116
PHARMACOLOGY RATIONALE
A.Y. 2017-2018

For 43-45: Answer: C

Match the antineoplastic with its MOA: IRInotecan causes DIARRHEA (Iri ka ng iri kasi may
A. Topoisomerase II inhibitor diarrhea ka)
B. Bifunctional alkylator Topotecan causes Myelosuppression
C. Topoisomerase I inhibitor
D. Microtubulin inhibitor

43. Topotecan

44. Etoposide Answer: A

DLT: Myelosuppression

45. Melphalan Answer: B

Tandaan lang ang MONOFUNCTIONAL ALKYLATORS:


PROcarbazine and DACARbazine

The rest ay bifunctional na.

Procarbazine Dacarbazine

Oral (PrOcarbazine) IV

MyelOsuppresion = DLT Nausea and Vomiting =


DLT

“Nahihilo in DACAR”

For 46-48: Answer: B

Match the most appropriate antineoplastic agent for CAMeL = Cytarabine for AML
the following sets of patients:
A. Tamoxifen
B. Cytarabine
C. 5-FU
D. Rituximab

46. 22 year old male with fever and


lymphadenopathies. CBC: hb 65 g/L; hct 0.11; wbc
65, segs-30, lymphos-20, blasts - 50, plt 23

Bone Marrow Biopsy: Acute Myelogenous Leukemia

Abraham, M. | Aquino, J. | Arada, M. | Aw, A. | Bagon, N. | Contreras, S. | Galapon, C.


Gison, R. | Obciana, D. | Palao, K. | Tangcueco, P. | Tumambing, M.
117
PHARMACOLOGY RATIONALE
A.Y. 2017-2018

47. 60 year old female with weight loss and Answer: C


decreased caliber of stools
Colonoscopy: mass partially obstructing the FU sa PWET
descending colon
Histopathology: adenocarcinoma of the colon 5-FU + IRInotecan
5-FU + Oxaliplatin

48. 62 year old female with breast mass, L s/p radical Answer: A
mastectomy with LN dissection
Histopathology: ductal adenocarcinoma of the breast, *Remember: Rituximab is for Non-Hodgkin Lymphoma
ER positive, PR positive

For 49-52: Answer: D

Match the patient with the most likely anti- Raloxifene is an SELECTIVE ESTROGEN RECEPTOR
osteoplastic drug that he/she is taking: MODULATOR (SERM)
A. Teriparatide - AGONIST in Bone
B. Alendronate - ANTAGONIST in breast and uterus
C. Zoledronic acid
D. Raloxifene Effect on estrogen causes hypercoagulable state
*Also note other estrogenic/anti-estrogenic drugs that
49. A 52 year old female with postmenopausal cause this: DES, Tamoxifen
osteoporosis complained of calf tenderness and
venous duplex scan showed blood clot.

50. A 65 year old history of hip fracture and blood Answer: A


exam showed high calcium levels
Teriparatide causes HYPERCALCEMIA, GERD (d/t/
hypocalcemia), OSTEOSARCOMA

51. A 45 year old female with SLE on chronic steroid Answer: B


intake developed atypical or subtrochanteric fractures
on x-ray after 5 years of intake of this drug Subtrochanteric fractures also caused by DENOSUMAB
Bisphosphonates A/E: OSTEONECROSIS OF THE JAW

52. A 60 year old male with fragility fracture Answer: C


developed fever, chills and bone pain after receiving
this drug intravenously Zoledronic acid is the ONLY IV Bisphosphonate.

For 53-55: Answer: A

Match the following mechanism of action with the Thioamides block Tyrosine Peroxidase (TPO) which
antithyroid agent: block:
A. PTU - Oxidation
B. Saturated solution of KI - Organification/Iodination
C. Lithium - Coupling/Conjugation
D. Isothiocyanate

53. Blocks conjugation of diiodothyronine with


another diiodothyronine

Abraham, M. | Aquino, J. | Arada, M. | Aw, A. | Bagon, N. | Contreras, S. | Galapon, C.


Gison, R. | Obciana, D. | Palao, K. | Tangcueco, P. | Tumambing, M.
118
PHARMACOLOGY RATIONALE
A.Y. 2017-2018

54. Blocks iodination of tyrosyl residues due to high Answer: B


iodine load
Iodides: inhibit iodination (via Wolff-Chaikoff Effect)
and decrease vascularity of the thyroid gland for
surgery

55. Inhibits release of preformed thyroid hormone Answer: C/B

Inhibitors of Thyroid hormone release:


- Lithium
- Anionic Inhibitors

56-60: Answer: B

A 48 year old woman experiencing hot flushes was Raloxifene may aggravate the patient‟s symptoms of
noted to have low bone mineral density on DEXA hot flushes.
scan. Her mother suffered hip fracture at the age of
55. She is afraid of suffering the same fate as her Bisphosphonates are indicated to TREAT and
mom. What drug is the most appropriate to give? PREVENT postmenopausal osteoporosis (note that she
Make a prescription good for 2 months is 48 y/o). Consider also that Teriparatide is given
second-line to Bisphosphonates. Kung papipiliin ka
A. Raloxifene without any restrictions (such as S/E, contraindications),
B. Alendronate Bisphosphonates would be a safe choice.
C. Teriparatide
(TESTMANSHIP: Doon sa exam, clue na hindi simple
arithmetic ang computation ng Teriparatide)

Abraham, M. | Aquino, J. | Arada, M. | Aw, A. | Bagon, N. | Contreras, S. | Galapon, C.


Gison, R. | Obciana, D. | Palao, K. | Tangcueco, P. | Tumambing, M.
119
PHARMACOLOGY RATIONALE
A.Y. 2017-2018

Abraham, M. | Aquino, J. | Arada, M. | Aw, A. | Bagon, N. | Contreras, S. | Galapon, C.


Gison, R. | Obciana, D. | Palao, K. | Tangcueco, P. | Tumambing, M.
120
PHARMACOLOGY RATIONALE
A.Y. 2017-2018
4TH SHIFTING EXAM

QUESTION ANSWER & RATIONALE

1. A 20 year old female was diagnosed with Non- Answer: B


Hodgkin’s lymphoma of the Diffuse Large B-cell
Lymphoma type. What monoclonal antibody is Treatment regimens emphasized during class
combined to CHOP (cyclophosphamide, doxorubicin, Non-Hodgkin’s lymphoma - RCHOP (rituximab,
vincristine and prednisone) regimen for this type of cyclophosphamide, doxorubicin, vincristine and
lymphoma? prednisone)
A. Denosumab Hodgkin’s lymphoma - AVBD (anthracycline,
B. Rituximab vinblastine, vincristine, bleomycin, dacarbazine)
C. Nivolumab Insulin secreting pancreatic CA - streptozin
D. Abciximab Brain tumor - nitrosoureas
AML - cytarabine/ara-C
AML blast crisis - hydroxyurea
APL - retinoic acid derivatives, arsenic trioxide
CLL - fludarabine
CML - Imatinib
Colorectal CA - irinotecan, 5-FU
Adrenocortical CA - mitotane
Post-menopausal breast CA ER (+) - anastrozole,
letrozole

2. A patient with ALL underwent induction Answer: B


chemotherapy. He developed constipation for 4 days
and was given laxatives to no relief. The condition Vincristine belly – obstipation and paralytic ileus
progressed to obstipation and paralytic ileus. The due to autonomic neuropathy (given in high doses)
agent that likely caused this is: Has SENSORY (stocking and glove), MOTOR (foot
A. Vinblastine drop) and AUTONOMIC AE
B. Vincristine
C. Vinorelbine
D. Paclitaxel

3. Tamoxifen is useful in the following condition: Answer: C


A. Prostatic cancer
B. Postmenopausal osteoporosis Tamoxifen - cell cycle specific (G1) competitive
C. Early and metastatic breast cancer PARTIAL AGONIST of estrogen indicated for early
D. Endometrial carcinoma and metastatic breast cancer
Raloxifene - postmenopausal osteoporosis
DES, cyproterone, flutamide + GnRH agonist, GnRH
analogues - prostatic cancer

Abraham, M. | Aquino, J. | Arada, M. | Aw, A. | Bagon, N. | Contreras, S. | Galapon, C.


Gison, R. | Obciana, D. | Palao, K. | Tangcueco, P. | Tumambing, M.
121
PHARMACOLOGY RATIONALE
A.Y. 2017-2018

4. A 28 year old male diagnosed with Answer: D


hyperthyroidism still complains of palpitations, finger
tremors and proximal mm weakness. Pt is already on Palpitations, finger tremors and proximal muscle
Methimazole 40 mg per day. What is the next best weakness – SYMPATHETIC effects
additional agent to give? Propranolol – Beta blockers (only used as an
A. Ivabradine adjunct) block peripheral effects (sympathetic) of
B. Diltiazem thyroid hormones
C. Amlodipine
D. Propanolol

5. A 60 year old patient with hypercalcemia due to Answer: B


renal insufficiency was diagnosed to have
osteoporosis on bone densitometry. The most Denosumab does not need any dose adjustment for
suitable treatment for her is: decreased kidney function unlike risedronate which
A. Risedronate should be limited to those with GFR >30-35 mL/min.
B. Denosumab
C. Teriparatide
D. Salmon calcitonin

6. Which of the following antibiotic can be safely Answer: A


given to her?
A. Amoxicillin Antibiotics considered safe in pregnancy:
B. Ciprofloxacin PENICILLINS (Pen G, Pen V, Ampicillin, Amoxicillin,
C. Gentamicin Co-amox)
D. Doxycycline CEPHALOSPORINS
ISONIAZID AND ETHAMBUTOL
Erythromycin

7. Which of these antibiotics should NOT be given to Answer: C


her because of the possibility of fetal cartilage
damage? Ciprofloxacin (fluoroquinolone) - causes arthropathy
A. Cephalexin Cephalexin (cephalosporin) - safe in pregnancy
B. Vancomycin Vancomycin - may be used in staphylococcal sepsis
C. Ciprofloxacin Co-trimoxazole (sulfonamide) - causes neonatal
D. Co-trimoxazole jaundice/hemolytic anemia if given close to delivery

Answer: B
8. Which of these antibiotics, if given to her, can
cause eight nerve toxicity? Vancomycin – limited studies but not mentioned in
A. Vancomycin handout (may cause ototoxicity)
B. Gentamicin Gentamicin – aminoglycoside – eight nerve toxicity
C. Erythromycin (vestibulo and ototoxicity)
D. Co-trimoxazole Erythromycin – safe in pregnancy
Co-trimoxazole – Sulfamethoxazole – part of
sulfonamides class which may cause neonatal
jaundice/hemolytic anemia; Trimethoprim – fetal
abnormality (1st tri), safe (2nd and 3rd tri)

Abraham, M. | Aquino, J. | Arada, M. | Aw, A. | Bagon, N. | Contreras, S. | Galapon, C.


Gison, R. | Obciana, D. | Palao, K. | Tangcueco, P. | Tumambing, M.
122
PHARMACOLOGY RATIONALE
A.Y. 2017-2018

At 38th week of gestation, a 30 year old woman gave birth to a healthy term baby boy with BW=2.7kg.
Three days later, the infant developed signs and symptoms of sepsis. Empiric antibiotic therapy was
started.

9. If you are to give ampicillin, what dose adjustment Answer: B


should be made?
A. Increase the dose Neonates have diminished renal function due to low
B. Increase the dosing interval renal plasma flow, GFR, and tubular function, thus,
C. Decrease the dose and dosing interval you should increase the dosing interval to allow time
D. No dose adjustment for the kidney to eliminate renally cleared drugs.
Renal function improves substantially during after
the 1st week of life.

Beta-lactams - tubular excretion


Aminoglycosides - glomerular excretion

Answer: B
10. The reason for the above dose adjustment is:
A. Deficient drug metabolyzing enzyme Ampicillin/Penicillins are cleared through tubular
activity secretion in the kidneys. Neonates have diminished
B. Diminished renal fx renal function (low RBF, GFR and tubular function).
C. Immaturity of the neonatal liver Specifically, tubular function is immature at birth
D. Expanded ECF volume and adult value is achieved by 1 year old.

11. Which of the following agents is the most likely Answer: B


cause of paresthesias of distal finger and lower
extremities and loss of deep tendon reflexes being Dose-limiting toxicities:
experienced by a male patient on cancer Vincristine - neurotoxicity (paresthesias of distal
chemotherapy? finger and lower extremities with loss of DTRs
A. Cytarabine initially and subsequent profound decrease in motor
B. Vincristine strength particularly dorsiflexion of the foot)
C. Carmustine Cytarabine - myelosuppression (all anti-metabolites
D. Irinotecan will have myelosuppression as the DLT since they
primarily affect rapidly dividing cells)
Carmustine - myelosuppression
Irinotecan - diarrhea (remember: iri ng iri kasi
nagdadiarrhea)

12. A 50 year old bank manager with lung cancer Answer: D


was given antineoplastic agent for which se
developed arrhythmia, conduction abnormalities and Arrhythmias, conduction abnormalities and
myocarditis. The agent that could have caused these myocarditis – CARDIOTOXICITY
problems is most likely: Anthracyclines (Doxorubicin and Daunorubicin) –
A. Dactinomycin dose limiting toxicity is cardiotoxicity
B. Streptomycin Dactinomycin – DLT: myelosuppression (leucopenia
C. Bleomycin and thrombocytopenia)
D. Doxorubicin Streptomycin – DLT: Ototoxicity and
Nephrotoxicity
Bleomycin: DLT – Pulmonary fibrosis

Abraham, M. | Aquino, J. | Arada, M. | Aw, A. | Bagon, N. | Contreras, S. | Galapon, C.


Gison, R. | Obciana, D. | Palao, K. | Tangcueco, P. | Tumambing, M.
123
PHARMACOLOGY RATIONALE
A.Y. 2017-2018

13. A patient with uncontrolled hyperglycemia was Answer: B


managed with insulin therapy to reverse glucose
toxicity. What metabolic process due to insulin will
INSULIN
take place?
A. Intracellular lipase will be activated favoring Hormone of ABUNDANCE so all functions are
ketone body formation as energy source anabolic
B. Both muscle and liver glycogenolysis will be
stimulated LIVER
C. Muscle ribosomal protein synthesis is Increase glycogenesis
stimulated Increase TG synthesis
D. Lipoprotein lipase is inhibited to inhibit Increase VLDL formation
hydrolysis of chylomicrons Decrease glycogenolysis
Decrease ketogenesis
Decrease gluconeogenesis

MUSCLE
Increase protein synthesis (AA transport &
ribosomal protein synthesis)
Increase glycogen synthesis (glucose transport)
Induces glycogen synthase
Inhibits glycogen phosphorylase

ADIPOSE TISSUE
Increase TG storage
Increase TG synthesis
Increase glycerol synthesis
Stimulate lipoprotein lipase (LIPOGENESIS)
Inhibits intracellular lipase (LIPOLYSIS)

14. A DM patient with abnormal renal fx recently Answer: A


prescribed with exogenous subcutaneous insulin
regimen wil have the ff alteration in Exogenous insulin – primarily cleared by kidneys
pharmacokinetics of insulin (with abnormal renal function, EXCRETION is
A. Duration of action will be longer compromised, drug is not cleared, DOA will be
B. Absorption of insulin from subcutaneous longer)
tissue is delayed Endogenous insulin – primarily cleared by liver
C. Time to peak effect is faster (60% liver/40% kidney _
D. Volume of distribution is increased

15. The reason for the pharmacokinetics alteration of Answer: A


exogenous insulin in question number 14 is that
exogenous insulin is:
Endogenous Insulin Exogenous Insulin
A. Metabolized predominantly by the kidneys
B. Metabolized predominantly by the liver 60% Liver 60% Kidney
C. More bound to a carrier protein 40% Kidney 40% Liver
D. Cleared by biliary excretion

Abraham, M. | Aquino, J. | Arada, M. | Aw, A. | Bagon, N. | Contreras, S. | Galapon, C.


Gison, R. | Obciana, D. | Palao, K. | Tangcueco, P. | Tumambing, M.
124
PHARMACOLOGY RATIONALE
A.Y. 2017-2018

16. A 25 year old lean female is found to have insulin Answer: B


resistance. The best drug to give to correct her deficit
has this mechanism of action Insulin resistance – combat with increase in insulin
A. Increase in the release of preformed sensitivity (action NOT on pancreas)
insulin from the pancreas Drugs that increase insulin sensitivity: Metformin
B. Modulation of gene expression of glucose and TZD
transporters A. Sulfonylureas
C. Decrease in gastric emptying time B. Thiazolidinediones
D. Blockade of depredation of complex C. GLP1 agonist and pramlintide (more for
carbohydrates to monosaccarides weight loss yung MOA)
D. Alpha-glucosidase inhibitors
(acarbose/voglibose)

17. This oral anti-diabetic drug decreases the Answer: A


demand for insulin production by the pancreas:
A. SGLT2 inhibitor SGLT2 inhibitor is the only choice that lowers the
B. DPP4 inhibitor glucose level without producing insulin.
C. Sulfonylurea
D. Meglitinide Glucose-lowering actions:
SGLT2 inhibitor - urinary glucose excretion
DPP4 inhibitor - inactivate enzyme that
degrades incretins & other GLP-1 like
molecules → increase GLP-1 and GIP →
increase glucose-mediated insulin
secretion and decrease glucagon
Sulfonylurea - stimulates release of preformed insulin
and reduce serum glucagon (chronic)
Meglitinide - same action as SURs

18. A 70 year old female recently was diagnosed to Answer: B


have type 2 DM. Her creatinine is 2.5mg/dl with GFR
of 29 ml/min. Which drug is safest and most effective Pt with abnormal renal function – avoid renally
A. Empagliflozin excreted drugs
B. Linagliptin Linagliptin – cleared biliary
C. Metformin
D. Glibenclamide

Abraham, M. | Aquino, J. | Arada, M. | Aw, A. | Bagon, N. | Contreras, S. | Galapon, C.


Gison, R. | Obciana, D. | Palao, K. | Tangcueco, P. | Tumambing, M.
125
PHARMACOLOGY RATIONALE
A.Y. 2017-2018

19. A 25 year old G1P0 was advised to start insulin Answer: A


injection due to gestational diabetes. She was
advised to inject glulisine 5 minutes before each Glulisine is a rapid-acting insulin with the following
meal. What is the correct characteristic of this characteristics:
insulin? Onset: 5-15 minutes
A. Peak effect in 30 minutes to 1 hour Peak: 30 minutes to 1 hour
B. Duration of action is approximately 16 hours Duration: 3-4 hours
C. Most effective in acidic pH
D. No change in amino acid sequence B probably refers to detemir (long-acting insulin)
C refers to glargine (long-acting insulin)
ALL insulin analogs have changes in amino acid
sequences

20. Which of the ff is a pharmacokinetic property of Answer: C


insulin defemir?
A. acidic pH in solution A. Insulin glargine
B. Protaminated insulin B. NPH Insulin
C. Has fatty acid side chain C. Insulin detemir – myristic acid is added to B29
D. Less albumin bound lysine
D. Insulin detemir is bound to serum and tissue
albumin

21. The main advantage of DPP4 inhibitors over Answer: C


sulfonylureas is:
A. More potent No hypoglycemia
B. More effective MET
C. Less hypoglycemia TZDs
D. Safe in patients with hepatic impairment GLP-1 agonists
DPP4 inhibitors
SGLT2 inhibitors

Answer: D
22. This uncommon AE of metformin is seen in
patients with renal insufficiency who received Biguanides are contraindicated in patients with
iodinated contrast agent for imaging procedure: renal disease, alcoholism, hepatic disease, or
A. Osmotic diarrhea conditions predisposing to tissue anoxia (eg,
B. Nausea and vomit chronic cardiopulmonary dysfunction) because of
C. Severe B12 deficiency the increased risk of lactic acidosis induced by
D. Lactic acidosis these drugs (Katzung).

Abraham, M. | Aquino, J. | Arada, M. | Aw, A. | Bagon, N. | Contreras, S. | Galapon, C.


Gison, R. | Obciana, D. | Palao, K. | Tangcueco, P. | Tumambing, M.
126
PHARMACOLOGY RATIONALE
A.Y. 2017-2018

23. A patient recently diagnosed with type 2 DM was Answer: D


prescribed with glimepiride. The following
pharmacodynamic cellular action in his pancreas will Sulfonylureas MOA: decrease K efflux by
occur resulting in release of insulin: closing ATP-sensitive K channels →
A. Decrease K efflux, depolarization of beta cell, DEPOLARIZATION → opening of Ca channels →
closure of Ca channel release of preformed insulin
B. Closure of K channel, hyperpolarized beta
cell, opening of Ca channel
C. Opening of K channel, depolarization of beta
cell, opening of Ca channel
D. Decrease K efflux, depolarization of beta cell,
opening of Ca channel

24. A newly diagnosed DM female patient comes to Answer: D


clinic with FBS 170mg/dl. She has BMI of 30 with
estimated GFR of 70 ml/min. She wants to lose A. SUR/Glinides – causes wt gain
weight but does not want to use injectable agents. B. TZD – causes wt gain
The best drug for her has this MOA C. GLP1 agonist – injected SQ
A. Stimulate pancreas to release preformed C. DPP4 Inhibitor – wt neutral (does not cause wt
insulin loss)
B. Increase GLUT4 transporter expression D. SGLT 2 Inhibitors – oral, causes modest wt loss
C. Improve glucose mediated release of of 2-5 kg, can be administered in pts with GFR
insulin from pancreas greater than 45 mL/min/1.73 m2
D. Decrease reabsorption of glucose from
proximal tubules

25. This sulfonylurea has significant active Answer: A


metabolite excreted through the kidneys hence
contraindicated in renal insufficiency: Glibenclamide - significantly active metabolite
A. Glibenclamide Glipizide - inactive metabolites
B. Gliclazide Glimepiride - weakly active metabolites
C. Glipizide
D. Glimepiride

26. What specific precaution should be advised to Answer: D


patients taking SGLT2 inhibitors to avoid adverse
drug reaction? SGLT 2 inhibitors cause osmotic diuresis leading to
A. Take diuretic to avoid pulmonary intravascular volume contraction, dehydration and
congestion hypotension.
B. Take snacks in between meals to avoid
hypoglycaemia A. For TZD + increased plasma volume leading to
C. Take the drug with or after meals to avoid edema/CHF
GI irritation B. For SUR/human insulin + causes hypoglycemia
D. Increase water intake by at least one glass C. For alpha glucosidase inh + causes GI disturbances
per day to avoid dehydration (diarrhea, flatulence, malabsorption)

Abraham, M. | Aquino, J. | Arada, M. | Aw, A. | Bagon, N. | Contreras, S. | Galapon, C.


Gison, R. | Obciana, D. | Palao, K. | Tangcueco, P. | Tumambing, M.
127
PHARMACOLOGY RATIONALE
A.Y. 2017-2018

27. Thiazolidinediones have this pancreatic sparing Answer: C


effect and will decrease the demand for insulin
production because of this action: A - SURs
A. Direct pancreatic stimulation to produce B - Pramlintide or GLP-1 agonists
insulin D - SGLT2 inhibitors
B. Delay gastric emptying and promote satiety
C. Improve GLUT4 expression in muscle and
adipose
D. Decrease glucose reabsorption in the
proximal tubules

Answer: A
28. A 60 year old diabetic female on gliclazide
complained of hypoglycemic episodes in between her Gliclazide – sulfonylurea
meals. You decided to shift her to Sitagliptin because Sitagliptin – DPP4 inh (inc glucose-mediated
of this pharmacodynamics action of the drug. insulin secretion – no glucose, no insulin release, no
A. Glucose dependent stimulation of insulin fasting hypoglycemia)
secretion
B. Decreases threshold for glucose excretion
through the kidneys
C. Stimulates appetite center in the
hypothalamus
D. Inhibits glucagon secretion by the A cells
of pancreas

29. A 26 year old female was recently diagnosed Answer: C


with type 2 DM. She has BMI of 30. You decided to
initiate GLP1 agonist. Which pathophysiologic defect A - SURs, DPP4 inhibitors, GLP1 agonists,
can be corrected EXCLUSIVELY with this drug? pramlintide (so not exclusive)
A. Excessive hepatic gluconeogenesis due to B - SURs, glinides
glucagon C - GLP1 agonists (pramlintide too according to
B. Pancreatic insulin deficiency hand-out but Doc Ho said this is the exclusive
C. Increased appetite due to neurotransmitter function of GLP1 agonists)
dysfunction in the CNS D - MET, TZD
D. Increased insulin resistance in the muscle and
adipose tissue

30. A 35 year old on her 25th wk of gestation was Answer: B


prescribed a medication to stop her contractions. She
accidentally doubled the dose and is now Salbutamol - Tocolytic agent (beta 2 agonist) with
experiencing tachycardia, headache, nervousness SYMPATHETIC SE (tachycardia, headache,
and anxiety. She most likely took: nervousness, anxiety)
A. Isoxuprine
B. Salbutamol
C. Indomethacin
D. Magnesium Sulfate

Abraham, M. | Aquino, J. | Arada, M. | Aw, A. | Bagon, N. | Contreras, S. | Galapon, C.


Gison, R. | Obciana, D. | Palao, K. | Tangcueco, P. | Tumambing, M.
128
PHARMACOLOGY RATIONALE
A.Y. 2017-2018

31. A 30 year old known asthmatic with elevated BP Answer: A


is about to give birth. In order to augment labor this
should be given DOC to augment labor is oxytocin!
A. Oxytocin
B. Misoprostol
C. Terbutaline
D. Dinoprostone

32. This hormone can be given in order to test for Answer: C


hypogonadism
A. Medroxyprogesterone derivative GnRH analogues – approved only for diagnostic use
B. HCG alpha recombinant in hypogonadism (see handout)
C. GnRH analogues
D. Human Menopausal Gonadotrophin

33. This drug can be given to a patient with prostate Answer: B


cancer with elevated liver enzymes, mild
gynecomastia, and increased libido. The most Cyproterone is the only anti-androgen that could be
appropriate anti-androgen is: used to reduce male hypersexuality (libido).
A. Ketoconazole Ketoconazole and flutamide causes gynecomastia
B. Cyproterone acetate while spironolactone has no use for prostatic cancer
C. Flutamide (its only indication is hirsutism).
D. Spironolactone

34. Principal effect of DHT include: Answer: C


A. Erythropoesis
B. LH inhibition
C. Prostate hypertrophy in elderly
D. Spermatogenesis

35. This is a pharmacologic effect of tamoxifen on Answer: A


the following tissue:
A. Agonist in the bone
Tamoxifen Raloxifene
B. Antagonist to prostate gland
C. Agonist to breast cell proliferation Breast Antagonist ---
D. Antagonist to the endometrium
Bone Agonist Agonist

Endometrium Agonist Antagonist

Abraham, M. | Aquino, J. | Arada, M. | Aw, A. | Bagon, N. | Contreras, S. | Galapon, C.


Gison, R. | Obciana, D. | Palao, K. | Tangcueco, P. | Tumambing, M.
129
PHARMACOLOGY RATIONALE
A.Y. 2017-2018

36. This is the most appropriate hormone Answer: D


replacement therapy for a 25 year old who
underwent hysterectomy and bilateral Pt who underwent hysterectomy does NOT need
salphingoophorectomy in order to prevent progesterone. Progestin is added to estrogen in
osteoporosis and improve vasomotor symptoms. HRT to decrease risk of endometrial hyperplasia
A. Mestranol and norgestrel and CA.
B. Norethisterone and ethynodiol
C. Levonorgestrel
D. Ethinyl estradiol

37. The mechanism of action of Bromocriptine as an Answer: B


ovulation-inducing agent:
A. Estrogen agonist
B. Dopamine receptor agonist
C. B-adrenergic agonist
D. Serotonin receptor antagonist

Answer: C
38. This cyclic 9 amino acid peptide synthesized in
the paraventricular nucleus of the hypothalamus, Verbatim in handout
when synthesized in low doses, produces slow
generalized uterine contractions with full relaxation
in between:
A. PGE1
B. Ergonovine
C. Oxytocin
D. Bromocriptine

39. The mechanism by which Atosiban exerts its Answer: C


tocolytic effect is:
A. Competes with calcium at voltage dependent A - CCBs
sites B - Indomethacin
B. Prevents prostaglandin-mediated increases D - B2 agonists
intracellular calcium
C. Oxytocin receptor antagonist
D. B2 receptor smooth muscle membrane
agonist

40. A 37 year old on her 30th week AOG is having Answer: D


preterm labor. She suddenly feels warm with nausea,
vomiting, and dryness of mouth. The most probable
tocolytic given to this patient is SE of Magnesium sulfate: Feeling of warmth,
A. Nifedipine nausea, vomiting, dry mouth, blurred vision and
B. Indomethacin transient hypotension.
C. Salbutamol
D. Magnesium sulphate

Abraham, M. | Aquino, J. | Arada, M. | Aw, A. | Bagon, N. | Contreras, S. | Galapon, C.


Gison, R. | Obciana, D. | Palao, K. | Tangcueco, P. | Tumambing, M.
130
PHARMACOLOGY RATIONALE
A.Y. 2017-2018

41. A 25 year old on OCPs for polycystic ovarian Answer: B


syndrome was diagnosed with PTB disease. Which
of the following medications has a significant drug Hepatic enzyme inducers (Si Rifa nagseizure sa bar
interaction with OCPs? na maraming alcohol)
A. Isoniazid Rifampicin
B. Rifampicin Alcohol
C. Pyrazinamide Barbiturates
D. Ethambutol Antiepileptics

Answer: C
42. This anti estrogen inhibits aromaization of
androgen to estradiol: A. GnRH inhibitor
A. Leuprolide B. SERM
B. Tamoxifen C. Aromatase inhibitor
C. Anastrazole D. Synthetic progestin
D. Medroxy progesterone

43. A 19 year old is on unrecalled medication for Answer: A


severe endometriosis. She noted that she was
gaining weight and had oily skin. The most likely Drugs used for endometriosis
medication taken was: Progesterone (through long-term ovarian
A. Danazol suppression)
B. Mifepristone OCPs
C. Flutamide Danazol
D. Menotropins GnRH agonists
Testosterone

44. The most appropriate treatment for a 60 year old Answer: D


male with BPPH:
A. Cyproterone Finasteride – 5-alpha reductase inhibitor, decreases
B. Flutamide DHT; mainly used for BPH and male pattern
C. Ketoconazole baldness
D. Finasteride

45. This synthetic testosterone derivative has Answer: B


minimal estrogenic, androgenic, and anabolic activity:
A. Hydroxyprogesterone *See last page for rationale
B. Dimethisterone
C. Medroxyprogesterone acetate
D. Desogestrel

46. MOA of Tamoxifen: Answer: A


A. Competitive inhibitor of estrogen at
specific target organ Tamoxifen – agonist at bone and endometrium;
B. Pure estrogen antagonist antagonist at breast
C. GnRH antagonist
D. Inhibits midcycle surge of LH and FSH

Abraham, M. | Aquino, J. | Arada, M. | Aw, A. | Bagon, N. | Contreras, S. | Galapon, C.


Gison, R. | Obciana, D. | Palao, K. | Tangcueco, P. | Tumambing, M.
131
PHARMACOLOGY RATIONALE
A.Y. 2017-2018

47. Indication for testosterone therapy: Answer: C


A. Prostatic cancer
B. Male pattern baldness Testosterone uses:
C. Endometriosis Replacement
D. Female infertility Endometriosis
Fibrocystic disease of the breast
Postmenopausal breast CA
Protein anabolism
Anemia
Osteoporosis
Growth stimulators
Hereditary angioneurotic edema

48. An ideal agent to be given post uterine surgery to Answer: B


prevent blood loss:
A. Prostaglandin E2 Oxytocin – used in control of postpartum
B. Oxytocin haemorrhage
C. Ergonovine I don’t know why ergonovine is not considered for
D. Isoxsuprine answer, I actually tried to correct for it but hindi ko
na nakita yung response, as far as I know, oxytocin
padin yung sagot.
D. Isoxuprine – tocolytic agent; NOT used for
postpartum hemorrhage

49. Death from paracetamol overdose is generally Answer: A


due to its:
A. Nephrotoxicity
B. Hepatotoxicity
C. CNS toxicity
D. Cardiotoxicity

50. Exposure to toxic levels of mercury is ideally Answer: D


managed with this antidote
A. Sodium-calcium EDTA EDTA – lead
B. N-acetylcysteine NAC – paracetamol
C. Dimercaprol (BAL) BAL – not anymore used for Hg poisoning because
D. Succimer (DMSA) it extracts Hg from circulation and deposits it in the
brain causing brain toxicity; for lead (combined
with DMSA)
DMSA - mercury

51. Atropine sulfate is used to counteract the effects Answer: C


of toxicity with:
A. Theophylline Antidotes
B. Carbon monoxide Carbon monoxide - oxygen
C. Insecticide Cyanide - sodium thiosulfate
D. Cyanide

Abraham, M. | Aquino, J. | Arada, M. | Aw, A. | Bagon, N. | Contreras, S. | Galapon, C.


Gison, R. | Obciana, D. | Palao, K. | Tangcueco, P. | Tumambing, M.
132
PHARMACOLOGY RATIONALE
A.Y. 2017-2018

Answer: D
52. An unconscious patient who comes in having
ingested 40 tablets of a benzodiazepine should be Flumazenil is the antidote for benzodiazepine
given toxicity. Desferoxamine is for iron poisoning ,
A. Desferoxamine EDTA for lead, Penicillamine is for copper
B. Penicillamine poisoning as it is usually used in the treatment of
C. EDTA Wilson’s Disease
D. Flumazenil

53. Benzodiazepines given in the anxious elderly Answer: C


should be monitored for development of:
A. Seizures
B. Depression
C. Ataxia
D. Vertigo/dizziness

54. The most consistent risk factor for adverse drug Answer: D
reactions in the elderly is the
A. Dose of drug taken Doctor Quison specifically emphasized that the
B. Protein binding of drugs number of drugs taken is the most consistent risk
C. Type of drug taken factor for adverse drug reactions that is why it is
D. Number of drugs taken imperative that polypharmacy should be avoided.

55. A 52 year old menopausal woman with history of Answer: B


DVT wants an HRT to relieve her symptoms of hot
flushes and sleep disturbance: Estrogen and progesterone are the only drugs
A. Oral contraceptive pills indicated for HRT. But since the patient has history
B. Desogestrel of DVT already, you should not give drugs that
C. Estradiol valeate contain estrogen (OCPs and estradiol valeate) so
D. Danazol your DOC is desogestrel (progesterone).

56. A 45 year old male with low sperm count can be Answer: C
given this drug to stimulate spermatogenesis:
A. Dutasteride Menotropin is used to stimulate spermatogenesis in
B. Clomiphene males with isolated gonadotropin deficiency.
C. Menotropin
D. GnRH analog

57. Which of the following physiologic changes in Answer: C


pregnancy will result in reduced Cmax of drugs: Delayed gastric emptying → decrease Cmax
A. Increased intestinal motility and increase Tmax
B. Decreased Vd Increase total body water → increase Vd →
C. Delayed gastric emptying decrease Cmax
D. Delayed hepatic extraction

Abraham, M. | Aquino, J. | Arada, M. | Aw, A. | Bagon, N. | Contreras, S. | Galapon, C.


Gison, R. | Obciana, D. | Palao, K. | Tangcueco, P. | Tumambing, M.
133
PHARMACOLOGY RATIONALE
A.Y. 2017-2018

58. The best time to breastfeed an infant after taking Answer: B


a relativel safe medication so that the total amount
he would receive would receive in a day is less than The best time is 30-60 mins after breastfeeding OR
what would be considered a therapeutic dose 3-4 HOURS BEFORE BREASTFEEDING to allow
A. 30-60 min before breastfeeding the drug to clear from maternal circulation so that
B. 30-60 min after breastfeeding drug excretion in milk will decrease
C. Anytime

59-62. Match the drug responsible for the following case scenarios of adverse drug reaction:
A. Acarbose
B. Pioglitazone
C. Liraglutide
D. Glibenclamide

59. A 70 year old diabetic female with dilated Answer: B (keyword: dilated cardiomyopathy)
cardiomyopathy developed pulmonary congestion

60. A 35 year old obese diabetic female complained Answer: C


of nausea and vomiting wit episodes of loose stool GLP1 agonist in general has NAUSEA as their MOST
COMMON SIDE EFFECT

61. A 55 year old male complained of abdominal Answer: A (keyword: abdominal bloating and
bloating and flatulence flatulence)

62. A 60 year old male complained of hunger pangs , Answer: D


palpitations, sweating, and tremors This patient is showing signs of hypoglycaemia.
Among the anti DM drugs, only insulin,
sulfonylurea (glibenclamide), and glinides will have
such adverse effect. Pramlintide may also cause
hypoglycaemia when combined with insulin , and
usually it is combined with insulin that is why
premixed dose with insulin should be lowered.

Abraham, M. | Aquino, J. | Arada, M. | Aw, A. | Bagon, N. | Contreras, S. | Galapon, C.


Gison, R. | Obciana, D. | Palao, K. | Tangcueco, P. | Tumambing, M.
134
PHARMACOLOGY RATIONALE
A.Y. 2017-2018

63-66. Match the mechanism for teratogenicity of the following drugs:


A. Aspirin
B. Thalidomide
C. Methotrexate
D. Bisphenol A

63. Folate antagonism Answer: C


MAPV (methotrexate, aminopterin, phenytoin,
valproic acid)

64. Endocrine disruption Answer: D

65. Oxidative stress Answer: B


TPV (thalidomide, phenytoin, valproic acid)

66. Vascular disruption Answer: A

Folate antagonism = methotrexate, aminopterin,


valproic acid, phenytoin (VAMP)
Neurcal Crest disruption = Valproic, Retinoid,
Carbamazepine (VCR)
Endocrine = DES, OCP, Bisphenol, Phthalate
(BPOD)
Oxidative stress = thalidomide, phenytoin, valproic
(PVT)
Vascular disruption = misoprostol, ergotamine,
cocaine, aspirin (CAME)
Specific receptor or enzyme mediated = ACE
inhibitor, valproic, warfarin (VAW)

67-70. Match the mechanism of action of the following drugs:


A. Purine antagonist
B. An alkylating agent that forms cross-link with DNA
C. Pyrimidine antagonist
D. Topoisomerase II inhibitor

67. Mitomycin Answer: B

68. 6 mercaptopurine Answer: A

69. 5-fluorouracil Answer: C

70. Tenoposide Answer: D

Alam nyo nayan hahahahaa, self explanatory

Abraham, M. | Aquino, J. | Arada, M. | Aw, A. | Bagon, N. | Contreras, S. | Galapon, C.


Gison, R. | Obciana, D. | Palao, K. | Tangcueco, P. | Tumambing, M.
135
PHARMACOLOGY RATIONALE
A.Y. 2017-2018

71-75. Match the DOSE LIMITING TOXICITY with the corresponding antineoplastic drug
A. Myelosuppression
B. Pulmonary fibrosis
C. Neurotoxicity
D. Nephrotoxicity

71. Cytarabine Answer: A

72. Cisplatin Answer: D


Do vigorous hydration and alkalinisation to prevent
this. AMIFOSTONE MAY BE USED AS AN ANTIDOTE
TO THIS AS IT CAN SCAVENGE FREE RADICALS.

73. Methotrexate Answer: A

74. Bleomycin Answer: B


May manifest as pneumonitis with cough, dyspnea,
crackles, infiltrates. It manifest after a single dose
of >25mg/msquared or cumulative dose of
>450mg/msquared

75. Oxaliplatin Answer: D

76-79. Match MOA of antiosteoporotic


A. Alendronate B. Denosumab C. Teriparatide D. Raloxifene

76. Induces apoptosis of osteoclast Answer: D

77. Blocks binding of RANK ligand to its receptor Answer: B

78. Stimulates osteoblast formation Answer: C

79. Inhibits mevalonic acid important in osteoclast Answer: A


survival

Abraham, M. | Aquino, J. | Arada, M. | Aw, A. | Bagon, N. | Contreras, S. | Galapon, C.


Gison, R. | Obciana, D. | Palao, K. | Tangcueco, P. | Tumambing, M.
136
PHARMACOLOGY RATIONALE
A.Y. 2017-2018

Match clinical scenario with most appropriate antidote


A. Naloxone B. Deferoxamine C. Physostigmine
D. IV sodium bicarbonate to promote excretion

80. 30 year old male with dilated pupils, tachycardia, Answer: C


mm twitching and urinary retention
This patient is showing symptoms of anti-
cholinergic poisoning (dilated pupil, tachycardia,
urinary retention, etc..) maybe due to ingestion
nightshade devil snare. The antidote for anti
cholinergic poisoning is physostigmine.

81. Acute ingestion of more than 200 mg/kg of this Answer: D


drug causing hyperventilation and respiratory
alkalosis This patient is showing symptoms of salicylate
poisoning characteristically manifested by
hyperventilation due to stimulation of respiratory
center

82. Newborn baby developed respiratory depression Answer: A


due to IV meperidine given to her mother
It is the antidote used for respiratory depression
due to use of opiod analgesics such as pethidine
(Meperidine).

83. A 53 year old child who accidentally ingest 10 Answer: B


tablets of this medication experience vomiting, Iron (keywords: vomiting, abdominal pain, bloody
abdominal pain and bloody diarrhea diarrhea)

TRUE OR FALSE
Write A if the statement is correct and B if the statement is incorrect

84. Overtreatment of levothyroxine in a post total Answer: A


thyroidectomy patient can produce tachycardia, Levothyroxine is a thyroid hormone synthetic
sweating and tremors preparation therefore overdose of it would lead to
symptoms of hyperthyroidism.

85. The most adverse reaction to oral Answer: B


bisphosphonates is deep vein thrombosis DVT is an adverse effect of SERMs

86. Salmon calcitonin can decrease pain dt vertebral Answer: A


fracture
Remember, salmon calcitonin is used for acute pain
relief in patients with vertebral fracture

87. The dose of denosumab needs to be reduced in Answer: B


renal insufficiency patients No dose adjustment needed for denosumab

Abraham, M. | Aquino, J. | Arada, M. | Aw, A. | Bagon, N. | Contreras, S. | Galapon, C.


Gison, R. | Obciana, D. | Palao, K. | Tangcueco, P. | Tumambing, M.
137
PHARMACOLOGY RATIONALE
A.Y. 2017-2018

Regarding neonatal pharmacokinetics

88. Increased percutaneous absorption of drugs may Answer: A


result in toxicity to topically applied rubbing alcohol Halata naman true eto

89. Low concentration of bile acids may decrease the Answer: B


adsorption of water soluble drug Low concentration of bile acids may decrease
absorption of lipid soluble drugs

90. Increased intestinal peristalsis may decrease Answer: A


amount of drug absorbed
If you increase peristalsis then the time the drug
stays in the intestine will decrease because mas
mabilis cya makakadating sa rectum and then sa
anus so mas less ang time for absorption so ergo,
mas onti din maabsorb

91. Increased rate of hepatic metabolism may lead to Answer: B


accumulation of toxic metabolites Decrease rate of hepatic metabolism

Regarding sites of accumulation of drugs

92. Phenytoin – adrenal gland Answer: A

93. Warfarin - teeth Answer: B


Preferential for bone

94. Tetracycline – bones Answer: A

Tetracycline = Teeth and bone, Warfarin = bone,


Aminoglycoside = middle ear, DES= vagina and
mullerian duct, Phenytoin and corticosteroid =
adrenal gland

95-98. Regarding pharmacokinetics/pharmacodynamic of oral anti-hyperglycemic agents:

95. Acarbose should be taken before meals to Answer: B


stimulate release of insulin immediately before food No release of insulin
intake Effect: delays digestion and absorption of starch and
disaccharides

Abraham, M. | Aquino, J. | Arada, M. | Aw, A. | Bagon, N. | Contreras, S. | Galapon, C.


Gison, R. | Obciana, D. | Palao, K. | Tangcueco, P. | Tumambing, M.
138
PHARMACOLOGY RATIONALE
A.Y. 2017-2018

96. Rosiglitazone has better lipid profile changes Answer: B


compared to pioglitazone
Pioglitazone has a triglyceride lowering effect
attributed to the PPAR alpha activity not seen in
Rosiglitazone, but both of them has PPAR gamma
activity

97. DPP4 inhibitors has added benefit of weight loss Answer: B


due to suppression of appetite Weight neutral

98. SGLT2 inhibitors is contraindicated in patients Answer: B


with renal failure due to accumulation of active
metabolites According to Doctora Jean Ho, the consequence of
the decrease in GFR seen in renal failure is that it
limits the efficacy of the drug, AND NOT because it
is attributed to an increase in toxicity of the drug.

99-102. Regarding acetaminophen poisoning:

99. Acute ingestion of more than 7 grams in adults is Answer: A


considered toxic Starting 6 grams is already toxic

100. The antidote acetylcysteine which act as Answer: A


glutathione substitute Acetylcysteine is the antidote for Acetaminophen
toxicity, apparently it also acts as a glutathione
substitute hehehe nanghula lang ako.

101. Renal but hepatic injury is common Answer: B


Toxicity: hepatic

Answer: B
102. The toxic metabolite, NAPQUI is produced in
the kidneys NAPQI is produced in the liver. NSAID toxicity is
attributed to the Cytochrome p450 system,
remember Patho

103-106. The following are physiologic changes in the elderly (>70 years old) that may alter their drug
handling:

103. Increase in total body water Answer: B


Decreased TBW
Decreased lean body mass

104. Decrease in serum albumin concentration Answer: A


Therefore it will lead to a decrease in binding site
for acidic drugs. For alkaline drugs, binding will
increase due to increase level of alpha acid
glycoprotein

Abraham, M. | Aquino, J. | Arada, M. | Aw, A. | Bagon, N. | Contreras, S. | Galapon, C.


Gison, R. | Obciana, D. | Palao, K. | Tangcueco, P. | Tumambing, M.
139
PHARMACOLOGY RATIONALE
A.Y. 2017-2018

105. Decrease in hepatic blood flow Answer: A


Also with decrease in liver mass

Answer: B
106. Capacity of liver to metabolize all drugs decline
with age NOT ALL DRUGS, SOME drugs lang, and for the
aforementioned some drugs, refer page 3 of your
Geria handout.

107-110. Reasons for the high incidence of adverse drug reactions in elderly patients:

107. Failure of the physician to consider age-related Answer: A


pharmacokinetic changes

108. Pts noncompliance to treatment regimen Answer: A

109. Inability of the physician to recognize age- Answer: A


related diseases

110. Pts physical disability that hinders proper drug Answer: A


intake

Abraham, M. | Aquino, J. | Arada, M. | Aw, A. | Bagon, N. | Contreras, S. | Galapon, C.


Gison, R. | Obciana, D. | Palao, K. | Tangcueco, P. | Tumambing, M.
140

Вам также может понравиться